Anda di halaman 1dari 107

Fundamentals of Nursing Exam 9: Pharmacology (20 Items)

1. A 2 year-old child is receiving temporary total parental nutrition (TPN) through a central venous line. This is the first day of TPN
therapy. Although all of the following nursing actions must be included in the plan of care of this child, which one would be a
priority at this time?
a. Use aseptic technique during dressing changes
b. Maintain central line catheter integrity
c. Monitor serum glucose levels
d. Check results of liver function tests
2. Nurse Jamie is administering the initial total parenteral nutrition solution to a client. Which of the following assessments
requires the nurses immediate attention?
a. Temperature of 37.5 degrees Celsius
b. Urine output of 300 cc in 4 hours
c. Poor skin turgor
d. Blood glucose of 350 mg/dl
3. Nurse Susan administered intravenous gamma globulin to an 18 month-old child with AIDS. The parent asks why this
medication is being given. What is the nurses best response?
a. It will slow down the replication of the virus.
b. This medication will improve your childs overall health status.
c. This medication is used to prevent bacterial infections.
d. It will increase the effectiveness of the other medications your child receives.
4. When caring for a client with total parenteral nutrition (TPN), what is the most important action on the part of the nurse?
a. Record the number of stools per day
b. Maintain strict intake and output records
c. Sterile technique for dressing change at IV site
d. Monitor for cardiac arrhythmias
5. The nurse is administering an intravenous vesicant chemotherapeutic agent to a client. Which assessment would require the
nurses immediate action?
a. Stomatitis lesion in the mouth
b. Severe nausea and vomiting
c. Complaints of pain at site of infusion
d. A rash on the clients extremities
6. Nurse Celine is caring for a client with clinical depression who is receiving a MAO inhibitor. When providing instructions about
precautions with this medication, the nurse should instruct the client to:
a. Avoid chocolate and cheese
b. Take frequent naps
c. Take the medication with milk
d. Avoid walking without assistance
7. While providing home care to a client with congestive heart failure, the nurse is asked how long diuretics must be taken. The
BEST response to this client should be:
a. As you urinate more, you will need less medication to control fluid.
b. You will have to take this medication for about a year.
c. The medication must be continued so the fluid problem is controlled.
d. Please talk to your physician about medications and treatments.
8. George, age 8, is admitted with rheumatic fever. Which clinical finding indicates to the nurse that George needs to continue
taking the salicylates he had received at home?
a. Chorea.
b. Polyarthritis.
c. Subcutaneous nodules.
d. Erythema marginatum.
9. An order is written to start an IV on a 74-year-old client who is getting ready to go to the operating room for a total hip
replacement. What gauge of catheter would best meet the needs of this client?
a. 18
b. 20
c. 21 butterfly
d. 25
10. A client with an acute exacerbation of rheumatoid arthritis is admitted to the hospital for treatment. Which drug, used to
treat clients with rheumatoid arthritis, has both an anti-inflammatory and immunosuppressive effect?
a. Gold sodium thiomalate (Myochrysine)
b. Azathioprine (Imuran)
c. Prednisone (Deltasone)
d. Naproxen (Naprosyn)
11. Which of the following is least likely to influence the potential for a client to comply with lithium therapy after discharge?
a. The impact of lithium on the clients energy level and life-style.
b. The need for consistent blood level monitoring.
c. The potential side effects of lithium.
d. What the clients friends think of his need to take medication
12. Which of the following is least likely to influence the potential for a client to comply with lithium therapy after discharge?
a. The impact of lithium on the clients energy level and life-style.
b. The need for consistent blood level monitoring.
c. The potential side effects of lithium.
d. What the clients friends think of his need to take medication.
13. The nurse is caring for an elderly client who has been diagnosed as having sundown syndrome. He is alert and oriented during
the day but becomes disoriented and disruptive around dinnertime. He is hospitalized for evaluation. The nurse asks the client
and his family to list all of the medications, prescription and nonprescription, he is currently taking. What is the primary reason
for this action?
a. Multiple medications can lead to dementia
b. The medications can provide clues regarding his medical background
c. Ability to recall medications is a good assessment of the clients level of orientation.
d. Medications taken by a client are part of every nursing assessment.
14. A 25-year-old woman is in her fifth month of pregnancy. She has been taking 20 units of NPH insulin for diabetes mellitus daily
for six years. Her diabetes has been well controlled with this dosage. She has been coming for routine prenatal visits, during
which diabetic teaching has been implemented. Which of the following statements indicates that the woman understands the
teaching regarding her insulin needs during her pregnancy?
a. Are you sure all this insulin wont hurt my baby?
b. Ill probably need my daily insulin dose raised.
c. I will continue to take my regular dose of insulin.
d. These finger sticks make my hand sore. Can I do them less frequently?
15. Mrs. Johansons physician has prescribed tetracycline 500 mg po q6h. While assessing Mrs. Johansons nursing history for
allergies, the nurse notes that Mrs. Johansons is also taking oral contraceptives. What is the most appropriate initial nursing
intervention?
a. Administer the dose of tetracycline.
b. Notify the physician that Mrs. Johanson is taking oral contraceptives.
c. Tell Mrs. Johanson, she should stop taking oral contraceptives since they are inactivated by tetracycline.
d. Tell Mrs. Johanson, to use another form of birth control for at least two months.
16. An adult clients insulin dosage is 10 units of regular insulin and 15 units of NPH insulin in the morning. The client should be
taught to expect the first insulin peak:
a. as soon as food is ingested.
b. in two to four hours.
c. in six hours.
d. in ten to twelve hours.
17. An adult is hospitalized for treatment of deep electrical burns. Burn wound sepsis develops and mafenide acetate 10%
(Sulfamylon) is ordered bid. While applying the Sulfamylon to the wound, it is important for the nurse to prepare the client for
expected responses to the topical application, which include:
a. severe burning pain for a few minutes following application.
b. possible severe metabolic alkalosis with continued use.
c. black discoloration of everything that comes in contact with this drug.
d. chilling due to evaporation of solution from the moistened dressings.
18. Ms.Clark has hyperthyroidism and is scheduled for a thyroidectomy. The physician has ordered Lugols solution for the client.
The nurse understands that the primary reason for giving Lugols solution preoperatively is to:
a. decrease the risk of agranulocytosis postoperatively.
b. prevent tetany while the client is under general anesthesia.
c. reduce the size and vascularity of the thyroid and prevent hemorrhage.
d. potentiate the effect of the other preoperative medication so less medicine can be given while the client is under anesthesia.
19. A two-year-old child with congestive heart failure has been receiving digoxin for one week. The nurse needs to recognize that
an early sign of digitalis toxicity is:
a. bradypnea.
b. failure to thrive.
c. tachycardia.
d. vomiting.
20. Mr. Bates is admitted to the surgical ICU following a left adrenalectomy. He is sleepy but easily aroused. An IV containing
hydrocortisone is running. The nurse planning care for Mr. Bates knows it is essential to include which of the following nursing
interventions at this time?
a. Monitor blood glucose levels every shift to detect development of hypo- or hyperglycemia.
b. Keep flat on back with minimal movement to reduce risk of hemorrhage following surgery.
c. Administer hydrocortisone until vital signs stabilize, then discontinue the IV.
d. Teach Mr. Bates how to care for his wound since he is at high risk for developing postoperative infection.

Answers & Rationale
Here are the answers and rationale for this exam. Counter check your answers to those below and tell us your scores. If you have
any disputes or need more clarification to a certain question, please direct them to the comments section.
1. Answer C.
Monitor serum glucose levels. Hyperglycemia may occur during the first day or 2 as the child adapts to the high-glucose load of the
TPN solution. Thus, a chief nursing responsibility is blood glucose testing.
2. Answer D.
Total parenteral nutrition formulas contain dextrose in concentrations of 10% or greater to supply 20% to 50% of the total calories.
Blood glucose levels should be checked every 4 to 6 hours. A sliding scale dose of insulin may be ordered to maintain the blood
glucose level below 200mg/dl.
3. Answer C.
Intravenous gamma globulin is given to help prevent as well as to fight bacterial infections in young children with AIDS.
4. Answer C.
Clients receiving TPN are very susceptible to infection. The concentrated glucose solutions are a good medium for bacterial growth.
Strict sterile
technique is crucial in preventing infection at IV infusion site.
5. Answer C.
A vesicant is a chemotherapeutic agent capable of causing blistering of tissues and possible tissue necrosis if there is extravasation.
These agents are irritants which cause pain along the vein wall, with or without inflammation.
6. Answer A.
Foods high in tryptophan, tyramine and caffeine, such as chocolate and cheese may precipitate hypertensive crisis.
7. Answer C.
This is the most therapeutic response and gives the client accurate information.
8. Answer B.
Chorea is the restless and sudden aimless and irregular movements of the extremities suddenly seen in persons with rheumatic
fever, especially girls. Polyarthritis is characterized by swollen, painful, hot joints that respond to salicylates. Subcutaneous nodules
are nontender swellings over bony prominences sometimes seen in persons with rheumatic fever. Erythema marginatum is a skin
condition characterized by nonpruritic rash, affecting trunk and proximal extremities, seen in persons with rheumatic fever.
9. Answer A.
Clients going to the operating room ideally should have an 18- gauge catheter. This is large enough to handle blood products safely
and to allow rapid administration of large amounts of fluid if indicated during the perioperative period. An 18-gauge catheter is
recommended. A 20-gauge catheter is a second choice. A 21-gauge needle is too small and a butterfly too unstable for a client going
to surgery. A 25-gauge needle is too small.
10. Answer C.
Gold sodium thiomalate is usually used in combination with aspirin and nonsteroidal anti-inflammatory drugs to relieve pain. Gold
has an immunosuppressive affect. Azathioprine is used for clients with life-threatening rheumatoid arthritis for its
immunosuppressive effects. Prednisone is used to treat persons with acute exacerbations of rheumatoid arthritis. This medication is
given for its anti-inflammatory and immunosuppressive effects. Naproxen is a nonsteroidal anti-inflammatory drug.
Immunosuppression does not occur.
11. Answer D.
The impact of lithium on the clients energy level and life style are great determinants to compliance. The frequent blood level
monitoring required is difficult for clients to follow for a long period of time. Potential side effects such as fine tremor, drowsiness,
diarrhea, polyuria, thirst, weight gain, and fatigue can be disturbing to the client. While the clients social network can influence the
client in terms of compliance, the influence is typically secondary to that of the other factors listed.
12. Answer D.
The impact of lithium on the clients energy level and life style are great determinants to compliance. The frequent blood level
monitoring required is difficult for clients to follow for a long period of time. Potential side effects such as fine tremor, drowsiness,
diarrhea, polyuria, thirst, weight gain, and fatigue can be disturbing to the client. While the clients social network can influence the
client in terms of compliance, the influence is typically secondary to that of the other factors listed.
13. Answer A.
Drugs commonly used by elderly people, especially in combination, can lead to dementia. Assessment of the medication taken may
or may not provide information on the clients medical background. However, this is not the primary reason for assessing
medications in a client who is exhibiting sundown syndrome. Ability to recall medications may indicate short-term memory and
recall. However, that is not the primary reason for assessing medications in a client with sundown syndrome. Medication history
should be a part of the nursing assessment. In this client there is an even more important reason for evaluating the medications
taken.
14. Answer B.
The client starts to need increased insulin in the second trimester. This statement indicates a lack of understanding. As a result of
placental maturation and placental production of lactogen, insulin requirements begin increasing in the second trimester and may
double or quadruple by the end of pregnancy. The client starts to need increased insulin in the second trimester. This statement
indicates a lack of understanding. Insulin doses depend on blood glucose levels. Finger sticks for glucose levels must be continued.
15. Answer B.
The nurse should be aware that tetracyclines decrease the effectiveness of oral contraceptives. The physician should be notified. The
physician should be notified. Tetracycline decreases the effectiveness of oral contraceptives. There may be an equally effective
antibiotic available that can be prescribed. Note on the clients chart that the physician was notified. The nurse should be aware that
tetracyclines decrease the effectiveness of oral contraceptives. The nurse should not tell the client to stop taking oral contraceptives
unless the physician orders this. The nurse should be aware that tetracyclines decrease the effectiveness of oral contraceptives. If
the physician chooses to keep the client on tetracycline, the client should be encouraged to use another form of birth control. The
first intervention is to notify the physician.
16. Answer B.
The first insulin peak will occur two to four hours after administration of regular insulin. Regular insulin is classified as rapid acting
and will peak two to four hours after administration. The second peak will be eight to twelve hours after the administration of NPH
insulin. This is why a snack must be eaten mid-morning and also three to four hours after the evening meal. The first insulin peak will
occur two to four hours after administration of regular insulin. The first insulin peak will occur two to four hours after administration
of regular insulin. The second peak will occur eight to twelve hours after the administration of NPH insulin.
17. Answer A.
Mafenide acetate 10% (Sulfamylon) does cause burning on application. An analgesic may be required before the ointment is applied.
Mafenide acetate 10% (Sulfamylon) is a strong carbonic anhydrase inhibitor that affects the renal tubular buffering system, resulting
in metabolic acidosis. Mafenide acetate 10% (Sulfamylon) does not cause discoloration. Silver nitrate solution, another topical
antibiotic used to treat burn sepsis, has the disadvantage of turning everything it touches black. Mafenide acetate 10% (Sulfamylon)
is an ointment that is applied directly to the wound. It has the ability to diffuse rapidly through the eschar. The wound may be left
open or dry dressing may be applied. Silver nitrate solution is applied by soaking the wound dressings and keeping them constantly
wet, which may cause chilling and hypotension.
18. Answer C.
Doses of over 30 mg/day may increase the risk of agranulocytosis. Lugols solution does not act to prevent tetany. Calcium is used to
treat tetany. The client may receive iodine solution (Lugols solution) for 10 to 14 days before surgery to decrease vascularity of the
thyroid and thus prevent excess bleeding. Lugols solution does not potentiate any other preoperative medication.
19. Answer D.
Bradypnea (slow breathing) is not associated with digitalis toxicity. Bradycardia is associated with digitalis toxicity. Although children
with congestive heart failure often have a related condition of failure to thrive, it is not directly related to digitalis administration. It
is more related to chronic hypoxia. Tachycardia is not a sign of digitalis toxicity. Bradycardia is a sign of digitalis toxicity. The earliest
sign of digitalis toxicity is vomiting, although one episode does not warrant discontinuing medication.
20. Answer A.
Hydrocortisone promotes gluconeogenesis and elevates blood glucose levels. Following adrenalectomy the normal supply of
hydrocortisone is interrupted and must be replaced to maintain the blood glucose at normal levels. Care for the client following
adrenalectomy is similar to that for any abdominal operation. The client is encouraged to change position, cough, and deep breathe
to prevent postoperative complications such as pneumonia or thrombophlebitis. Maintenance doses of hydrocortisone will be
administered IV until the client is able to take it by mouth and will be necessary for six months to two years or until the remaining
gland recovers. The client undergoing an adrenalectomy is at increased risk for infection and delayed wound healing and will need to
learn about wound care, but not at this time while he is in the ICU.

Fundamentals of Nursing Exam 10: Pharmacology (20 Items)
1. The nursery nurse is putting erythromycin ointment in the newborns eyes to prevent infection. She places it in the following
area of the eye:
A. under the eyelid
B. on the cornea.
C. in the lower conjunctival sac
D. by the optic disc.
2. The physician orders penicillin for a patient with streptococcal pharyngitis. The nurse administers the drug as ordered, and the
patient has an allergic reaction. The nurse checks the medication order sheet and finds that the patient is allergic to penicillin.
Legal responsibility for the error is:
A. only the nursesshe should have checked the allergies before administering the medication.
B. only the physiciansshe gave the order, the nurse is obligated to follow it.
C. only the pharmacistshe should alert the floor to possible allergic reactions.
D. the pharmacist, physician, and nurse are all liable for the mistake
3. James Perez, a nurse on a geriatric floor, is administering a dose of digoxin to one of his patients. The woman asks why she
takes a different pill than her niece, who also has heart trouble. James replies that as people get older, liver and kidney function
decline, and if the dose is as high as her nieces, the drug will tend to:
A. have a shorter half-life.
B. accumulate.
C. have decreased distribution.
D. have increased absorption.
4. The nurse is administering augmentin to her patient with a sinus infection. Which is the best way for her to insure that she is
giving it to the right patient?
A. Call the patient by name
B. Read the name of the patient on the patients door
C. Check the patients wristband
D. Check the patients room number on the unit census list
5. The most important instructions a nurse can give a patient regarding the use of the antibiotic ampicillin prescribed for her are
to
A. call the physician if she has any breathing difficulties.
B. take it with meals so it doesnt cause an upset stomach.
C. take all of the medication prescribed even if the symptoms stop sooner.
D. not share the pills with anyone else.
6. Mr. Jessie Ray, a newly admitted patient, has a seizure disorder which is being treated with medication. Which of the following
drugs would the nurse question if ordered for him?
A. Phenobarbitol, 150 mg hs
B. Amitriptylene (Elavil), 10 mg QID.
C. Valproic acid (Depakote), 150 mg BID
D. Phenytoin (Dilantin), 100 mg TID
7. Mrs. Jane Gately has been dealing with uterine cancer for several months. Pain management is the primary focus of her current
admission to your oncology unit. Her vital signs on admission are BP 110/64, pulse 78, respirations 18, and temperature 99.2 F.
Morphine sulfate 6mg IV, q 4 hours, prn has been ordered. During your assessment after lunch, your findings are: BP 92/60, pulse
66, respirations 10, and temperature 98.8. Mrs. Gately is crying and tells you she is still experiencing severe pain. Your action
should be to
A. give her the next ordered dose of MS.
B. give her a back rub, put on some light music, and dim the lights in the room.
C. report your findings to the MD, requesting an alternate medication order
D. be obtained from the physician.
E. call her daughter to come and sit with her.
8. When counseling a patient who is starting to take MAO (monoamine oxidase) inhibitors such as Nardil for depression, it is
essential that they be warned not to eat foods containing tyramine, such as:
A. Roquefort, cheddar, or Camembert cheese.
B. grape juice, orange juice, or raisins.
C. onions, garlic, or scallions.
D. ground beef, turkey, or pork.
9. The physician orders an intramuscular injection of Demerol for the postoperativepatients pain. When preparing to draw up the
medication, the nurse is careful to remove the correct vial from the narcotics cabinet. It is labeled
A. simethicone.
B. albuterol.
C. meperidine.
D. ibuprofen.
10. The nurse is administering an antibiotic to her pediatric patient. She checks the patients armband and verifies the correct
medication by checking the physicians order, medication kardex, and vial. Which of the following is not considered one of the
five rights of drug administration?
A. Right dose
B. Right route
C. Right frequency
D. Right time
11. A nurse is preparing the clients morning NPH insulin dose and notices a clumpy precipitate inside the insulin vial. The nurse
should:
A. draw up and administer the dose
B. shake the vial in an attempt to disperse the clumps
C. draw the dose from a new vial
D. warm the bottle under running water to dissolve the clump
12. A client with histoplasmosis has an order for ketoconazole (Nizoral). The nurse teaches the client to do which of the following
while taking this medication?
A. take the medication on an empty stomach
B. take the medication with an antacid
C. avoid exposure to sunlight
D. limit alcohol to 2 ounces per day
13. A nurse has taught a client taking a xanthine bronchodilator about beverages to avoid. The nurse determines that the client
understands the information if the client chooses which of the following beverages from the dietary menu?
A. chocolate milk
B. cranberry juice
C. coffee
D. cola
14. A client is taking famotidine (Pepcid) asks the home care nurse what would be the best medication to take for a headache.
The nurse tells the client that it would be best to take:
A. aspirin (acetylsalicylic acid, ASA)
B. ibuprofen (Motrin)
C. acetaminophen (Tylenol)
D. naproxen (Naprosyn)
15. A nurse is planning dietary counseling for the client taking triamterene (Dyrenium). The nurse plans to include which of the
following in a list of foods that are acceptable?
A. baked potato
B. bananas
C. oranges
D. pears canned in water
16. A client with advanced cirrhosis of the liver is not tolerating protein well, as eveidenced by abnormal laboratory values. The
nurse anticipates that which of the following medications will be prescribed for the client?
A. lactulose (Chronulac)
B. ethacrynic acid (Edecrin)
C. folic acid (Folvite)
D. thiamine (Vitamin B1)
17. A female client tells the clinic nurse that her skin is very dry and irritated. Which product would the nurse suggest that the
client apply to the dry skin?
A. glycerin emollient
B. aspercreame
C. myoflex
D. acetic acid solution
18. A nurse is providing instructions to a client regarding quinapril hydrochloride (Accupril). The nurse tells the client:
A. to take the medication with food only
B. to rise slowly from a lying to a sitting position
C. to discontinue the medication if nausea occurs
D. that a therapeutic effect will be noted immediately
19. Auranofin (Ridaura) is prescribed for a client with rheumatoid arthritis, and the nurse monitors the client for signs of an
adverse effect related to the medication. Which of the following indicates an adverse effect?
A. nausea
B. diarrhea
C. anorexia
D. proteinuria
20. A client has been taking benzonatate (Tessalon) as ordered. The nurse tells the client that this medication should do which of
the following?
A. take away nausea and vomiting
B. calm the persistent cough
C. decrease anxiety level
D. increase comfort level
Answers & Rationale
Here are the answers and rationale for this exam. Counter check your answers to those below and tell us your scores. If you have
any disputes or need more clarification to a certain question, please direct them to the comments section.
1. C. The ointment is placed in the lower conjunctival sac so it will not scratch the eye itself and will get well distributed.
2. D. The physician, nurse, and pharmacist all are licensed professionals and share responsibility for errors.
3. B. The decreased circulation to the kidney and reduced liver function tend to allow drugs to accumulate and have toxic
effects.
4. C. The correct way to identify a patient before giving a medication is to check the name on the medication administration
record with the patients identification band. The nurse should also ask the patient to state their name. The name on the
door or the census list are not sufficient proof of identification. Calling the patient by name is not as effective as having the
patient state their name; patients may not hear well or understand what the nurse is saying, and may respond to a name
which is not their own.
5. C. Frequently patients do not complete an entire course of antibiotic therapy, and the bacteria are not destroyed.
6. B. Elavil is an antidepressant that lowers the seizure threshold, so would not be appropriate for this patient. The other
medications are anti-seizure drugs.
7. C. Morphine sulfate depresses the respiratory center. When the rate is less than 10, the MD should be notified.
8. A. Monoamine oxidase inhibitors react with foods high in the amino acid tyramine to cause dangerously high blood
pressure. Aged cheeses are all high in this amino acid; the other foods are not.
9. C. The generic name for Demerol is meperidine.
10. C. The five rights of medication administration are right drug, right dose, right route, right time, right patient. Frequency is
not included.
11. C. The nurse should always inspect the vial of insulin before use for solution changes that may signify loss of potency. NPH
insulin is normally uniformly cloudy. Clumping, frosting, and precipitates are signs of insulin damage. In this situation,
because potency is questionable, it is safer to discard the vial and draw up the dose from a new vial.
12. C. The client should be taught that ketoconazole is an antifungal medication. It should be taken with food or milk. Antacids
should be avoided for 2 hours after it is taken because gastric acid is needed to activate the medication. The client should
avoid concurrent use of alcohol, because the medication is hepatotoxic. The client should also avoid exposure to sunlight,
because the medication increases photosensitivity.
13. B. Cola, coffee, and chocolate contain xanthine and should be avoided by the client taking a xanthine bronchodilator. This
could lead to an increased incidence of cardiovascular and central nervous system side effects that can occur with the use
of these types of bronchodilators.
14. C. The client is taking famotidine, a histamine receptor antagonist. This implies that the client has a disorder characterized
by gastrointestinal (GI) irritation. The only medication of the ones listed in the options that is not irritating to the GI tract is
acetaminophen. The other medications could aggravate an already existing GI problem.
15. D. Triamterene is a potassium-sparing diuretic, and clients taking this medication should be cautioned against eating foods
that are high in potassium, including many vegetables, fruits, and fresh meats. Because potassium is very water-soluble,
foods that are prepared in water are often lower in potassium.
16. A. The client with cirrhosis has impaired ability to metabolize protein because of liver dysfunction. Administration of
lactulose aids in the clearance of ammonia via the gastrointestinal (GI) tract. Ethacrynic acid is a diuretic. Folic acid and
thiamine are vitamins, which may be used in clients with liver disease as supplemental therapy.
17. A. Glycerin is an emollient that is used for dry, cracked, and irritated skin. Aspercreme and Myoflex are used to treat
muscular aches. Acetic acid solution is used for irrigating, cleansing, and packing wounds infected by Pseudomonas
aeruginosa.
18. B. Accupril is an angiotensin-converting enzyme (ACE) inhibitor. It is used in the treatment of hypertension. The client
should be instructed to rise slowly from a lying to sitting position and to permit the legs to dangle from the bed
momentarily before standing to reduce the hypotensive effect. The medication does not need to be taken with meals. It
may be given without regard to food. If nausea occurs, the client should be instructed to take a non cola carbonated
beverage and salted crackers or dry toast. A full therapeutic effect may be noted in 1 to 2 weeks.
19. D. Auranofin (Ridaura) is a gold preparation that is used as an antirheumatic. Gold toxicity is an adverse effect and is
evidenced by decreased hemoglobin, leukopenia, reduced granulocyte counts, proteinuria, hematuria, stomatitis,
glomerulonephritis, nephrotic syndrome, or cholestatic jaundice. Anorexia, nausea, and diarrhea are frequent side effects
of the medication.
20. B. Benzonatate is a locally acting antitussive. Its effectiveness is measured by the degree to which it decreases the intensity
and frequency of cough, without eliminating the cough reflex.

Fundamentals of Nursing Exam 11: Pharmacology (30 Items)
1. An infection in a central venous access device is not eliminated by giving antibiotics through the catheter. How would bacterial
glycocalyx contribute to this?
a. It protects the bacteria from antibiotic and immunologic destruction.
b. Glycocalyx neutralizes the antibiotic rendering it ineffective.
c. It competes with the antibiotic for binding sites on the microbe.
d. Glycocalyx provides nutrients for microbial growth.
2. Central venous access devices are beneficial in pediatric therapy because:
a. They dont frighten children.
b. Use of the arms is not restricted.
c. They cannot be dislodged.
d. They are difficult to see.
3. How can central venous access devices (CVADs) be of value in a patient receiving chemotherapy who has stomatitis and severe
diarrhea?
a. The chemotherapy can be rapidly completed allowing the stomatitis and diarrhea to resolve.
b. Crystalloid can be administered to prevent dehydration.
c. Concentrated hyperalimentation fluid can be administered through the CVAD.
d. The chemotherapy dose can be reduced.
4. Some central venous access devices (CVAD) have more than one lumen. These multi lumen catheters:
a. Have an increased risk of infiltration.
b. Only work a short while because the small bore clots off.
c. Are beneficial to patient care but are prohibitively expensive.
d. Allow different medications or solutions to be administered simultaneously.
5. Some institutions will not infuse a fat emulsion, such as Intralipid, into central venous access devices (CVAD) because:
a. Lipid residue may accumulate in the CVAD and occlude the catheter.
b. If the catheter clogs, there is no treatment other than removal and replacement.
c. Lipids are necessary only in the most extreme cases to prevent essential fatty acid (EFA) deficiency.
d. Fat emulsions are very caustic.
6. A male patient needs a percutaneously inserted central catheter (PICC) for prolonged IV therapy. He knows it can be inserted
without going to the operating room. He mentions that, at least the doctor wont be wearing surgical garb, will he? How will
the nurse answer the patient?
a. You are correct. It is a minor procedure performed on the unit and does not necessitate surgical attire.
b. To decrease the risk of infection, the doctor inserting the PICC will wear a cap, mask, and sterile gown and gloves.
c. It depends on the doctors preference.
d. Most doctors only wear sterile gloves, not a cap, mask, or sterile gown.
7. A male patient is to receive a percutaneously inserted central catheter (PICC). He asks the nurse whether the insertion will hurt.
How will the nurse reply?
a. You will have general anesthesia so you wont feel anything.
b. It will be inserted rapidly, and any discomfort is fleeting.
c. The insertion site will be anesthetized. Threading the catheter through the vein is not painful.
d. You will receive sedation prior to the procedure.
8. What volume of air can safely be infused into a patient with a central venous access device (CVAD)?
a. It is dependent on the patients weight and height.
b. Air entering the patient through a CVAD will follow circulation to the lungs where it will be absorbed and cause no problems.
c. It is dependent on comorbidities such as asthma or chronic obstructive lung disease.
d. None.
9. Kent a new staff nurse asks her preceptor nurse how to obtain a blood sample from a patient with a portacath device. The
preceptor nurse teaches the new staff nurse:
a. The sample will be withdrawn into a syringe attached to the portacath needle and then placed into a vacutainer.
b. Portacath devices are not used to obtain blood samples because of the risk of clot formation.
c. The vacutainer will be attached to the portacath needle to obtain a direct sample.
d. Any needle and syringe may be utilized to obtain the sample.
10. What is the purpose of tunneling (inserting the catheter 2-4 inches under the skin) when the surgeon inserts a Hickman
central catheter device? Tunneling:
a. Increases the patients comfort level.
b. Decreases the risk of infection.
c. Prevents the patients clothes from having contact with the catheter
d. Makes the catheter less visible to other people.
11. The primary complication of a central venous access device (CVAD) is:
a. Thrombus formation in the vein.
b. Pain and discomfort.
c. Infection.
d. Occlusion of the catheter as the result of an intra-lumen clot.
12. Nurse Blessy is doing some patient education related to a patients central venous access device. Which of the following
statements will the nurse make to the patient?
a. These type of devices are essentially risk free.
b. These devices seldom work for more than a week or two necessitating replacement.
c. The dressing should only the changed by your doctor.
d. Heparin in instilled into the lumen of the catheter to decrease the risk of clotting.
13. The chemotherapeutic DNA alkylating agents such as nitrogen mustards are effective because they:
a. Cross-link DNA strands with covalent bonds between alkyl groups on the drug and guanine bases on DNA.
b. Have few, if any, side effects.
c. Are used to treat multiple types of cancer.
d. Are cell cycle-specific agents.
14. Hormonal agents are used to treat some cancers. An example would be:
a. Thyroxine to treat thyroid cancer.
b. ACTH to treat adrenal carcinoma.
c. Estrogen antagonists to treat breast cancer.
d. Glucagon to treat pancreatic carcinoma.
15. Chemotherapeutic agents often produce a certain degree of myelosuppression including leukopenia. Leukopenia does not
present immediately but is delayed several days to weeks because:
a. The patients hemoglobin and hematocrit are normal.
b. Red blood cells are affected first.
c. Folic acid levels are normal.
d. The current white cell count is not affected by chemotherapy.
16. Currently, there is no way to prevent myelosuppression. However, there are medications available to elicit a more rapid bone
marrow recovery. An example is:
a. Epoetin alfa (Epogen, Procrit).
b. Glucagon.
c. Fenofibrate (Tricor).
d. Lamotrigine (Lamictal).
17. Estrogen antagonists are used to treat estrogen hormone-dependent cancer, such as breast carcinoma. Androgen antagonists
block testosterone stimulation of androgen-dependent cancers. An example of an androgen-dependent cancer would be:
a. Prostate cancer.
b. Thyroid cancer.
c. Renal carcinoma.
d. neuroblastoma.
18. Serotonin release stimulates vomiting following chemotherapy. Therefore, serotonin antagonists are effective in preventing
and treating nausea and vomiting related to chemotherapy. An example of an effective serotonin antagonist antiemetic is:
a. ondansetron (Zofran).
b. fluoxetine (Prozac).
c. paroxetine (Paxil).
d. sertraline (Zoloft).
19. Methotrexate, the most widely used antimetabolite in cancer chemotherapy does not penetrate the central nervous system
(CNS). To treat CNS disease this drug must be administered:
a. Intravenously.
b. Subcutaneously.
c. Intrathecally.
d. By inhalation.
20. Methotrexate is a folate antagonist. It inhibits enzymes required for DNA base synthesis. To prevent harm to normal cells, a
fully activated form of folic acid known as leucovorin (folinic acid; citrovorum factor) can be administered. Administration of
leucovorin is known as:
a. Induction therapy.
b. Consolidation therapy.
c. Pulse therapy.
d. Rescue therapy.
21. A male Patient is undergoing chemotherapy may also be given the drug allopurinol (Zyloprim, Aloprim). Allopurinol inhibits
the synthesis of uric acid. Concomitant administration of allopurinol prevents:
a. Myelosuppression.
b. Gout and hyperuricemia.
c. Pancytopenia.
d. Cancer cell growth and replication
22. Superficial bladder cancer can be treated by direct instillation of the antineoplastic antibiotic agent mitomycin (Mutamycin).
This process is termed:
a. Intraventricular administration.
b. Intravesical administration.
c. Intravascular administration.
d. Intrathecal administration.
23. The most common dose-limiting toxicity of chemotherapy is:
a. Nausea and vomiting.
b. Bloody stools.
c. Myelosuppression.
d. Inability to ingest food orally due to stomatitis and mucositis.
24. Chemotherapy induces vomiting by:
a. Stimulating neuroreceptors in the medulla.
b. Inhibiting the release of catecholamines.
c. Autonomic instability.
d. Irritating the gastric mucosa.
25. Myeloablation using chemotherapeutic agents is useful in cancer treatment because:
a. It destroys the myelocytes (muscle cells).
b. It reduces the size of the cancer tumor.
c. After surgery, it reduces the amount of chemotherapy needed.
d. It destroys the bone marrow prior to transplant.
26. Anticipatory nausea and vomiting associated with chemotherapy occurs:
a. Within the first 24 hours after chemotherapy.
b. 1-5 days after chemotherapy.
c. Before chemotherapy administration.
d. While chemotherapy is being administered.
27. Medications bound to protein have the following effect:
a. Enhancement of drug availability.
b. Rapid distribution of the drug to receptor sites.
c. The more drug bound to protein, the less available for desired effect.
d. Increased metabolism of the drug by the liver.
28. Some drugs are excreted into bile and delivered to the intestines. Prior to elimination from the body, the drug may be
absorbed. This process is known as:
a. Hepatic clearance.
b. Total clearance.
c. Enterohepatic cycling.
d. First-pass effect.
29. An adult patient has been taking a drug (Drug A) that is highly metabolized by the cytochrome p-450 system. He has been on
this medication for 6 months. At this time, he is started on a second medication (Drug B) that is an inducer of the cytochrome p-
450 system. You should monitor this patient for:
a. Increased therapeutic effects of Drug A.
b. Increased adverse effects of Drug B.
c. Decreased therapeutic effects of Drug A.
d. Decreased therapeutic effects of Drug B.
30. Epinephrine is administered to a female patient. The nurse should expect this agent to rapidly affect:
a. Adrenergic receptors.
b. Muscarinic receptors.
c. Cholinergic receptors.
d. Nicotinic receptors.
Answers & Rationale
Here are the answers and rationale for this exam. Counter check your answers to those below and tell us your scores. If you have
any disputes or need more clarification to a certain question, please direct them to the comments section.
1. Answer C. Glycocalyx is a viscous polysaccharide or polypeptide slime that covers microbes. It enhances adherence to
surfaces, resists phagocytic engulfment by the white blood cells, and prevents antibiotics from contacting the microbe.
Glycocalyx does not have the effects in options B-D.
2. Answer B. The child can move his extremities and function in a normal fashion. This lessens stress associated with position
restriction and promotes normal activity. Fear may not be eliminated. All lines can be dislodged. Even small catheters can
be readily seen.
3. Answer C. In patients unable to take oral nutrition, parenteral hyperalimentation is an option for providing nutritional
support. High concentrations of dextrose, protein, minerals, vitamins, and trace elements can be provided. Dosing is not
affected with options a and d. Crystalloid can provide free water but has very little nutritional benefits. Hyperalimentation
can provide free water and considerable nutritional benefits.
4. Answer D. A multilumen catheter contains separate ports and means to administer agents. An agent infusing in one port
cannot mix with an agent infusing into another port. Thus, agents that would be incompatible if given together can be given
in separate ports simultaneously.
5. Answer A. Occlusion occurs with slow infusion rates and concurrent administration of some medications. Lipid occlusions
may be treated with 70 percent ethanol or with 0.1 mmol/mL NaOH. Lipids provide essential fatty acids. It is recommended
that approximately 4 percent of daily calories be EFAs. A deficiency can quickly develop. Daily essential fatty acids are
necessary for constant prostaglandin production. Lipids are almost isotonic with blood.
6. Answer B. Strict aseptic technique including the use of cap, mask, and sterile gown and gloves is require when placing a
central venous line including a PICC. Options A, C, and D are incorrect statements. They increase the risk of infection.
7. Answer C. Pain related to PICC insertion occurs with puncture of the skin. When inserting PICC lines, the insertion site is
anesthetized so no pain is felt. The patient will not receive general anesthesia or sedation. Statement 2 is false.
Unnecessary pain should be prevented.
8. Answer D. Any air entering the right heart can lead to a pulmonary embolus. All air should be purged from central venous
lines; none should enter the patient.
9. Answer A. A special portacath needle is used to access the portacath device. A syringe is attached and the sample is
obtained. One of the primary reasons for insertion of a portacath device is the need for frequent or long-term blood
sampling. A vacutainer will exert too much suction on the central line resulting in collapse of the line. Only special portacath
needles should be used to access the portacath device.
10. Answer B. The actual access to the subclavian vein is still just under the clavicle, but by tunneling the distal portion of the
catheter several inches under the skin the risk of migratory infection is reduces compared to a catheter that enters the
subclavian vein directly and is not tunneled. The catheter is tunneled to prevent infection.
11. Answer C. A foreign body in a blood vessel increases the risk of infection. Catheters that come outside the body have an
even higher risk of infection. Most infections are caused by skin bacteria. Other infective organisms include yeasts and
fungi. Options 1 and 4 are complications of a CVAD but are not the primary problem. Once placed, these lines do not cause
pain and discomfort.
12. Answer D. A solution containing heparin is used to reduce catheter clotting and maintain patency. The concentration of
heparin used depends on the patients age, comorbidities, and the frequency of catheter access/flushing. Although patients
have few complications, the device is not risk free. Patients may develop infection, catheter clots, vascular obstruction,
pneumothorax, hemothorax, or mechanical problems (catheter breakage). Strict adherence to protocol enhances the
longevity of central access devices. They routinely last weeks to months and sometimes years. The patient will be taught
how to perform dressing changes at home.
13. Answer A. Alkylating agents are highly reactive chemicals that introduce alkyl radicals into biologically active molecules and
thereby prevent their proper functioning, replication, and transcription. Alkylating agents have numerous side effects
including alopecia, nausea, vomiting, and myelosuppression. Nitrogen mustards have a broad spectrum of activity against
chronic lymphocytic leukemia, non-Hodgkins lymphoma, and breast and ovarian cancer, but they are effective
chemotherapeutic agents because of DNA cross-linkage. Alkylating agents are noncell cycle-specific agents.
14. Answer C. Estrogen antagonists are used to treat estrogen hormone-dependent cancer, such as breast carcinoma. A well-
known estrogen antagonist used in breast cancer therapy is tamoxifen (Nolvadex). This drug, in combination with surgery
and other chemotherapeutic drugs reduces breast cancer recurrence by 30 percent. Estrogen antagonists can also be
administered to prevent breast cancer in women who have a strong family history of the disease. Thyroxine is a natural
thyroid hormone. It does not treat thyroid cancer. ACTH is an anterior pituitary hormone, which stimulates the adrenal
glands to release glucocorticoids. It does not treat adrenal cancer. Glucagon is a pancreatic alpha cell hormone, which
stimulates glycogenolysis and gluconeogenesis. It does not treat pancreatic cancer.
15. Answer D. The time required to clear circulating cells before the effect that chemotherapeutic drugs have on precursor cell
maturation in the bone marrow becomes evident. Leukopenia is an abnormally low white blood cell count. Answers A-C
pertain to red blood cells.
16. Answer A. Epoetin alfa (Epogen, Procrit) is a recombinant form of endogenous erythropoietin, a hematopoietic growth
factor normally produced by the kidney that is used to induce red blood cell production in the bone marrow and reduce the
need for blood transfusion. Glucagon is a pancreatic alpha cell hormone, which cause glycogenolysis and gluconeogenesis.
Fenofibrate (Tricor) is an antihyperlipidemic agent that lowers plasma triglycerides. Lamotrigine (Lamictal) is an
anticonvulsant.
17. Answer A. Prostate tissue is stimulated by androgens and suppressed by estrogens. Androgen antagonists will block
testosterone stimulation of prostate carcinoma cells. The types of cancer in options 2-4 are not androgen dependent.
18. Answer A. Chemotherapy often induces vomiting centrally by stimulating the chemoreceptor trigger zone (CTZ) and
peripherally by stimulating visceral afferent nerves in the GI tract. Ondansetron (Zofran) is a serotonin antagonist that bocks
the effects of serotonin and prevents and treats nausea and vomiting. It is especially useful in single-day highly emetogenic
cancer chemotherapy (for example, cisplatin). The agents in options 2-4 are selective serotonin reuptake inhibitors. They
increase the available levels of serotonin.
19. Answer C. With intrathecal administration chemotherapy is injected through the theca of the spinal cord and into the
subarachnoid space entering into the cerebrospinal fluid surrounding the brain and spinal cord. The methods in options A,
B, and D are ineffective because the medication cannot enter the CNS.
20. Answer B. Leucovorin is used to save or rescue normal cells from the damaging effects of chemotherapy allowing them to
survive while the cancer cells die. Therapy to rapidly reduce the number of cancerous cells is the induction phase.
Consolidation therapy seeks to complete or extend the initial remission and often uses a different combination of drugs
than that used for induction. Chemotherapy is often administered in intermittent courses called pulse therapy. Pulse
therapy allows the bone marrow to recover function before another course of chemotherapy is given.
21. Answer B. Prevent uric acid nephropathy, uric acid lithiasis, and gout during cancer therapy since chemotherapy causes the
rapid destruction of cancer cells leading to excessive purine catabolism and uric acid formation. Allopurinol can induce
myelosuppression and pancytopenia. Allopurinol does not have this function.
22. Answer B. Medications administered intravesically are instilled into the bladder. Intraventricular administration involves
the ventricles of the brain. Intravascular administration involves blood vessels. Intrathecal administration involves the fluid
surrounding the brain and spinal cord.
23. Answer C. The overall goal of cancer chemotherapy is to give a dose large enough to be lethal to the cancer cells, but small
enough to be tolerable for normal cells. Unfortunately, some normal cells are affected including the bone marrow.
Myelosuppression limits the bodys ability to prevent and fight infection, produce platelets for clotting, and manufacture
red blood cells for oxygen portage. Even though the effects in options a, b, and d are uncomfortable and distressing to the
patient, they do not have the potential for lethal outcomes that myelosuppression has.
24. Answer A. Vomiting (emesis) is initiated by a nucleus of cells located in the medulla called the vomiting center. This center
coordinates a complex series of events involving pharyngeal, gastrointestinal, and abdominal wall contractions that lead to
expulsion of gastric contents. Catecholamine inhibition does not induce vomiting. Chemotherapy does not induce vomiting
from autonomic instability. Chemotherapy, especially oral agents, may have an irritating effect on the gastric mucosa,
which could result in afferent messages to the solitary tract nucleus, but these pathways do not project to the vomiting
center.
25. Answer A. Myelo comes from the Greek word myelos, which means marrow. Ablation comes from the Latin word ablatio,
which means removal. Thus, myeloablative chemotherapeurtic agents destroy the bone marrow. This procedure destroys
normal bone marrow as well as the cancerous marrow. The patients bone marrow will be replaced with a bone marrow
transplant. Myelocytes are not muscle cells Tumors are solid masses typically located in organs. Surgery may be performed
to reduce tumor burden and require less chemotherapy afterward.
26. Answer C. Nausea and vomiting (N&V) are common side effects of chemotherapy. Some patients are able to trigger these
events prior to actually receiving chemotherapy by anticipating, or expecting, to have these effects. N&V occurring post-
chemotherapeutic administration is not an anticipatory event but rather an effect of the drug. N&V occurring during the
administration of chemotherapy is an effect of the drug.
27. Answer C. Only an unbound drug can be distributed to active receptor sites. Therefore, the more of a drug that is bound to
protein, the less it is available for the desired drug effect. Less drug is available if bound to protein. Distribution to receptor
sites is irrelevant since the drug bound to protein cannot bind with a receptor site. Metabolism would not be increased. The
liver will first have to remove the drug from the protein molecule before metabolism can occur. The protein is then free to
return to circulation and be used again.
28. Answer C. Drugs and drug metabolites with molecular weights higher than 300 may be excreted via the bile, stored in the
gallbladder, delivered to the intestines by the bile duct, and then reabsorbed into the circulation. This process reduces the
elimination of drugs and prolongs their half-life and duration of action in the body. Hepatic clearance is the amount of drug
eliminated by the liver. Total clearance is the sum of all types of clearance including renal, hepatic, and respiratory. First-
pass effect is the amount of drug absorbed from the GI tract and then metabolized by the liver; thus, reducing the amount
of drug making it into circulation.
29. Answer C. Drug B will induce the cytochrome p-450 enzyme system of the liver; thus, increasing the metabolism of Drug A.
Therefore, Drug A will be broken down faster and exert decreased therapeutic effects. Drug A will be metabolized faster,
thus reducing, not increasing its therapeutic effect. Inducing the cytochrome p-450 system will not increase the adverse
effects of Drug B. Drug B induces the cytochrome p-450 system but is not metabolized faster. Thus, the therapeutic effects
of Drug B will not be decreased.
30. Answer A. Epinephrine (adrenaline) rapidly affects both alpha and beta adrenergic receptors eliciting a sympathetic (fight
or flight) response. Muscarinic receptors are cholinergic receptors and are primarily located at parasympathetic junctions.
Cholinergic receptors respond to acetylcholine stimulation. Cholinergic receptors include muscarinic and nicotinic
receptors. Nicotinic receptors are cholinergic receptors activated by nicotine and found in autonomic ganglia and somatic
neuromuscular junctions.








Fundamentals of Nursing Exam 11: Pharmacology (30 Items)
InText Mode: All questions and answers are given for reading and answering at your own pace. You can also copy this exam and
make a print out.
1. An infection in a central venous access device is not eliminated by giving antibiotics through the catheter. How would bacterial
glycocalyx contribute to this?
a. It protects the bacteria from antibiotic and immunologic destruction.
b. Glycocalyx neutralizes the antibiotic rendering it ineffective.
c. It competes with the antibiotic for binding sites on the microbe.
d. Glycocalyx provides nutrients for microbial growth.
2. Central venous access devices are beneficial in pediatric therapy because:
a. They dont frighten children.
b. Use of the arms is not restricted.
c. They cannot be dislodged.
d. They are difficult to see.
3. How can central venous access devices (CVADs) be of value in a patient receiving chemotherapy who has stomatitis and severe
diarrhea?
a. The chemotherapy can be rapidly completed allowing the stomatitis and diarrhea to resolve.
b. Crystalloid can be administered to prevent dehydration.
c. Concentrated hyperalimentation fluid can be administered through the CVAD.
d. The chemotherapy dose can be reduced.
4. Some central venous access devices (CVAD) have more than one lumen. These multi lumen catheters:
a. Have an increased risk of infiltration.
b. Only work a short while because the small bore clots off.
c. Are beneficial to patient care but are prohibitively expensive.
d. Allow different medications or solutions to be administered simultaneously.
5. Some institutions will not infuse a fat emulsion, such as Intralipid, into central venous access devices (CVAD) because:
a. Lipid residue may accumulate in the CVAD and occlude the catheter.
b. If the catheter clogs, there is no treatment other than removal and replacement.
c. Lipids are necessary only in the most extreme cases to prevent essential fatty acid (EFA) deficiency.
d. Fat emulsions are very caustic.
6. A male patient needs a percutaneously inserted central catheter (PICC) for prolonged IV therapy. He knows it can be inserted
without going to the operating room. He mentions that, at least the doctor wont be wearing surgical garb, will he? How will
the nurse answer the patient?
a. You are correct. It is a minor procedure performed on the unit and does not necessitate surgical attire.
b. To decrease the risk of infection, the doctor inserting the PICC will wear a cap, mask, and sterile gown and gloves.
c. It depends on the doctors preference.
d. Most doctors only wear sterile gloves, not a cap, mask, or sterile gown.
7. A male patient is to receive a percutaneously inserted central catheter (PICC). He asks the nurse whether the insertion will hurt.
How will the nurse reply?
a. You will have general anesthesia so you wont feel anything.
b. It will be inserted rapidly, and any discomfort is fleeting.
c. The insertion site will be anesthetized. Threading the catheter through the vein is not painful.
d. You will receive sedation prior to the procedure.
8. What volume of air can safely be infused into a patient with a central venous access device (CVAD)?
a. It is dependent on the patients weight and height.
b. Air entering the patient through a CVAD will follow circulation to the lungs where it will be absorbed and cause no problems.
c. It is dependent on comorbidities such as asthma or chronic obstructive lung disease.
d. None.
9. Kent a new staff nurse asks her preceptor nurse how to obtain a blood sample from a patient with a portacath device. The
preceptor nurse teaches the new staff nurse:
a. The sample will be withdrawn into a syringe attached to the portacath needle and then placed into a vacutainer.
b. Portacath devices are not used to obtain blood samples because of the risk of clot formation.
c. The vacutainer will be attached to the portacath needle to obtain a direct sample.
d. Any needle and syringe may be utilized to obtain the sample.
10. What is the purpose of tunneling (inserting the catheter 2-4 inches under the skin) when the surgeon inserts a Hickman
central catheter device? Tunneling:
a. Increases the patients comfort level.
b. Decreases the risk of infection.
c. Prevents the patients clothes from having contact with the catheter
d. Makes the catheter less visible to other people.
11. The primary complication of a central venous access device (CVAD) is:
a. Thrombus formation in the vein.
b. Pain and discomfort.
c. Infection.
d. Occlusion of the catheter as the result of an intra-lumen clot.
12. Nurse Blessy is doing some patient education related to a patients central venous access device. Which of the following
statements will the nurse make to the patient?
a. These type of devices are essentially risk free.
b. These devices seldom work for more than a week or two necessitating replacement.
c. The dressing should only the changed by your doctor.
d. Heparin in instilled into the lumen of the catheter to decrease the risk of clotting.
13. The chemotherapeutic DNA alkylating agents such as nitrogen mustards are effective because they:
a. Cross-link DNA strands with covalent bonds between alkyl groups on the drug and guanine bases on DNA.
b. Have few, if any, side effects.
c. Are used to treat multiple types of cancer.
d. Are cell cycle-specific agents.
14. Hormonal agents are used to treat some cancers. An example would be:
a. Thyroxine to treat thyroid cancer.
b. ACTH to treat adrenal carcinoma.
c. Estrogen antagonists to treat breast cancer.
d. Glucagon to treat pancreatic carcinoma.
15. Chemotherapeutic agents often produce a certain degree of myelosuppression including leukopenia. Leukopenia does not
present immediately but is delayed several days to weeks because:
a. The patients hemoglobin and hematocrit are normal.
b. Red blood cells are affected first.
c. Folic acid levels are normal.
d. The current white cell count is not affected by chemotherapy.
16. Currently, there is no way to prevent myelosuppression. However, there are medications available to elicit a more rapid bone
marrow recovery. An example is:
a. Epoetin alfa (Epogen, Procrit).
b. Glucagon.
c. Fenofibrate (Tricor).
d. Lamotrigine (Lamictal).
17. Estrogen antagonists are used to treat estrogen hormone-dependent cancer, such as breast carcinoma. Androgen antagonists
block testosterone stimulation of androgen-dependent cancers. An example of an androgen-dependent cancer would be:
a. Prostate cancer.
b. Thyroid cancer.
c. Renal carcinoma.
d. neuroblastoma.
18. Serotonin release stimulates vomiting following chemotherapy. Therefore, serotonin antagonists are effective in preventing
and treating nausea and vomiting related to chemotherapy. An example of an effective serotonin antagonist antiemetic is:
a. ondansetron (Zofran).
b. fluoxetine (Prozac).
c. paroxetine (Paxil).
d. sertraline (Zoloft).
19. Methotrexate, the most widely used antimetabolite in cancer chemotherapy does not penetrate the central nervous system
(CNS). To treat CNS disease this drug must be administered:
a. Intravenously.
b. Subcutaneously.
c. Intrathecally.
d. By inhalation.
20. Methotrexate is a folate antagonist. It inhibits enzymes required for DNA base synthesis. To prevent harm to normal cells, a
fully activated form of folic acid known as leucovorin (folinic acid; citrovorum factor) can be administered. Administration of
leucovorin is known as:
a. Induction therapy.
b. Consolidation therapy.
c. Pulse therapy.
d. Rescue therapy.
21. A male Patient is undergoing chemotherapy may also be given the drug allopurinol (Zyloprim, Aloprim). Allopurinol inhibits
the synthesis of uric acid. Concomitant administration of allopurinol prevents:
a. Myelosuppression.
b. Gout and hyperuricemia.
c. Pancytopenia.
d. Cancer cell growth and replication
22. Superficial bladder cancer can be treated by direct instillation of the antineoplastic antibiotic agent mitomycin (Mutamycin).
This process is termed:
a. Intraventricular administration.
b. Intravesical administration.
c. Intravascular administration.
d. Intrathecal administration.
23. The most common dose-limiting toxicity of chemotherapy is:
a. Nausea and vomiting.
b. Bloody stools.
c. Myelosuppression.
d. Inability to ingest food orally due to stomatitis and mucositis.
24. Chemotherapy induces vomiting by:
a. Stimulating neuroreceptors in the medulla.
b. Inhibiting the release of catecholamines.
c. Autonomic instability.
d. Irritating the gastric mucosa.
25. Myeloablation using chemotherapeutic agents is useful in cancer treatment because:
a. It destroys the myelocytes (muscle cells).
b. It reduces the size of the cancer tumor.
c. After surgery, it reduces the amount of chemotherapy needed.
d. It destroys the bone marrow prior to transplant.
26. Anticipatory nausea and vomiting associated with chemotherapy occurs:
a. Within the first 24 hours after chemotherapy.
b. 1-5 days after chemotherapy.
c. Before chemotherapy administration.
d. While chemotherapy is being administered.
27. Medications bound to protein have the following effect:
a. Enhancement of drug availability.
b. Rapid distribution of the drug to receptor sites.
c. The more drug bound to protein, the less available for desired effect.
d. Increased metabolism of the drug by the liver.
28. Some drugs are excreted into bile and delivered to the intestines. Prior to elimination from the body, the drug may be
absorbed. This process is known as:
a. Hepatic clearance.
b. Total clearance.
c. Enterohepatic cycling.
d. First-pass effect.
29. An adult patient has been taking a drug (Drug A) that is highly metabolized by the cytochrome p-450 system. He has been on
this medication for 6 months. At this time, he is started on a second medication (Drug B) that is an inducer of the cytochrome p-
450 system. You should monitor this patient for:
a. Increased therapeutic effects of Drug A.
b. Increased adverse effects of Drug B.
c. Decreased therapeutic effects of Drug A.
d. Decreased therapeutic effects of Drug B.
30. Epinephrine is administered to a female patient. The nurse should expect this agent to rapidly affect:
a. Adrenergic receptors.
b. Muscarinic receptors.
c. Cholinergic receptors.
d. Nicotinic receptors.
Answers & Rationale
Here are the answers and rationale for this exam. Counter check your answers to those below and tell us your scores. If you have
any disputes or need more clarification to a certain question, please direct them to the comments section.
1. Answer C. Glycocalyx is a viscous polysaccharide or polypeptide slime that covers microbes. It enhances adherence to
surfaces, resists phagocytic engulfment by the white blood cells, and prevents antibiotics from contacting the microbe.
Glycocalyx does not have the effects in options B-D.
2. Answer B. The child can move his extremities and function in a normal fashion. This lessens stress associated with position
restriction and promotes normal activity. Fear may not be eliminated. All lines can be dislodged. Even small catheters can
be readily seen.
3. Answer C. In patients unable to take oral nutrition, parenteral hyperalimentation is an option for providing nutritional
support. High concentrations of dextrose, protein, minerals, vitamins, and trace elements can be provided. Dosing is not
affected with options a and d. Crystalloid can provide free water but has very little nutritional benefits. Hyperalimentation
can provide free water and considerable nutritional benefits.
4. Answer D. A multilumen catheter contains separate ports and means to administer agents. An agent infusing in one port
cannot mix with an agent infusing into another port. Thus, agents that would be incompatible if given together can be given
in separate ports simultaneously.
5. Answer A. Occlusion occurs with slow infusion rates and concurrent administration of some medications. Lipid occlusions
may be treated with 70 percent ethanol or with 0.1 mmol/mL NaOH. Lipids provide essential fatty acids. It is recommended
that approximately 4 percent of daily calories be EFAs. A deficiency can quickly develop. Daily essential fatty acids are
necessary for constant prostaglandin production. Lipids are almost isotonic with blood.
6. Answer B. Strict aseptic technique including the use of cap, mask, and sterile gown and gloves is require when placing a
central venous line including a PICC. Options A, C, and D are incorrect statements. They increase the risk of infection.
7. Answer C. Pain related to PICC insertion occurs with puncture of the skin. When inserting PICC lines, the insertion site is
anesthetized so no pain is felt. The patient will not receive general anesthesia or sedation. Statement 2 is false.
Unnecessary pain should be prevented.
8. Answer D. Any air entering the right heart can lead to a pulmonary embolus. All air should be purged from central venous
lines; none should enter the patient.
9. Answer A. A special portacath needle is used to access the portacath device. A syringe is attached and the sample is
obtained. One of the primary reasons for insertion of a portacath device is the need for frequent or long-term blood
sampling. A vacutainer will exert too much suction on the central line resulting in collapse of the line. Only special portacath
needles should be used to access the portacath device.
10. Answer B. The actual access to the subclavian vein is still just under the clavicle, but by tunneling the distal portion of the
catheter several inches under the skin the risk of migratory infection is reduces compared to a catheter that enters the
subclavian vein directly and is not tunneled. The catheter is tunneled to prevent infection.
11. Answer C. A foreign body in a blood vessel increases the risk of infection. Catheters that come outside the body have an
even higher risk of infection. Most infections are caused by skin bacteria. Other infective organisms include yeasts and
fungi. Options 1 and 4 are complications of a CVAD but are not the primary problem. Once placed, these lines do not cause
pain and discomfort.
12. Answer D. A solution containing heparin is used to reduce catheter clotting and maintain patency. The concentration of
heparin used depends on the patients age, comorbidities, and the frequency of catheter access/flushing. Although patients
have few complications, the device is not risk free. Patients may develop infection, catheter clots, vascular obstruction,
pneumothorax, hemothorax, or mechanical problems (catheter breakage). Strict adherence to protocol enhances the
longevity of central access devices. They routinely last weeks to months and sometimes years. The patient will be taught
how to perform dressing changes at home.
13. Answer A. Alkylating agents are highly reactive chemicals that introduce alkyl radicals into biologically active molecules and
thereby prevent their proper functioning, replication, and transcription. Alkylating agents have numerous side effects
including alopecia, nausea, vomiting, and myelosuppression. Nitrogen mustards have a broad spectrum of activity against
chronic lymphocytic leukemia, non-Hodgkins lymphoma, and breast and ovarian cancer, but they are effective
chemotherapeutic agents because of DNA cross-linkage. Alkylating agents are noncell cycle-specific agents.
14. Answer C. Estrogen antagonists are used to treat estrogen hormone-dependent cancer, such as breast carcinoma. A well-
known estrogen antagonist used in breast cancer therapy is tamoxifen (Nolvadex). This drug, in combination with surgery
and other chemotherapeutic drugs reduces breast cancer recurrence by 30 percent. Estrogen antagonists can also be
administered to prevent breast cancer in women who have a strong family history of the disease. Thyroxine is a natural
thyroid hormone. It does not treat thyroid cancer. ACTH is an anterior pituitary hormone, which stimulates the adrenal
glands to release glucocorticoids. It does not treat adrenal cancer. Glucagon is a pancreatic alpha cell hormone, which
stimulates glycogenolysis and gluconeogenesis. It does not treat pancreatic cancer.
15. Answer D. The time required to clear circulating cells before the effect that chemotherapeutic drugs have on precursor cell
maturation in the bone marrow becomes evident. Leukopenia is an abnormally low white blood cell count. Answers A-C
pertain to red blood cells.
16. Answer A. Epoetin alfa (Epogen, Procrit) is a recombinant form of endogenous erythropoietin, a hematopoietic growth
factor normally produced by the kidney that is used to induce red blood cell production in the bone marrow and reduce the
need for blood transfusion. Glucagon is a pancreatic alpha cell hormone, which cause glycogenolysis and gluconeogenesis.
Fenofibrate (Tricor) is an antihyperlipidemic agent that lowers plasma triglycerides. Lamotrigine (Lamictal) is an
anticonvulsant.
17. Answer A. Prostate tissue is stimulated by androgens and suppressed by estrogens. Androgen antagonists will block
testosterone stimulation of prostate carcinoma cells. The types of cancer in options 2-4 are not androgen dependent.
18. Answer A. Chemotherapy often induces vomiting centrally by stimulating the chemoreceptor trigger zone (CTZ) and
peripherally by stimulating visceral afferent nerves in the GI tract. Ondansetron (Zofran) is a serotonin antagonist that bocks
the effects of serotonin and prevents and treats nausea and vomiting. It is especially useful in single-day highly emetogenic
cancer chemotherapy (for example, cisplatin). The agents in options 2-4 are selective serotonin reuptake inhibitors. They
increase the available levels of serotonin.
19. Answer C. With intrathecal administration chemotherapy is injected through the theca of the spinal cord and into the
subarachnoid space entering into the cerebrospinal fluid surrounding the brain and spinal cord. The methods in options A,
B, and D are ineffective because the medication cannot enter the CNS.
20. Answer B. Leucovorin is used to save or rescue normal cells from the damaging effects of chemotherapy allowing them to
survive while the cancer cells die. Therapy to rapidly reduce the number of cancerous cells is the induction phase.
Consolidation therapy seeks to complete or extend the initial remission and often uses a different combination of drugs
than that used for induction. Chemotherapy is often administered in intermittent courses called pulse therapy. Pulse
therapy allows the bone marrow to recover function before another course of chemotherapy is given.
21. Answer B. Prevent uric acid nephropathy, uric acid lithiasis, and gout during cancer therapy since chemotherapy causes the
rapid destruction of cancer cells leading to excessive purine catabolism and uric acid formation. Allopurinol can induce
myelosuppression and pancytopenia. Allopurinol does not have this function.
22. Answer B. Medications administered intravesically are instilled into the bladder. Intraventricular administration involves
the ventricles of the brain. Intravascular administration involves blood vessels. Intrathecal administration involves the fluid
surrounding the brain and spinal cord.
23. Answer C. The overall goal of cancer chemotherapy is to give a dose large enough to be lethal to the cancer cells, but small
enough to be tolerable for normal cells. Unfortunately, some normal cells are affected including the bone marrow.
Myelosuppression limits the bodys ability to prevent and fight infection, produce platelets for clotting, and manufacture
red blood cells for oxygen portage. Even though the effects in options a, b, and d are uncomfortable and distressing to the
patient, they do not have the potential for lethal outcomes that myelosuppression has.
24. Answer A. Vomiting (emesis) is initiated by a nucleus of cells located in the medulla called the vomiting center. This center
coordinates a complex series of events involving pharyngeal, gastrointestinal, and abdominal wall contractions that lead to
expulsion of gastric contents. Catecholamine inhibition does not induce vomiting. Chemotherapy does not induce vomiting
from autonomic instability. Chemotherapy, especially oral agents, may have an irritating effect on the gastric mucosa,
which could result in afferent messages to the solitary tract nucleus, but these pathways do not project to the vomiting
center.
25. Answer A. Myelo comes from the Greek word myelos, which means marrow. Ablation comes from the Latin word ablatio,
which means removal. Thus, myeloablative chemotherapeurtic agents destroy the bone marrow. This procedure destroys
normal bone marrow as well as the cancerous marrow. The patients bone marrow will be replaced with a bone marrow
transplant. Myelocytes are not muscle cells Tumors are solid masses typically located in organs. Surgery may be performed
to reduce tumor burden and require less chemotherapy afterward.
26. Answer C. Nausea and vomiting (N&V) are common side effects of chemotherapy. Some patients are able to trigger these
events prior to actually receiving chemotherapy by anticipating, or expecting, to have these effects. N&V occurring post-
chemotherapeutic administration is not an anticipatory event but rather an effect of the drug. N&V occurring during the
administration of chemotherapy is an effect of the drug.
27. Answer C. Only an unbound drug can be distributed to active receptor sites. Therefore, the more of a drug that is bound to
protein, the less it is available for the desired drug effect. Less drug is available if bound to protein. Distribution to receptor
sites is irrelevant since the drug bound to protein cannot bind with a receptor site. Metabolism would not be increased. The
liver will first have to remove the drug from the protein molecule before metabolism can occur. The protein is then free to
return to circulation and be used again.
28. Answer C. Drugs and drug metabolites with molecular weights higher than 300 may be excreted via the bile, stored in the
gallbladder, delivered to the intestines by the bile duct, and then reabsorbed into the circulation. This process reduces the
elimination of drugs and prolongs their half-life and duration of action in the body. Hepatic clearance is the amount of drug
eliminated by the liver. Total clearance is the sum of all types of clearance including renal, hepatic, and respiratory. First-
pass effect is the amount of drug absorbed from the GI tract and then metabolized by the liver; thus, reducing the amount
of drug making it into circulation.
29. Answer C. Drug B will induce the cytochrome p-450 enzyme system of the liver; thus, increasing the metabolism of Drug A.
Therefore, Drug A will be broken down faster and exert decreased therapeutic effects. Drug A will be metabolized faster,
thus reducing, not increasing its therapeutic effect. Inducing the cytochrome p-450 system will not increase the adverse
effects of Drug B. Drug B induces the cytochrome p-450 system but is not metabolized faster. Thus, the therapeutic effects
of Drug B will not be decreased.
30. Answer A. Epinephrine (adrenaline) rapidly affects both alpha and beta adrenergic receptors eliciting a sympathetic (fight
or flight) response. Muscarinic receptors are cholinergic receptors and are primarily located at parasympathetic junctions.
Cholinergic receptors respond to acetylcholine stimulation. Cholinergic receptors include muscarinic and nicotinic
receptors. Nicotinic receptors are cholinergic receptors activated by nicotine and found in autonomic ganglia and somatic
neuromuscular junctions.











Fundamentals of Nursing Exam 12: Pharmacology (30 Items)
Introduction
Take the last part of our Nursing Pharmacology exam series with this 30-item exam. This exam focuses more on providing
interventions when adverse effects occurs.
Topics
Guidelines
Questions
InText Mode: All questions and answers are given for reading and answering at your own pace. You can also copy this exam and
make a print out.
1. Walter, a teenage patient is admitted to the hospital because of acetaminophen (Tylenol) overdose. Overdoses of
acetaminophen can precipitate life-threatening abnormalities in which of the following organs?
a. Lungs
b. Liver
c. Kidney
d. Adrenal Glands
2. A contraindication for topical corticosteroid usage in a male patient with atopic dermatitis (eczema) is:
a. Parasite infection.
b. Viral infection.
c. Bacterial infection.
d. Spirochete infection.
3. In infants and children, the side effects of first generation over-the-counter (OTC) antihistamines, such as diphenhydramine
(Benadryl) and hydroxyzine (Atarax) include:
a. Reyes syndrome.
b. Cholinergic effects.
c. Paradoxical CNS stimulation.
d. Nausea and diarrhea.
4. Reyes syndrome, a potentially fatal illness associated with liver failure and encephalopathy is associated with the
administration of which over-the-counter (OTC) medication?
a. acetaminophen (Tylenol)
b. ibuprofen (Motrin)
c. aspirin
d. brompheniramine/pseudoephedrine (Dimetapp)
5. The nurse is aware that the patients who are allergic to intravenous contrast media are usually also allergic to which of the
following products?
a. Eggs
b. Shellfish
c. Soy
d. acidic fruits
6. A 13-month-old child recently arrived in the United States from a foreign country with his parents and needs childhood
immunizations. His mother reports that he is allergic to eggs. Upon further questioning, you determine that the allergy to eggs is
anaphylaxis. Which of the following vaccines should he not receive?
a. Hepatitis B
b. inactivated polio
c. diphtheria, acellular pertussis, tetanus (DTaP)
d. mumps, measles, rubella (MMR)
7. The cell and Coombs classification system categorizes allergic reactions and is useful in describing and classifying patient
reactions to drugs. Type I reactions are immediate hypersensitivity reactions and are mediated by:
a. immunoglobulin E (IgE).
b. immunoglobulin G (IgG).
c. immunoglobulin A (IgA).
d. immunoglobulin M (IgM).
8. Drugs can cause adverse events in a patient. Bone marrow toxicity is one of the most frequent types of drug-induced toxicity.
The most serious form of bone marrow toxicity is:
a. aplastic anemia.
b. thrombocytosis.
c. leukocytosis.
d. granulocytosis.
9. Serious adverse effects of oral contraceptives include:
a. Increase in skin oil followed by acne.
b. Headache and dizziness.
c. Early or mid-cycle bleeding.
d. Thromboembolic complications.
10. The most serious adverse effect of Alprostadil (Prostin VR pediatric injection) administration in neonates is:
a. Apnea.
b. Bleeding tendencies.
c. Hypotension.
d. Pyrexia.
11. Mandy, a patient calls the clinic today because he is taking atorvastatin (Lipitor) to treat his high cholesterol and is having pain
in both of his legs. You instruct him to:
a. Stop taking the drug and make an appointment to be seen next week.
b. Continue taking the drug and make an appointment to be seen next week.
c. Stop taking the drug and come to the clinic to be seen today.
d. Walk for at least 30 minutes and call if symptoms continue.
12. Which of the following adverse effects is associated with levothyroxine (Synthroid) therapy?
a. Tachycardia
b. Bradycardia
c. Hypotension
d. Constipation
13. Which of the following adverse effects is specific to the biguanide diabetic drug metformin (Glucophage) therapy?
a. Hypoglycemia
b. GI distress
c. Lactic acidosis
d. Somnolence
14. The most serious adverse effect of tricyclic antidepressant (TCA) overdose is:
a. Seizures.
b. Hyperpyrexia.
c. Metabolic acidosis.
d. Cardiac arrhythmias.
15. The nurse is aware that the following solutions is routinely used to flush an IV device before and after the administration of
blood to a patient is:
a. 0.9 percent sodium chloride
b. 5 percent dextrose in water solution
c. Sterile water
d. Heparin sodium
16. Cris asks the nurse whether all donor blood products are cross-matched with the recipient to prevent a transfusion reaction.
Which of the following always require cross-matching?
a. packed red blood cells
b. platelets
c. plasma
d. granulocytes
17. A month after receiving a blood transfusion an immunocompromised male patient develops fever, liver abnormalities, a rash,
and diarrhea. The nurse would suspect this patient has:
a. Nothing related to the blood transfusion.
b. Graft-versus-host disease (GVHD).
c. Myelosuppression.
d. An allergic response to a recent medication.
18. Jonas comes into the local blood donation center. He says he is here to donate platelets only today. The nurse knows this
process is called:
a. Directed donation.
b. Autologous donation.
c. Allogeneic donation.
d. Apheresis.
19. Nurse Bryan knows that the age group that uses the most units of blood and blood products is:
a. Premature infants.
b. Children ages 1-20 years.
c. Adults ages 21-64 years.
d. The elderly above age 65 years.
20. A child is admitted with a serious infection. After two days of antibiotics, he is severely neutropenic. The physician orders
granulocyte transfusions for the next four days. The mother asks the nurse why? The nurse responds:
a. This is the only treatment left to offer the child.
b. This therapy is fast and reliable in treating infections in children.
c. The physician will have to explain his rationale to you.
d. Granulocyte transfusions replenish the low white blood cells until the body can produce its own.
21. A neighbor tells nurse Maureen he has to have surgery and is reluctant to have any blood product transfusions because of a
fear of contracting an infection. He asks the nurse what are his options. The nurse teaches the person that the safest blood
product is:
a. An allogeneic product.
b. A directed donation product.
c. An autologous product.
d. A cross-matched product.
22. A severely immunocompromised female patient requires a blood transfusion. To prevent GVHD, the physician will order:
a. Diphenhydramine hydrochloride (Benadryl).
b. The transfusion to be administered slowly over several hours.
c. Irradiation of the donor blood.
d. Acetaminophen (Tylenol).
23. Louie who is to receive a blood transfusion asks the nurse what is the most common type of infection he could receive from
the transfusion. The nurse teaches him that approximately 1 in 250,000 patients contract:
a. Human immunodeficiency disease (HIV).
b. Hepatitis C infection.
c. Hepatitis B infection.
d. West Nile viral disease.
24. A male patient with blood type AB, Rh factor positive needs a blood transfusion. The Transfusion Service (blood bank) sends
type O, Rh factor negative blood to the unit for the nurse to infuse into this patient. The nurse knows that:
a. This donor blood is incompatible with the patients blood.
b. Premedicating the patient with diphenhydramine hydrochloride (Benadryl) and acetaminophen (Tylenol) will prevent any
transfusion reactions or side effects.
c. This is a compatible match.
d. The patient is at minimal risk receiving this product since it is the first time he has been transfused with type O, Rh negative blood.
25. Dr. Rodriguez orders 250 milliliters of packed red blood cells (RBC) for a patient. This therapy is administered for treatment of:
a. Thrombocytopenia.
b. Anemia.
c. Leukopenia.
d. Hypoalbuminemia.
26. A female patient needs a whole blood transfusion. In order for transfusion services (the blood bank) to prepare the correct
product a sample of the patients blood must be obtained for:
a. A complete blood count and differential.
b. A blood type and crossmatch.
c. A blood culture and sensitivity.
d. A blood type and antibody screen.
27. A male patient needs to receive a unit of whole blood. What type of intravenous (IV) device should the nurse consider
starting?
a. A small catheter to decrease patient discomfort
b. The type of IV device the patient has had in the past, which worked well
c. A large bore catheter
d. The type of device the physician prefers
28. Dr. Smith orders a gram of human salt poor albumin product for a patient. The product is available in a 50 milliliter vial with a
concentration of 25 percent. What dosage will the nurse administer?
a. The nurse should use the entire 50 milliliter vial.
b. The nurse should determine the volume to administer from the physician.
c. This concentration of product should not be used.
d. The nurse will administer 4 milliliters.
29. Central venous access devices (CVADs) are frequently utilized to administer chemotherapy. What is a distinct advantage of
using the CVAD for chemotherapeutic agent administration?
a. CVADs are less expensive than a peripheral IV.
b. Once a week administration is possible.
c. Caustic agents in small veins can be avoided.
d. The patient or his family can administer the drug at home.
30. A female patients central venous access device (CVAD) becomes infected. Why would the physician order antibiotics to be
given through the line rather than through a peripheral IV line?
a. To prevent infiltration of the peripheral line
b. To reduce the pain and discomfort associated with antibiotic administration in a small vein
c. To lessen the chance of an allergic reaction to the antibiotic
d. To attempt to sterilize the catheter and prevent having to remove it
Answers & Rationale
Here are the answers and rationale for this exam. Counter check your answers to those below and tell us your scores. If you have
any disputes or need more clarification to a certain question, please direct them to the comments section.
1. Answer B. Acetaminophen is extensively metabolized by pathways in the liver. Toxic doses of acetaminophen deplete hepatic
glutathione, resulting in accumulation of the intermediate agent, quinine, which leads to hepatic necrosis. Prolonged use of
acetaminophen may result in an increased risk of renal dysfunction, but a single overdose does not precipitate life-threatening
problems in the respiratory system, renal system, or adrenal glands.
2. Answer B. Topical agents produce a localized, rather than systemic effect. When treating atopic dermatitis with a steroidal
preparation, the site is vulnerable to invasion by organisms. Viruses, such as herpes simplex or varicella-zoster, present a risk of
disseminated infection. Educate the patient using topical corticosteroids to avoid crowds or people known to have infections
and to report even minor signs of an infection. Topical corticosteroid usage results in little danger of concurrent infection with
these agents.
3. Answer C. Typically, first generation OTC antihistamines have a sedating effect because of passage into the CNS. However, in
some individuals, especially infants and children, paradoxical CNS stimulation occurs and is manifested by excitement,
euphoria, restlessness, and confusion. For this reason, use of first generation OTC antihistamines has declined, and second
generation product usage has increased. Reyes syndrome is a systemic response to a virus. First generation OTC
antihistamines do not exhibit a cholinergic effect. Nausea and diarrhea are uncommon when first generation OTC
antihistamines are taken.
4. Answer C. Virus-infected children who are given aspirin to manage pain, fever, and inflammation are at an increased risk of
developing Reyes syndrome. Use of acetaminophen has not been associated with Reyes syndrome and can be safely given to
patients with fever due to viral illnesses. Ibuprofen adverse effects include GI irritation and bleeding, and in toxic doses, both
renal and hepatic failure are reported. However, ibuprofen has not been associated with the onset of Reyes disease.
Brompheniramine/pseudoephedrine contains a first generation OTC antihistamine and a decongestant. Neither agent has been
associated with the development of Reyes syndrome.
5. Answer B. Some types of contrast media contain iodine as an ingredient. Shellfish also contain significant amounts of iodine.
Therefore, a patient who is allergic to iodine will exhibit an allergic response to both iodine containing contrast media and
shellfish. These products do not contain iodine.
6. Answer D. The measles portion of the MMR vaccine is grown in chick embryo cells. The current MMR vaccine does not contain
a significant amount of egg proteins, and even children with dramatic egg allergies are extremely unlikely to have an
anaphylactic reaction. However, patients that do respond to egg contact with anaphylaxis should be in a medically controlled
setting where full resuscitation efforts can be administered if anaphylaxis results. The vaccines in options a,b and c do not
contain egg protein.
7. Answer A. IgE, the least common serum immunoglobulin (Ig) binds very tightly to receptors on basophils and mast cells and is
involved in allergic reactions. Binding of the allergen to the IgE on the cells results in the release of various pharmacological
mediators that result in allergic symptoms. IgG is the major Ig (75 percent of serum Ig is IgG). Most versatile Ig because it is
capable of carrying out all of the functions of Ig molecules. IgG is the only class of Ig that crosses the placenta. It is an opsonin,
a substance that enhances phagocytosis. IgA, the second most common serum Ig is found in secretions (tears, saliva,
colostrum, and mucus). It is important in local (mucosal) immunity. IgM, the third most common serum Ig, is the first Ig to be
made by the fetus and the first Ig to be made by a virgin B cell when it is stimulated by antigen. IgM antibodies are very
efficient in leading to the lysis of microorganisms.
8. Answer A. Aplastic anemia is the result of a hypersensitivity reaction and is often irreversible. It leads to pancytopenia, a
severe decrease in all cell types: red blood cells, white blood cells, and platelets. A reduced number of red blood cells causes
hemoglobin to drop. A reduced number of white blood cells make the patient susceptible to infection. And, a reduced number
of platelets cause the blood not to clot as easily. Treatment for mild cases is supportive. Transfusions may be necessary. Severe
cases require a bone marrow transplant. Option 2 is an elevated platelet count. Option 3 is an elevated white count. Option 4
is an elevated granulocyte count. A granulocyte is a type of white blood cell.
9. Answer D. Oral contraceptives have been associated with an increased risk of stroke, myocardial infarction, and deep vein
thrombosis. These risks are increased in women who smoke. Increased skin oil and acne are effects of progestin excess.
Headache and dizziness are effects of estrogen excess. Early or mid-cycle bleeding are effects of estrogen deficiency.
10. Answer A. All items are adverse reactions of the drug. However, apnea appearing during the first hour of drug infusion occurs
in 10-12 percent of neonates with congenital heart defects. Clinicians deciding to utilize alprostadil must be prepared to
intubate and mechanically ventilate the infant. Careful monitoring for apnea or respiratory depression is mandatory. In some
institutions, elective intubation occurs prior to initiation of the medication.
11. Answer C. Muscle aches, soreness, and weakness may be early signs of myopathy such as rhabdomyolysis associated with the
HMG-CoA reductase class of antilipemic agents. This patient will need an immediate evaluation to rule out myopathy.
Additional doses may exacerbate the problem. Exercise will not reverse myopathy and delays diagnosis.
12. Answer A. Levothyroxine, especially in higher doses, can induce hyperthyroid-like symptoms including tachycardia. An agent
that increases the basal metabolic rate would not be expected to induce a slow heart rate. Hypotension would be a side effect
of bradycardia. Constipation is a symptom of hypothyroid disease.
13. Answer C. Lactic acidosis is the most dangerous adverse effect of metformin administration with death resulting in
approximately 50 percent of individuals who develop lactic acidosis while on this drug. Metformin does not induce insulin
production; thus, administration does not result in hypoglycemic events. Some nausea, vomiting, and diarrhea may develop
but is usually not severe. NVD is not specific for metformin. Metformin does not induce sleepiness.
14. Answer D. Excessive ingestion of TCAs result in life-threatening wide QRS complex tachycardia. TCA overdose can induce
seizures, but they are typically not life-threatening. TCAs do not cause an elevation in body temperature. TCAs do not cause
metabolic acidosis.
15. Answer A. 0.9 percent sodium chloride is normal saline. This solution has the same osmolarity as blood. Its use prevents red
cell lysis. The solutions given in options 2 and 3 are hypotonic solutions and can cause red cell lysis. The solution in option 4
may anticoagulate the patient and result in bleeding.
16. Answer A. Red blood cells contain antigens and antibodies that must be matched between donor and recipient. The blood
products in options 2-4 do not contain red cells. Thus, they require no cross-match.
17. Answer B. GVHD occurs when white blood cells in donor blood attack the tissues of an immunocompromised recipient. This
process can occur within a month of the transfusion. Options 1 and 4 may be a thought, but the nurse must remember that
immunocompromised transfusion recipients are at risk for GVHD.
18. Answer D. The process of apheresis involves removal of whole blood from a donor. Within an instrument that is essentially
designed as a centrifuge, the components of whole blood are separated. One of the separated portions is then withdrawn, and
the remaining components are retransfused into the donor. Directed donation is collected from a blood donor other than the
recipient, but the donor is known to the recipient and is usually a family member or friend. Autologous donation is the
collection and reinfusion of the patients own blood. Allogeneic donation is collected from a blood donor other than the
recipient.
19. Answer D. People older than 65 years use 43 percent of donated blood. This number is expected to increase as the population
ages.
20. Answer D. Granulocyte (neutrophil) replacement therapy is given until the patients blood values are normal and he is able to
fight the infection himself. Options 1 and 3 are not therapeutic responses. The treatment in option 2 takes days and is not
always able to prevent morbidity and mortality.
21. Answer C. This process is the collection and reinfusion of the patients own blood. It is recommended by the American Medical
Associations Council on Scientific Affairs as the safest product since it eliminates recipient incompatibility and infection. The
product in option 1 is collected from a blood donor other than the recipient. The process in option 2 is also collected from a
blood donor other than the recipient, but the donor is known to the recipient and is usually a family member or friend. Cross-
matching significantly enhances compatibility. It does not detect infection.
22. Answer C. This process eliminates white blood cell functioning, thus, preventing GVHD. Diphenhydramine HCl is an
antihistamine. Its use prior to a blood transfusion decreases the likelihood of a transfusion reaction. Option 2 will not prevent
GVHD. Use of acetaminophen prevents and treats the common side effects of blood administration caused by the presence of
white blood cells in the transfusion product: fever, headache, and chills.
23. Answer C. Hepatitis B is the most common infection spread via blood transfusion. Donors are screened by a questionnaire that
includes symptoms. The donated blood is also tested for infection. The risk of infection with the agents in options 2 and 3 has
decreased to approximately 1 in 2 million secondary to donor questioning and donor blood testing. The incidence of West Nile
viral transmission is unknown, but donor infection is still relatively rare.
24. Answer C. Type O, Rh negative blood has none of the major antigens and is safely administered to patients of all blood types. It
is also known as the universal donor. Premedicating with these agents will not prevent a major transfusion reaction if the
blood type and Rh factors of the donor blood are incompatible with the recipients blood.
25. Answer B. A red blood cell transfusion is used to correct anemia in patients in which the low red blood cell count must be
rapidly corrected. RBC transfusion will not correct a low platelet count. RBC transfusion will not correct a low white blood cell
count. Packed RBCs contain very little plasma and, thus, only a small amount of albumin. This amount will not correct low
albumin levels.
26. Answer B. This is needed to utilize the correct type of donor blood and to match the donor product with the patient.
Incompatible matches would result in severe adverse events and possible death. The tests in options 1 and 3 are unnecessary.
The test in option 4 is utilized to determine the patients blood type and presence of antibodies to blood antigens. It does not
determine donor blood compatibility with the patient.
27. Answer C. Large bore catheters prevent damage to blood components and are less likely to develop clotting problems than a
small bore catheter. The nurse should determine the correct device without asking the patient what type has been used before
or asking the physician which type he prefers and start the IV.
28. Answer D. A 25 percent solution contains one quarter of a gram per milliliter. Thus, the nurse will administer 4 milliliters to
provide a complete gram of albumin. The volume in option 1 would provide 12.5 grams of albumin. The nurse should
determine the volume. It is unnecessary to seek the answer from the physician. A 25 percent solution is an acceptable product
and can safely be used.
29. Answer C. Many chemotherapeutic drugs are vesicants (highly active corrosive materials that can produce tissue damage even
in low concentrations). Extravasations of a vesicant can result in significant tissue necrosis. Administration into a large vein is
optimal. CVADs are more expensive than a peripheral IV. Dosing depends on the drug. IV chemotherapeutic agents are not
administered at home. They are given in an outpatient or clinic setting if not given during hospitalization.
30. Answer D. Microorganisms that infect CVADs are often coagulase-negative staphylococci, which can be eliminated by
antibiotic administration through the catheter. If unsuccessful in eliminating the microorganism, the CVAD must be removed.
CVAD use lessens the need for peripheral IV lines and, thus, the risk of infiltration. In this case however, the antibiotics are
given to eradicate microorganisms from the CVAD. CVAD use has this effect, but in this case, the antibiotics are given through
the CVAD to eliminate the infective agent. The third option would not occur.





Nursing Practice Test about Pharmacology (NLE 1-20)
A client with bacterial pneumonia is to be started on intravenous antibiotics. Which of the following diagnostic tests must be
completed before antibiotic therapy begins?

a.) urinalysis
b.) sputum culture
c.) chest radiograph
d.) red blood cell count


Which of the following anti-tuberculosis drugs can cause damage to the eight cranial nerve?

a. streptomycin
b. isoniazid (INH)
c. para-aminosalicylic acids (PAS)
d. ethambutol hydrochloride (myambutol)

What is the rationale that supports multi-drug treatment for clients with tuberculosis?

a) multiple drug potentiate the drug's actions
b) multiple drugs reduce undesirable drug side effects
c) multiple drugs allow reduced drug dosages to be given
d) multiple drugs reduce development of resistant strains of bacteria

The nurse should caution sexually active female clients taking that the drug has which of the following effects?

a) increases the risk of vaginal infection
b) has mutagenic effects on ova
c) decreases the effectiveness of oral contraceptives
d) inhibits ovulation

Isoniazid (INH) and Rifamficin (Rifadin) have been prescribed for a client with tuberculosis. A nurse reviews the medical record of the
client. Which of the following, if noted in the client's history, would require physician notification?

a) heart disease
b) allergy to penicillin
c) hepatitis B
d) rheumatic fever

Digoxin is administered intravenously to a client with heart failure, primarily because the drug acts to:

a) dilate coronary artery
b) increase myocardial contractility
c) decrease cardiac dysrhytmias
d) decrease electrical conductivity in the heart

The nurse teaches the client with angina about the common expected side effects of nitroglycerin, including:

a) headache
b) high blood pressure
c) shortness of breath
d) stomach cramps

Sublingual nitroglycerin tablets begin to work within 1 to 2 minutes. How should the nurse instruct the client to use the drug when
chest pain occurs?

a) take one tablet every 2 to 5 minutes until chest pain stops
b) take one tablet and rest for 10 minutes. Call the physician if pain persists after 10 minutes
c) take one tablet, then an additional tablet every 5 minutes for a total of three tablets. Call the physician if pain persists after three
tablets
d) take one tablet. If the pain persists after 5 minutes, take two tablets. If pain still persists 5 minutes later, call the physician

Because smoking cessation is a critical strategy for the client diagnosed with Buerger's disease, the nurse anticipates that the client
will go home with a prescription for which of the following medications?

a) nicotine ( nicontrol)
b) nitroglycerin
c) furosimide ( lasix)
d) ibuprofen

A client with leukemia is receiving busulfan (myleran). Allopurinol (zyloprim) is prescribed for the client. The purpose of the
allopurinol is to:

a) prevent gouty arthritis
b) prevent hyperuricemia
c) prevent diarrhea
d) prevent stomatitis

11. Epidural analgesia is administered to woman for pain relief following a CS birth. The nurse assigned
to care for the woman ensures that which medication is readily available if respiratory depression
occurs?

a) Bethamethasone (celestone)
b) Morphine sulfate
c) Meperidine hydrochloride (demelor)
d) Nalaxone (narcan)

12. Methylerogonovine (methergine) is prescribed for a postpartum woman to treat postpartum hemorrhage.
Before administration, the priority nursing assessment is to check the:

a) Amount if lochia
b) BP
c) Deep tendon reflexes
d) uterine tone

13. A nurse is caring for the patient who is receiving pitocin (oxytocin) for the induction of labor. The nurse
discontinues the pitocin infusion if which one of the following is noted on the assessment of the client?

a) drowsiness
b) fatigue
c) early decelerations of fetal heart rate
d) uterine hyperstimulation

14. A pregnant client is magnesium sulfate for the management of preeclamsia. A nurse determines that the client experiencing
toxicity from the medication if which of the following is noted
on assessment?

a) presence of deep tendon reflexes
b) serum magnesium level of 6 mEq/L
c) proteinuria +3
d) respirations of 10 per minute

15. A woman with pre-eclampsia is receiving magnesium sulfate. The nurse assigned to care for the client determines that the
magnesium sulfate therapy is effective if?

a) ankle clonus is noted
b) BP decreases
c) seizure do not occur
d) scotomas are present

16. Methylergonovine (metthergine) is prescribed for a client with postpartum hemorrhage. Prior to administering the medication, a
nurse contacts the health care provider who prescribed the medication if which of the following conditions is documented?

a) perpheral vascular disease
b) hypothyroidism
c) hypotension
d) diabetes mellitus

17. Vitamin K ( Aquamephyton) is prescribed for a neonate. Nurse Jarell prepares the medication and selects which muscle site to
administer the medication?

a) deltoid
b) dorso gluteal
c) vastus lateralis
d) biceps

18. Ms. Nanette, a clinical instructor, asks a nursing student to describe the procedure of administering erythromycin ointment to
the eyes of a neonate. The instructor determines that student needs further teaching if she states;

a) " I will cleanse the neonate's eyes before instilling ointment"
b) " I will flush the eyes after instilling the ointment"
c) I will instill the eye ointment into each of the neonate's conjunctival sacs within 1 hour after birth
d) "administration of the eye ointment may be delayed until an hour or so after birth so that eye contact and parent-infant bonding
can occur

19. Dr. Garcia has prescribed Phenobartal sodium (luminal sodium), 25 mg PO bid, for a child with febrile seizures. The medication
label reads: Phenobartal sodium, 20 mg per 5ml. Nurse Kris has determined that the dosage prescribed is a safe for a child. The
nurse prepares to administer how many milliliters per dose to the child?

a) 4.7 ml
b) 4.5 ml
c) 6.25 ml
d) 7 ml

20. Cloxacillin (tegopen), 100 mg PO every 8 hours, is prescribed for a child with an elevated remperature who is suspected of having
a respiratory tract infection. The child weighs 17 lbs. the safe pediatric dosage is 50 mg/kg/day. The nurse determines that:

a) the dose is too high
b) the dose is too low
c) the dose is safe
d) there is not enough information to determine safe dose


ANSWERS

1.) B ..... 2.) A ..... 3.) D ..... 4.) c .....5) c

6) b ..... 7) a ..... 8) c ..... 9) a ..... 10) b

11) A ... 12) B ... 13) D ... 14) D ... 15) C

16) A ... 17) C ... 18) B ... 19) C ... 20) B






Nursing Practice Test about Pharmacology (NLE 21-40)
21. Diphenhydramine ydrochloride (Benadryl), 25mg PO every 6 hours, is prescribed for a child with an allergic reaction. The child
weighs 25 kg. The safe pediatric dosage is 5 mg/kg/day. The nurse determines that:
a) the dose is to high
b) the ose is too low
c) the dose is safe
d) there is not enough information to determine safe dose

22.Morphine sulfate, 2.5mg, IV piggyback, in 10ml of normal saline, is prescribed for a child post-operatively. The medication label
reads: 1/15gr/ml. The nurse prepares to add how many ml of morphine sulfate to the 10ml NSS?

a) 0.62ml
b) 0.82ml
c) 0.72ml
d) 0.92ml

23. Dr. Lauzon's order reads: Ampicillin (omnipen), 125 mg IV per 6 hour. The medication label reads: 1g and reconstitute with 7.4ml
of bacteriostatic water. A nurse prepares to draw up how many ml to administer one dose?

a) 0.54ml
b) 0.92ml
c) 1.1ml
d) 7.4ml

24. Salicylic acid is prescribed for Mang Macario, client diagnosed with psoriasis. A nurse monitors him, knowing that which of the
following would indicate the presence of systemic toxicity from this medication?

a) decrease RR
b) diarrhea
c) constipation
d) tinnitus

25. Nurse Arlene is monitoring the lab reuslts for a client receiving an antineoplastic medication by the IV route. The nurse prepares
to initiate bleeding precautions if which lab results is noted?

a) WBC count of 5000 cells/ml
b) platelet of 50,000 cells/ml
c) clotting time of 10 minutes
d) ammonia level of 20mg/dL

26. A nurse is providing medication instructions to a client with breast cancer who is taking cyclophosphamide (cytoxan), the nurse
tells the client to:

a) take the medication with food
b) increase fluid intake to 2000 to 3000 ml per day
c) decrease sodium intake while taking the medication
d) increase potassium intake while taking the medication

27. A clinic nurse prepares a teaching plan for a client receiving an antineoplastic medication. When implementing the plan, the
nurse tells the client to:

a) take aspirin (ASA) as needed for headache
b) drink beverages containing alcohol in moderate amounts
c) consult with the physician before receiving immunizations
d) be sure to receive flu and pneumonia vaccines

28. A client with ovarian cancer is being rteated with vincristine (Oncovin). A nurse monitors the client, knowing that which of the
following is a side effect specific to this medication?

a) diarrhea
b) numbness and tingling in the fingers and toes
c) chest pain
d) hair loss

29. A client with metastatic breast cancer is receiving Tamoxifen (Nolvadex). A nurse specifically monitors which of the following
laboratory values while the client is taking this medication?

a) potassium level
b) glucose level
c) calcium level
d) prothrombin time

30. Megestrol acetate (Megace), an antineoplastic medication, is prescribed for a client with metastatic endometrial carcinoma. A
nurse reviews the client's history and contacts the physician if which of the following is documented in the client's history?

a) asthma
b) myocardial infarction
c) thrombophlebitis
d) gout

31. Marissa has a breast cancer and is admitted to a hospital for treatment with IV vincristine (Oncovin). She tells the nurse that she
has been told by friends that she is going to loose all her hair. The most appropriate response in which of the following?

a) you will not lose your hair
b) your friends are connect
c) hair loss may occur, but it will grow back just it is now
d) hair loss may occur, and it will grow back, but it may have a different color and texture

32. A client with acute leukemia is being treated with busulfan (Myeleran). Which of the following laboratory values would a nurse
specifically monitor during treatment with this medication?

a) blood glucose
b) uric acid level
c) potassium level
d) cltting time

33. Vasopressin ( Pitressin) is oredered for a client with diabetes insipidus. A nurse is particularly cautious in monitoring the client
receiving this medication if the client has which o the following preexisting condition?

a) depreression
b) endometriosis
c) coronary artery disease
d) phechromocytoma

34. Nurse Christian provides teaching to a client who is taking levothyroxine (Synthroid). The nurse tells the client to take the
medication:

a) with food
b) on an empty stomach
c) at bedtime with snack
d) at lunchtime

35. A nurse provides medication instructions to a client who is taking Synthroid. The nurse instructs the client to notify the physician
if which of the following occurs?

a) cold intolerance
b) tremors
c) excessively dry skin
d) fatigue

36. A nurse performs an admission assessment on Marina who visits the clinic for the first time. The client tells that she is taking
propylthiouracil (PTU) daily. The nurse continues to collect data from the client, suspecting that the client has history of:

a) cushing's syndrome
b) addison's disease
c) myxedema
d) grave's disease

37. A client is receiving somatropin (Humatrope). the nurse monitors which significant laboratory study during therapy with this
medication?

a) amylase
b) lipase
c) BUN
d) TSH

38. A client is scheduled for a subtotal thyroidectomy. Iodine solution (lugol solution) is prescribed. A nurse prepares to administer
the medication, knowing the therapeutic effect of this medication to:

a) increase thyroid hormone production
b) supresses thyroid hormone production
c) replace thyroid hormone
d) prevent the oxidation of iodine

39. Calcium carbonate is prescribed for a client with hypocalcemia. Nurse Billy should instruct the client to take the medication:

a) with meals
b) one hour after meals
c) just aftr meals
d) evry 4 hours

40. A daily dose of prednisone (Deltasone) is prescribed for a client. A nurse provides instruction to the client regarding medication.
The nurse is correct to say that the best time to take the medication is:

a) at bedtime
b) at noon
c) early morning
d) anytime, but must be on the same time, each day


ANSWERS


21) B ..... 22) B ..... 23) B ..... 24) D ..... 25) B

26) A ..... 27) C ..... 28) B ..... 29) B ..... 30) C

31) D ..... 32) C ..... 33) C ..... 34) A ..... 35) B

36) D ..... 37) C ..... 38) C ..... 39) C ..... 40) D








Fundamentals of Nursing Exam 13: NLE Pre-Boards (100 Items)
2014/01/26

Introduction
Pre-board examinations are mock exams that simulate the licensure examinations in the Philippines. This 100-item exam will tackle
the concepts of Fundamentals of Nursing.
Topics
Guidelines
Questions
InText Mode: All questions and answers are given for reading and answering at your own pace. You can also copy this exam and
make a print out.
Situation 1 Mr. Ibarra is assigned to the triage area and while on duty, he assesses the condition of Mrs. Simon who came in with
asthma. She has difficulty breathing and her respiratory rate is 40 per minute. Mr. Ibarra is asked to inject the client epinephrine
0.3mg subcutaneously
1. The indication for epinephrine injection for Mrs Simon is to:
a. Reduce anaphylaxis
b. Relieve hypersensitivity to allergen
c. Relieve respiratory distress due to bronchospasm
d. Restore clients cardiac rhythm
2. When preparing the epinephrine injection from an ampule, the nurse initially:
a. Taps the ampule at the top to allow fluid to flow to the base of the ampule
b. Checks expiration date of the medication ampule
c. Removes needle cap of syringe and pulls plunger to expel air
d. Breaks the neck of the ampule with a gauze wrapped around it
3. Mrs. Simon is obese. When administering a subcutaneous injection to an obese patient, it is best for the nurse to:
a Inject needle at a 15 degree angle over the stretched skin of the client
b. Pinch skin at the Injection site and use airlock technique
c. Pull skin of patient down to administer the drug in a Z track
d. Spread skin or pinch at the injection site and inject needle at a 45-90 degree angle
4. When preparing for a subcutaneous injection, the proper size of syringe and needle would be:
a. Syringe 3-5ml and needle gauge 21 to 23
b. Tuberculin syringe 1 ml with needle gauge 26 or 27
c. Syringe 2ml and needle gauge 22
d. Syringe 1-3 ml and needle gauge 25 to 27
5. The rationale for giving medications through the subcutaneous route is:
a. There are many alternative sites for subcutaneous injection
b. Absorption time of the medicine is slower
c. There are less pain receptors in this area
d. The medication can be injected while the client is in any position
Situation 2 The use of massage and meditation to help decrease stress and pain have been strongly recommended based on
documented testimonials.
6. Martha wants to do a study on, this topic. Effects of massage and meditation on stress and pain. The type of research that
best suits this topic is:
a. applied research
b. qualitative research
c. basic research
d. quantitative research
7. The type of research design that does not manipulate independent variable is:
a. experimental design
b. quasi-experimental design
c. non-experimental design
d. quantitative design
8. This research topic has the potential to contribute to nursing because it seeks to:
a. include new modalities of care
b. resolve a clinical problem
c. clarify an ambiguous modality of care
d. enhance client care
9. Martha does review of related literature for the purpose of:
a. determine statistical treatment of data research
b. gathering data about what is already known or unknown
c. to identify if problem can be replicated
d. answering the research question
10. Clients rights should be protected when doing research using human subjects. Martha identifies these rights as follows
EXCEPT:
a. right of self-determination
b. right to compensation
c. right of privacy
d. right not to be harmed
Situation 3 Richard has a nursing diagnosis of ineffective airway clearance related to excessive secretions and is at risk for infection
because of retained secretions. Part of Nurse Marios nursing care plan is to loosen and remove excessive secretions in the airway,
11. Mario listens to Richards bilateral sounds and finds that congestion is in the upper lobes of the lungs. The appropriate
position to drain the anterior and posterior apical segments of the lungs when Mario does percussion would be:
a. Client lying on his back then flat on his abdomen on Trendelenburg position
b. Client seated upright in bed or on a chair then leaning forward in sitting position then flat on his back and on his abdomen
c. Client lying flat on his back and then flat on his abdomen
d. Client lying on his right then left side on Trendelenburg position
12. When documenting outcome of Richards treatment Mario should include the following in his recording EXCEPT:
a. Color, amount and consistent of sputum
b. Character of breath sounds and respiratory/rate before and after procedure
c. Amount of fluid intake of client before and after the procedure
d. Significant changes in vital signs
13. When assessing Richard for chest percussion or chest vibration and postural drainage Mario would focus on the following
EXCEPT:
a. Amount of food and fluid taken during the last meal before treatment
b. Respiratory rate, breath sounds and location of congestion
c. Teaching the clients relatives to perform the procedure
d. Doctors order regarding position restriction and clients tolerance for lying flat
14. Mario prepares Richard for postural drainage and percussion. Which of the following is a special consideration when doing the
procedure?
a. Respiratory rate of 16 to 20 per minute
b. Client can tolerate sitting and lying position
c. Client has no signs of infection
d. Time of fast food and fluid intake of the client
15. The purpose of chest percussion and vibration is to loosen secretions in the lungs. The difference between the procedure is:
a. Percussion uses only one hand white vibration uses both hands
b. Percussion delivers cushioned blows to the chest with cupped palms while gently shakes secretion loose on the exhalation cycle
c. In both percussion and vibration the hands are on top of each other and hand action is in tune with clients breath rhythm
d. Percussion slaps the chest to loosen secretions while vibration shakes the secretions along with the inhalation of air
Situation 4 A 61 year old man, Mr. Regalado, is admitted to the private ward for observation after complaints of severe chest pain.
You are assigned to take care of the client.
16. When doing an initial assessment, the best way for you to identify the clients priority problem is to:
a. Interview the client for chief complaints and other symptoms
b. Talk to the relatives to gather data about history of illness
c. Do auscultation to check for chest congestion
d. Do a physical examination white asking the client relevant questions
17. Upon establishing Mr. Regalados nursing needs, the next nursing approach would be to:
a. introduce the client to the ward staff to put the client and family at ease
b. Give client and relatives a brief tour of the physical set up the unit
c. Take his vital signs for a baseline assessment
d. Establish priority needs and implement appropriate interventions
18. Mr. Regalado says he has trouble going to sleep. In order to plan your nursing intervention you will.
a. Observe his sleeping patterns in the next few days
b. Ask him what he means by this statement
c. Check his physical environment to decrease noise level
d. Take his blood pressure before sleeping and upon waking up
19. Mr. Regalados lower extremities are swollen and shiny. He has pitting pedal edema. When taking care of Mr. Regalado,
which of the following intervention would be the most appropriate immediate nursing approach.
a. Moisturize lower extremities to prevent skin irritation
b. Measure fluid intake and output to decrease edema
c. Elevate lower extremities for postural drainage
d. Provide the client a list of food low in sodium
20. Mr. Regalado will be discharged from your unit within the hour. Nursing actions when preparing a client for discharge include
all EXCEPT:
a. Making a final physical assessment before client leaves the hospital
b. Giving instructions about his medication regimen
c. Walking the client to the hospital exit to ensure his safety
d. Proper recording of pertinent data
Situation 5 Nancy, mother of 2 young kids. 36 years old, had a mammogram and was told that she has breast cysts and that she
may need surgery. This causes her anxiety as shown by increase in her pulse and respiratory rate, sweating and feelings of tension.
21. Considering her level of anxiety, the nurse can best assist Nancy by:
a. Giving her activities to divert her attention
b. Giving detailed explanations about the treatments she will undergo
c. Preparing her and her family in case surgery is not successful
d. Giving her clear but brief information at the level of her understanding
22. Nancy blames God for her situation. She is easily provoked to tears and wants to be left alone, refusing to eat or talk to her
family. A religious person before, she now refuses to pray or go to church stating that God has abandoned her. The nurse
understands that Nancy is grieving for her self and is in the stage of:
a. bargaining
b. denial
c. anger
d. acceptance
23. The nurse visits Nancy and prods her to eat her food. Nancy replies whats the use? My time is running out. The nurses best
response would be:
a. The doctor ordered full diet for you so that you will be strong for surgery.
b. I understand how you fee! but you have 1o try for your childrens sake.
c. Have you told your, doctor how you feel? Are you changing your mind) about surgery?
d. You sound like you are giving up.
24. The nurse feels sad about Nancys illness and tells her head nurse during the end of shift endorsement that its unfair for
Nancy to have cancer when she is still so young and with two kinds. The best response of the head nurse would be:
a. Advise the nurse to be strong and learn to control her feelings
b. Assign the nurse to another client to avoid sympathy for the client
c. Reassure the nurse that the client has hope if she goes through all statements prescribed for her
c. Ask the other nurses what they feel about the patient to find out if they share the same feelings
25. Realizing that she feels angry about Nancys condition, the nurse sees that being self-aware is a conscious process that she
should do in any situation like this because:
a. This is a necessary part of the nurse -client relationship process
b. The nurse is a role model for the client and should be strong
C. How the nurse thinks and feels affect her actions towards her client and her work
d. The nurse has to be therapeutic at all times and should not be affected
Situation 6 Mrs. Seva, 32 years old, asks you about possible problems regarding her elimination now that she is in the menopausal
stage.
26. Instruction on health promotion regarding urinary elimination is important. Which would you include?
a. Hold urine, as long as she can before emptying the bladder to strengthen her sphincters muscles
b. If burning sensation is experienced while voiding, drink pineapple-juice
c. After urination, wipe from anal area up towards the pubis
d. Jell client to empty the bladder at each voiding
27. Mrs. Seva also tells the nurse that she is often constipated. Because she is aging, what physical changes predispose her to
constipation?
a. inhibition of the parasympathetic reflex
b. weakness of sphincter muscles of the anus
c. loss of tone of the smooth muscles of the color
d. decreased ability to absorb fluids in the lower intestines
28. The nurse understands that one of these factors contributes to constipation:
a. excessive exercise
b. high fiber diet
c. no regular time for defecation daily
d. prolonged use of laxatives
29. Mrs. Seva talks about rear of being incontinent due to a prior experience of dribbling urine when laughing or sneezing and
when she has a full bladder. Your most appropriate .instruction would be to:
a. tell client to drink less fluids to avoid accidents
b. instruct client to start wearing thin adult diapers
c. ask the client to bring change of underwear just in case
d. teach client pelvic exercise to strengthen perineal muscles
30. Mrs. Seva asked for instructions for skin care for her mother who has urinary incontinence and is almost always in bed. Your
instruction would focus on prevention of skin irritation and breakdown by
a. Using thick diapers to absorb urine well
b. Drying the skin with baby powder to prevent or mask the smell of ammonia
c. Thorough washing, rising and during of skin area that get wet with urine
d. Making sure that linen are smooth and dry at all times
Situation 7 Using Maslows need theory, Airway, Breathing and Circulation are the physiological needs vital to life. The nurses
knowledge and ability to identify and immediately intervene to meet these needs is important to save lives.
31. Which of these clients has a problem with the transport of oxygen from the lungs to the tissues:
a. Carol with a tumor in the brain
b. Theresa with anemia
c. Sonny Boy with a fracture in the femur
d. Brigette with diarrhea
32. You noted from the lab exams in the chart of Mr. Santos that he has reduced oxygen in the blood.
This condition is called:
a. Cyanosis
b. Hypoxia
c. Hypoxemia
d. Anemia
33. You will nasopharyngeal suctioning Mr. Abad. Your guide for the length of insertion of the tubing for an adult would be:
a. tip of the nose to the base of the .neck
b. the distance from the tip of the nose to the middle of the cheek
c. the distance from the tip of the nose to the tip of the ear lobe
d. eight to ten inches
34. While doing nasopharyngeal suctioning on .Mr. Abad, the nurse can avoid trauma to the area by:
a. Apply suction for at least 20-30 seconds each time to ensure that all secretions are removed
b. Using gloves to prevent introduction of pathogens to the respiratory system
c. Applying no suction while inserting the catheter
d. Rotating catheter as it is inserted with gentle suction
35. Myrna has difficulty breathing when on her back and must sit upright in bed to breath, effectively and comfortably. The nurse
documents this condition as:
a. Apnea
b. Orthopnea
c. Dyspnea
d. Tachypnea
Situation 8 You are assigned to screen for hypertension: Your task is to take blood pressure readings and you are informed about
avoiding the common mistakes in BP taking that lead to false or inaccurate blood pressure readings.
36. When taking blood pressure reading the cuff should be:
a. deflated fully then immediately start second reading for same client
b deflated quickly after inflating up to 180 mmHg
c. large enough to wrap around upper arm of the adult client 1 cm above brachial artery
d. inflated to 30 mmHg above the estimated systolic BP based on palpation of radial or bronchial artery
37. Chronic Obstructive Pulmonary Disease (COPD) in one of the leading causes of death worldwide and is a preventable disease.
The primary cause of COPD is:
a. tobacco hack
b. bronchitis
c. asthma
d. cigarette smoking
38. In your health education class for clients with diabetes you teach, them the areas, for control . Diabetes which include all
EXCEPT:
a. regular physical activity
b. thorough knowledge of foot care
c. prevention nutrition
d. proper nutrition
39. You teach your clients the difference between, Type I (IDDM) and Type II (NDDM) Diabetes. Which of the following is true?
a. both types diabetes mellitus clients are all prone to developing ketosis
b. Type II (NIDDM) is more common and is also preventable compared to Type I (IDDM) diabetes which is genetic in etiology
c. Type I (IDDM) is characterized by fasting hyperglycemia
d. Type II (NIDDM) is characterized by abnormal immune response
40. Lifestyle-related diseases in general share areas common risk factors. These are the following except
a. physical activity
b. smoking
c. genetics
d. nutrition
Situation 9 Nurse Rivera witnesses a vehicular accident near the hospital where she works. She decides to get involved and help
the victims of the accident.
41. Her priority nursing action would be to:
a. Assess damage to property
b. Assist in the police investigation since she is a witness
c. Report the incident immediately to the local police authorities
d. Assess the extent of injuries incurred by the victims, of the accident
42. Priority attention should be given to which of these clients?
a. Linda who shows severe anxiety due to trauma of the accident
b. Ryan who has chest injury, is pate and with difficulty of breathing
c. Noel who has lacerations on the arms with mild-bleeding
c. Andy whose left ankle swelled and has some abrasions
43. In the emergency room, Nurse Rivera is assigned to attend to the client with .lacerations on the arms, while assessing the
extent of the wound the nurse observes that the wound is now starting to bleed profusely. The most immediate nursing action
would be to:
a. Apply antiseptic to prevent infection
b. Clean the wound vigorously of contaminants
c. Control and. reduce bleeding of the wound
d. Bandage the wound and elevate the arm
44. The nurse applies pressure dressing on the bleeding site. This intervention is done to:
a. Reduce the need to change dressing frequently
b. Allow the pus to surface faster
c. Protect the wound from micro organisms in the air
d. Promote hemostasis
45. After the treatment, the client is sent home and asked to come back for follow-up care. Your responsibilities when the client is
to be discharged include the following EXCEPT:
a. Encouraging the client to go to the, outpatient clinic for follow up care
b. Accurate recording, of treatment done and instructions given to client
c. Instructing the client to see you after discharge for further assistance
d. Providing instructions regarding wound care
Situation 10 While working in the clinic, a new client, Geline, 35 years old, arrives for her doctors appointment. As the clinic nurse,
you are to assist the client fill up forms, gather data and make an assessment.
46. The nurse purpose of your initial nursing interview is to:
a. Record pertinent information in the client chart for health team to read
b Assist the client find solutions to her health concerns
c. Understand her lifestyle, health needs and possible problems to develop a plan of care
d. Make nursing diagnoses for identified health problems
47. While interviewing Geline, she starts to moan and doubles up in pain, She tells you that this pain occurs about an hour after
taking black coffee without breakfast for a few weeks now. You will record this as follows:
a. Claims to have abdominal pains after intake of coffee unrelieved by analgesics
b. After drinking coffee, the client experienced severe abdominal pain
c. Client complained of intermittent abdominal pain an hour after drinking coffee
d. Client reported abdominal pain an hour after drinking black coffee for three weeks now
48. Geline tells you that she drinks black coffee frequently within the day to have energy and be wide awake and she eats
nothing for breakfast and eats strictly vegetable salads for lunch and dinner to lose weight. She has lost weight during the past
two weeks, in planning a healthy balanced diet with Geline, you will:
a. Start her off with a cleansing diet to free her body of toxins then change to a vegetarian, diet and drink plenty of fluids
b. Plan a high protein, diet, low carbohydrate diet for her considering her favorite food
c. Instruct her to attend classes in nutrition to find food rich in complex carbohydrates to maintain daily high energy level
d. Discuss with her the importance of eating a variety of food from the major food groups with plenty of fluids
49. Geline tells you that she drinks 4-5 cups of black coffee and diet cola drinks. She also smokes up to a pack of cigarettes daily.
She confesses that she is in her 2nd month of pregnancy but she does not want to become fat that is why she limits her food
intake. You warn or caution her about which of the following?
a. Caffeine products affect the central nervous system and may cause the mother to have a nervous breakdown
b. Malnutrition and its possible effects on growth and development problems in the unborn fetus
c. Caffeine causes a stimulant effect on both the mother and the baby
d. Studies show conclusively that caffeine causes mental retardation
50. Your health education plan for Geline stresses proper diet for a pregnant woman and the prevention
of noncommunicable diseases that are influenced by her lifestyle these include of the following EXCEPT:
a. Cardiovascular diseases
b. Cancer
c. Diabetes Mellitus
d. Osteoporosis
Situation 11 Management of nurse practitioners is done by qualified nursing leaders who have had clinical experience and
management experience.
51. An example of a management function of a nurse is:
a. Teaching patient do breathing and coughing exercises
b. Preparing for a surprise party for a client
c. Performing nursing procedures for clients
d. Directing and evaluating the staff nurses
52. Your head nurse in the unit believes that the staff nurses are not capable of decision making so she makes the decisions for
everyone without consulting anybody. This type of leadership is:
a. Laissez faire leadership
b. Democratic leadership
c. Autocratic leadership
d. Managerial leadership
53. When the head nurse in your ward plots and approves your work schedules and directs your work, she is demonstrating:
a. Responsibility
b. Delegation
c. Accountability
d. Authority
54. The following tasks can be safely delegated by a nurse to a non-nurse health worker EXCEPT:
a. Transfer a client from bed to chair
b. Change IV infusions
c. Irrigation of a nasogastric tube
d. Take vital signs
55. You made a mistake in giving the medicine to the wrong client You notify the clients doctor and write an incident report. You
are demonstrating:
a. Responsibility
b. Accountability
c. Authority
d. Autocracy
Situation 12 Mr. Dizon, 84 years old, is brought to the Emergency Room for complaint of hypertension flushed face, severe
headache, and nausea. You are doing the initial assessment of vital signs.
56. You are to measure the clients initial blood pressure reading by doing all of the following EXCEPT:
a. Take the blood pressure reading on both arms for comparison
b. Listen to and identify the phases of Korotkoffs sounds
c. Pump the cuff up to around 50 mmHg above the point where the pulse is obliterated
d. Observe procedures for infection control
57. A pulse oximeter is attached to Mr. Dizons finger to:
a. Determine if the clients hemoglobin level is low and if he needs blood transfusion
b. Check level of clients tissue perfusion
c. Measure the efficacy of the clients antihypertensive medications
d. Detect oxygen saturation of arterial blood before symptoms of hypoxemia develops
58. After a few hours in the Emergency Room, Mr. Dizon is admitted to the ward with an order of hourly monitoring of blood
pressure. The nurse finds that the cuff is too narrow and this will cause the blood pressure reading to be:
a. Inconsistent
b. low systolic and high diastolic pressure
c. higher than what the reading should be
d. lower than what the reading should be
59. Through the clients health history, you gather that Mr. Dizon smokes and drinks coffee. When taking the blood pressure of a
client who recently smoked or drank coffee, how long should be the nurse wait before taking the clients blood pressure for
accurate reading?
a. 15 minutes
b. 30 minutes
c. 1 hour
d. 5 minutes
60. While the client has the pulse oximeter on his fingertip, you notice that the sunlight is shining on .the area where the
oximeter is. Your action will be to:
a. Set and turn on the alarm of the oximeter
b. Do nothing since there is no identified problem
c. Cover the fingertip sensor with a towel or bedsheet
d. Change the location of the sensor every four hours
Situation 13 The nurses understanding of ethico-legal responsibilities will guide his/her nursing practice.
61. The principles that .govern right and proper conducts of a person regarding life, biology and the health professions is referred
to as:
a. Morality
b. Religion
c. Values
d. Bioethics
62. The purpose of having nurses code of ethics is:
a. Delineate the scope and areas of nursing practice
b. Identify nursing action recommended for specific health care situations
c. To help the public understand professional conduct, expected of nurses
d. To define the roles and functions of the health caregiver, nurses, clients
63. The most important nursing responsibility where ethical situations emerge in patient care is to:
a. Act only when advised that the action is ethically sound
b. Not take sides remain neutral and fair
c. Assume that ethical questions are the responsibility: of the health team
d. Be accountable for his or her own actions
64. You inform the patient about his rights which include the following EXCEPT:
a. Right to expect reasonable continuity of care
b. Right to consent to or decline to participate in research studies or experiments
c. Right to obtain information about another patient
d. Right to expect that the records about his care will be treated as confidential
65. The principle states that a person has unconditional worth and has the capacity to determine his own destiny.
a. Bioethics
b. Justice
c. Fidelity
d. Autonomy
Situation 14 Your director of nursing wants to improve the quality of health care offered in the hospital. As a staff nurse in that
hospital you know that this entails quality assurance programs.
66. The following mechanisms can be utilized as part of the quality assessment program of your hospital EXCEPT:
a. Patient satisfaction surveys provided
b. Peer review clinical records of care of client
c. Review of the Nursing Intervention Classification
d. Use of Nursing Interventions Classification
67. The nurse of the Standards of Nursing Practice is important in the hospital. Which of the following statements best describes
what it is?
a. These are statements that describe the maximum or highest level of acceptable performance in nursing practice.
b. It refers to the scope of nursing as defined in Republic Act 9173
c. It is a license issued by the Professional Regulation Commission to protect the public from substandard nursing practice.
d. The Standards of care includes the various steps of the nursing process and the standards of professional performance.
68. You are taking care of critically ill client and the doctor in charge calls to order a DNR (do not resuscitate) for the client. Which
of the following is the appropriate action when getting DNR order over the phone?
a. Have the registered nurse, family spokesperson, nurse supervisor and doctor sign
b. Have two nurses validate the phone order, both nurses sign the order and the doctor should sign his order within 24 hours.
c. Have the registered nurse, family and doctor sign the order
d. Have 1 nurse take the order and sign it and have the doctor sign it within 24 hours
69. To ensure the client safety before starting blood transfusion the following are needed before the procedure can be done
EXCEPT:
a. take baseline vital signs
b. blood should be warmed to room temperature for 30 minutes before blood transfusion is administered
c. have two nurses verify client identification, blood type, unit number and expiration date of blood
d. get a consent signed for blood transfusion
70. Part of standards of care has to do with the use of restraints. Which of the following statements is NOT true?
a. Doctors order for restraints should be signed within 24 hours
b. Remove and reapply restraints every two hours
c. Check clients pulse, blood pressure and circulation every four hours
d. Offer food and toileting every two hours
Situation 15 During the NUTRITION EDUCATION class discussion a 58 year old man, Mr. Bruno shows increased interest.
71. Mr. Bruno asks what the normal allowable salt intake is. Your best response to Mr. Bruno is:
a. 1 tsp of salt/day with iodine and sprinkle of MSG
b. 5 gms per day or 1 tsp of table salt/day
c. 1 tbsp of salt/day with some patis and toyo
d. 1 tsp of salt/day but not patis or toyo
72. Your instructions to reduce or limit salt intake include all the following EXCEPT:
a. eat natural food with little or no salt added
b. limit use of table salt and use condiments instead
c. use herbs and spices
d. limit intake of preserved or processed food
73. Teaching strategies and approaches when giving nutrition education is influenced by age, sex and immediate concerns of the
group. Your presentation for a group of young mothers would be best if you focus on:
a. diets limited in salt and fat
b. harmful effect on drugs and alcohol intake
c. commercial preparation of dishes
d. cooking demonstration and meal planning
74. Cancer cure is dependent on
a. use of alternative methods of healing
b. watching out for warning signs of cancer
c. proficiency in doing breast self-examination
d. early detection and prompt treatment
75. The role of the health worker in health education is to:
a. report incidence of non-communicable disease to community health center
b. educate as many people about warning signs of non-communicable diseases
c. focus on smoking cessation projects
d. monitor clients with hypertension
Situation 16 You are assigned to take care of 10 patients during the morning shift. The endorsement includes the IV infusion and
medications for these clients.
76. Mr. Felipe, 36 years old is to be given 2700 ml of D5RL to infuse for 18 hours starting at 8am. At what rate should the IV fluid
be flowing hourly?
a. 100 ml/hour
b. 210 ml/hour
c. 150 ml/hour
d. 90 ml/hour
77. Mr. Atienza is to receive 150 mg/hour of D5W IV infusion for 12 hours for a total of 1800 ml. He is also losing gastric fluid
which must be replaced every two hours. Between 8am to 10am. Mr. Atienza has lost 250 ml of gastric fluid. How much fluid
should he receive at 11am?
a. 350 ml/hour
b. 275 ml/hour
c. 400 ml/hour
d. 200 ml/hour
78. You are to apply a transdermal patch of nitroglycerin to your client. The following important guidelines to observe EXCEPT:
a. Apply to hairlines clean are of the skin not subject to much wrinkling
b. Patches may be applied to distal part of the extremities like forearm
c. Change application and site regularly to prevent irritation of the skin
d. Wear gloves to avoid any medication of your hand
79. You will be applying eye drops to Miss Romualdez. After checking all the necessary information and cleaning the affected
eyelid and eyelashes you administer the ophthalmic drops by instilling the eye drops.
a. directly onto the cornea
b. pressing on the lacrimal duct
c. into the outer third of the lower conjunctival sac
d. from the inner canthus going towards the side of the eye
80. When applying eye ointment, the following guidelines apply EXCEPT:
a. squeeze about 2 cm of ointment and gently close but not squeeze eye
b. apply ointment from the inner canthus going outward of the affected eye
c. discard the first bead of the eye ointment before application because the tube likely to expel more than desired amount of
ointment
d. hold the tube above the conjunctival sac do not let tip touch the conjunctiva
Situation 17 The staff nurse supervisor request all the staff nurses to brainstorm and learn ways to instruct diabetic clients on
self-administration of insulin. She wants to ensure that there are nurses available daily to do health education classes.
81. The plan of the nurse supervisor is an example of
a. in service education process
b. efficient management of human resources
c. increasing human resources
d. primary prevention
82. When Mrs. Guevarra, a nurse, delegates aspects of the clients care to the nurse-aide who is an unlicensed staff, Mrs.
Guevarra.
a. makes the assignment to teach the staff member
b. is assigning the responsibility to the aide but not the accountability for those tasks
c. does not have to supervise or evaluate the aide
d. most know how to perform task delegated
83. Connie, the-new nurse, appears tired and sluggish and lacks the enthusiasms she give six weeks ago when she started the job.
The nurse supervisor should:
a. empathize with the nurse and listen to her
b. tell her to take the day off
c. discuss how she is adjusting to her new job
d. ask about her family life
84. Process of formal negotiations of working conditions between a group of registered nurses and employer is:
a. grievance
b. arbitration
c. collective bargaining
d. strike
85. You are attending a certification program on cardiopulmonary resuscitation (CPR) offered and required by the hospital
employing you. This is:
a. professional course towards credits
b. in-service education
c. advance training
d. continuing education
Situation 18 There are various developments in health education that the nurse should know about.
86. The provision of health information in the rural areas nationwide through television and radio programs and video
conferencing is referred to as:
a. Community health program
b. Telehealth program
c. Wellness program
d. Red cross program
87. A nearby community provides blood pressure screening, height and weight measurement smoking cessation classes and
aerobics class services. This type of program is referred to as:
a. outreach program
b. hospital extension program
c. barangay health center
d. wellness center
88. Part of teaching client in health promotion is responsibility for ones health. When Danica states she need to improve her
nutritional status this means:
a. Goals and interventions to be followed by client are based on nurses priorities
b. Goals and intervention developed by nurse and client should be approved by the doctor
c. Nurse will decide goals and, interventions needed to meet client goals
d. Client will decide the goals and interventions required to meet her goals
89. Nurse Beatrice is providing tertiary prevention to Mrs. De Villa. An example of tertiary prevention is:
a. Marriage counseling
b. Self-examination for breast cancer
c. Identifying complication of diabetes
d. Poison, control
90. Mrs. Ostrea has a schedule for Pap Smear. She has a strong family history of cervical cancer. This is an example of:
a. tertiary prevention
b. secondary prevention
c. health screening
d. primary prevention
Situation: 19 Ronnie has a vehicular accident where he sustained injury to his left ankle. In the Emergency Room, you notice how
anxious he looks.
91. You establish rapport with him and to reduce his anxiety you initially
a. Take him to the radiology, section for X-ray of affected extremity
b. Identify yourself and state your purpose in being with the client
c. Talk to the physician for an order of Valium
d. Do inspection and palpation to check extent of his injuries
92. While doing your assessment, Ronnie asks you Do I have a fracture? I dont want to have a cast. The most appropriate
nursing response would be:
a. You have to have an X-ray first to know if you have a fracture.
b. Why do you sound so scared? It is just a cast and its not painful
c. You seem to be concerned about being in a cast.
d. Based on my assessment, there doesnt seem to be a fracture.
93. A technique in physical examination that is use to assess the movement of air through the tracheobronchial tree:
A. Palpation
B. Auscultation
C. Inspection
D. Percussion
94. An instrument used for auscultation is:
A. Percussion-hammer
B. Audiometer
C. Stethoscope
D. Sphygmomanometer
95. Resonance is best describe as:
A. Sounds created by air filled lungs
B. Short, high pitch and thudding
C. Moderately loud with musical quality
D. Drum-like
96. The best position for examining the rectum is:
A. Prone
B. Sims
C. Knee-chest
D. Lithotomy
97. It refers to the manner of walking
A. Gait
B. Range of motion
C. Flexion and extension
D. Hopping
98. The nurse asked the client to read the Snellen chart. Which of the following is tested:
A. Optic
B. Olfactory
C. Oculomotor
D. Trochlear
99. Another name for knee-chest position is:
A. Genu-dorsal
B. Genu-pectoral
C. Lithotomy
D. Sims
100. The nurse prepares an IM injection that is irritating to the subcutaneous tissue. Which of the following is the best action in
order to prevent tracking of the medication
A. Use a small gauge needle
B. Apply ice on the injection site
C. Administer at a 45 angle
D. Use the Z-track technique
Answers
Here are the answers for the exam. Unfortunately, rationales are not given. If you need clarifications or disputes, please direct them
to the comments section and well be glad to give you an explanation.
1. C
2. B
3. D
4. D
5. B
6. B
7. C
8. D
9. B
10. B
11. B
12. C
13. C
14. D
15. A
16. A
17. C
18. B
19. A
20. C
21. D
22. C
23. D
24. D
25. C
26. D
27. C
28. D
29. D
30. C
31. B
32. C
33. C
34. C
35. B
36. D
37. D
38. B
39. B
40. C
41. D
42. B
43. D
44. D
45. C
46. C
47. D
48. D
49. B
50. D
51. D
52. C
53. D
54. B
55. B
56. C
57. D
58. C
59. B
60. C
61. D
62. C
63. D
64. C
65. D
66. D
67. A
68. D
69. D
70. C
71. B
72. B
73. D
74. D
75. B
76. C
77. B
78. B
79. B
80. C
81. C
82. B
83. C
84. C
85. B
86. B
87. A
88. D
89. C
90. B
91. B
92. C
93. B
94. C
95. A
96. C
97. A
98. A
99. B
100. D








Fundamentals of Nursing Exam 14: NLE Pre-Boards (100 Items)
InText Mode: All questions and answers are given for reading and answering at your own pace. You can also copy this exam and
make a print out.
1. The nurse-in-charge in labor and delivery unit administered a dose of terbutaline to a client without checking the clients pulse.
The standard that would be used to determine if the nurse was negligent is:
a. The physicians orders.
b. The action of a clinical nurse specialist who is recognized expert in the field.
c. The statement in the drug literature about administration of terbutaline.
d. The actions of a reasonably prudent nurse with similar education and experience.
2. Nurse Trish is caring for a female client with a history of GI bleeding, sickle cell disease, and a platelet count of 22,000/l. The
female client is dehydrated and receiving dextrose 5% in half-normal saline solution at 150 ml/hr. The client complains of severe
bone pain and is scheduled to receive a dose of morphine sulfate. In administering the medication, Nurse Trish should avoid
which route?
a. I.V
b. I.M
c. Oral
d. S.C
3. Dr. Garcia writes the following order for the client who has been recently admitted Digoxin .125 mg P.O. once daily. To
prevent a dosage error, how should the nurse document this order onto the medication administration record?
a. Digoxin .1250 mg P.O. once daily
b. Digoxin 0.1250 mg P.O. once daily
c. Digoxin 0.125 mg P.O. once daily
d. Digoxin .125 mg P.O. once daily
4. A newly admitted female client was diagnosed with deep vein thrombosis. Which nursing diagnosis should receive the highest
priority?
a. Ineffective peripheral tissue perfusion related to venous congestion.
b. Risk for injury related to edema.
c. Excess fluid volume related to peripheral vascular disease.
d. Impaired gas exchange related to increased blood flow.
5. Nurse Betty is assigned to the following clients. The client that the nurse would see first after endorsement?
a. A 34 year-old postoperative appendectomy client of five hours who is complaining of pain.
b. A 44 year-old myocardial infarction (MI) client who is complaining of nausea.
c. A 26 year-old client admitted for dehydration whose intravenous (IV) has infiltrated.
d. A 63 year-old postoperative abdominal hysterectomy client of three days whose incisional dressing is saturated with
serosanguinous fluid.
6. Nurse Gail places a client in a four-point restraint following orders from the physician. The client care plan should include:
a. Assess temperature frequently.
b. Provide diversional activities.
c. Check circulation every 15-30 minutes.
d. Socialize with other patients once a shift.
7. A male client who has severe burns is receiving H2 receptor antagonist therapy. The nurse In-charge knows the purpose of this
therapy is to:
a. Prevent stress ulcer
b. Block prostaglandin synthesis
c. Facilitate protein synthesis.
d. Enhance gas exchange
8. The doctor orders hourly urine output measurement for a postoperative male client. The nurse Trish records the following
amounts of output for 2 consecutive hours: 8 a.m.: 50 ml; 9 a.m.: 60 ml. Based on these amounts, which action should the nurse
take?
a. Increase the I.V. fluid infusion rate
b. Irrigate the indwelling urinary catheter
c. Notify the physician
d. Continue to monitor and record hourly urine output
9. Tony, a basketball player twist his right ankle while playing on the court and seeks care for ankle pain and swelling. After the
nurse applies ice to the ankle for 30 minutes, which statement by Tony suggests that ice application has been effective?
a. My ankle looks less swollen now.
b. My ankle feels warm.
c. My ankle appears redder now.
d. I need something stronger for pain relief
10. The physician prescribes a loop diuretic for a client. When administering this drug, the nurse anticipates that the client may
develop which electrolyte imbalance?
a. Hypernatremia
b. Hyperkalemia
c. Hypokalemia
d. Hypervolemia
11. She finds out that some managers have benevolent-authoritative style of management. Which of the following behaviors will
she exhibit most likely?
a. Have condescending trust and confidence in their subordinates.
b. Gives economic and ego awards.
c. Communicates downward to staffs.
d. Allows decision making among subordinates.
12. Nurse Amy is aware that the following is true about functional nursing
a. Provides continuous, coordinated and comprehensive nursing services.
b. One-to-one nurse patient ratio.
c. Emphasize the use of group collaboration.
d. Concentrates on tasks and activities.
13. Which type of medication order might read Vitamin K 10 mg I.M. daily 3 days?
a. Single order
b. Standard written order
c. Standing order
d. Stat order
14. A female client with a fecal impaction frequently exhibits which clinical manifestation?
a. Increased appetite
b. Loss of urge to defecate
c. Hard, brown, formed stools
d. Liquid or semi-liquid stools
15.Nurse Linda prepares to perform an otoscopic examination on a female client. For proper visualization, the nurse should
position the clients ear by:
a. Pulling the lobule down and back
b. Pulling the helix up and forward
c. Pulling the helix up and back
d. Pulling the lobule down and forward
16. Which instruction should Nurse Tommy would give to a male client who is having external radiation therapy:
a. Protect the irritated skin from sunlight.
b. Eat 3 to 4 hours before treatment.
c. Wash the skin over regularly.
d. Apply lotion or oil to the irradiated area when it is red or sore.
17. In assisting a female client for immediate surgery, the nurse In-charge is aware that she should:
a. Encourage the client to void following preoperative medication.
b. Explore the clients fears and anxieties about the surgery.
c. Assist the client in removing dentures and nail polish.
d. Encourage the client to drink water prior to surgery.
18. A male client is admitted and diagnosed with acute pancreatitis after a holiday celebration of excessive food and alcohol.
Which assessment finding reflects this diagnosis?
a. Blood pressure above normal range.
b. Presence of crackles in both lung fields.
c. Hyperactive bowel sounds
d. Sudden onset of continuous epigastric and back pain.
19. Which dietary guidelines are important for nurse Oliver to implement in caring for the client with burns?
a. Provide high-fiber, high-fat diet
b. Provide high-protein, high-carbohydrate diet.
c. Monitor intake to prevent weight gain.
d. Provide ice chips or water intake.
20. Nurse Hazel will administer a unit of whole blood, which priority information should the nurse have about the client?
a. Blood pressure and pulse rate.
b. Height and weight.
c. Calcium and potassium levels
d. Hgb and Hct levels.
21. Nurse Michelle witnesses a female client sustain a fall and suspects that the leg may be broken. The nurse takes which priority
action?
a. Takes a set of vital signs.
b. Call the radiology department for X-ray.
c. Reassure the client that everything will be alright.
d. Immobilize the leg before moving the client.
22. A male client is being transferred to the nursing unit for admission after receiving a radium implant for bladder cancer. The
nurse in-charge would take which priority action in the care of this client?
a. Place client on reverse isolation.
b. Admit the client into a private room.
c. Encourage the client to take frequent rest periods.
d. Encourage family and friends to visit.
23. A newly admitted female client was diagnosed with agranulocytosis. The nurse formulates which priority nursing diagnosis?
a. Constipation
b. Diarrhea
c. Risk for infection
d. Deficient knowledge
24. A male client is receiving total parenteral nutrition suddenly demonstrates signs and symptoms of an air embolism. What is
the priority action by the nurse?
a. Notify the physician.
b. Place the client on the left side in the Trendelenburg position.
c. Place the client in high-Fowlers position.
d. Stop the total parenteral nutrition.
25. Nurse May attends an educational conference on leadership styles. The nurse is sitting with a nurse employed at a large
trauma center who states that the leadership style at the trauma center is task-oriented and directive. The nurse determines that
the leadership style used at the trauma center is:
a. Autocratic.
b. Laissez-faire.
c. Democratic.
d. Situational
26. The physician orders DS 500 cc with KCl 10 mEq/liter at 30 cc/hr. The nurse in-charge is going to hang a 500 cc bag. KCl is
supplied 20 mEq/10 cc. How many ccs of KCl will be added to the IV solution?
a. .5 cc
b. 5 cc
c. 1.5 cc
d. 2.5 cc
27. A child of 10 years old is to receive 400 cc of IV fluid in an 8 hour shift. The IV drip factor is 60. The IV rate that will deliver this
amount is:
a. 50 cc/ hour
b. 55 cc/ hour
c. 24 cc/ hour
d. 66 cc/ hour
28. The nurse is aware that the most important nursing action when a client returns from surgery is:
a. Assess the IV for type of fluid and rate of flow.
b. Assess the client for presence of pain.
c. Assess the Foley catheter for patency and urine output
d. Assess the dressing for drainage.
29. Which of the following vital sign assessments that may indicate cardiogenic shock after myocardial infarction?
a. BP 80/60, Pulse 110 irregular
b. BP 90/50, Pulse 50 regular
c. BP 130/80, Pulse 100 regular
d. BP 180/100, Pulse 90 irregular
30.Which is the most appropriate nursing action in obtaining a blood pressure measurement?
a. Take the proper equipment, place the client in a comfortable position, and record the appropriate information in the clients
chart.
b. Measure the clients arm, if you are not sure of the size of cuff to use.
c. Have the client recline or sit comfortably in a chair with the forearm at the level of the heart.
d. Document the measurement, which extremity was used, and the position that the client was in during the measurement.
31. Asking the questions to determine if the person understands the health teaching provided by the nurse would be included
during which step of the nursing process?
a. Assessment
b. Evaluation
c. Implementation
d. Planning and goals
32. Which of the following item is considered the single most important factor in assisting the health professional in arriving at a
diagnosis or determining the persons needs?
a. Diagnostic test results
b. Biographical date
c. History of present illness
d. Physical examination
33. In preventing the development of an external rotation deformity of the hip in a client who must remain in bed for any period
of time, the most appropriate nursing action would be to use:
a. Trochanter roll extending from the crest of the ileum to the midthigh.
b. Pillows under the lower legs.
c. Footboard
d. Hip-abductor pillow
34. Which stage of pressure ulcer development does the ulcer extend into the subcutaneous tissue?
a. Stage I
b. Stage II
c. Stage III
d. Stage IV
35. When the method of wound healing is one in which wound edges are not surgically approximated and integumentary
continuity is restored by granulations, the wound healing is termed
a. Second intention healing
b. Primary intention healing
c. Third intention healing
d. First intention healing
36. An 80-year-old male client is admitted to the hospital with a diagnosis of pneumonia. Nurse Oliver learns that the client lives
alone and hasnt been eating or drinking. When assessing him for dehydration, nurse Oliver would expect to find:
a. Hypothermia
b. Hypertension
c. Distended neck veins
d. Tachycardia
37. The physician prescribes meperidine (Demerol), 75 mg I.M. every 4 hours as needed, to control a clients postoperative pain.
The package insert is Meperidine, 100 mg/ml. How many milliliters of meperidine should the client receive?
a. 0.75
b. 0.6
c. 0.5
d. 0.25
38. A male client with diabetes mellitus is receiving insulin. Which statement correctly describes an insulin unit?
a. Its a common measurement in the metric system.
b. Its the basis for solids in the avoirdupois system.
c. Its the smallest measurement in the apothecary system.
d. Its a measure of effect, not a standard measure of weight or quantity.
39. Nurse Oliver measures a clients temperature at 102 F. What is the equivalent Centigrade temperature?
a. 40.1 C
b. 38.9 C
c. 48 C
d. 38 C
40. The nurse is assessing a 48-year-old client who has come to the physicians office for his annual physical exam. One of the first
physical signs of aging is:
a. Accepting limitations while developing assets.
b. Increasing loss of muscle tone.
c. Failing eyesight, especially close vision.
d. Having more frequent aches and pains.
41. The physician inserts a chest tube into a female client to treat a pneumothorax. The tube is connected to water-seal drainage.
The nurse in-charge can prevent chest tube air leaks by:
a. Checking and taping all connections.
b. Checking patency of the chest tube.
c. Keeping the head of the bed slightly elevated.
d. Keeping the chest drainage system below the level of the chest.
42. Nurse Trish must verify the clients identity before administering medication. She is aware that the safest way to verify
identity is to:
a. Check the clients identification band.
b. Ask the client to state his name.
c. State the clients name out loud and wait a client to repeat it.
d. Check the room number and the clients name on the bed.
43. The physician orders dextrose 5 % in water, 1,000 ml to be infused over 8 hours. The I.V. tubing delivers 15 drops/ml. Nurse
John should run the I.V. infusion at a rate of:
a. 30 drops/minute
b. 32 drops/minute
c. 20 drops/minute
d. 18 drops/minute
44. If a central venous catheter becomes disconnected accidentally, what should the nurse in-charge do immediately?
a. Clamp the catheter
b. Call another nurse
c. Call the physician
d. Apply a dry sterile dressing to the site.
45. A female client was recently admitted. She has fever, weight loss, and watery diarrhea is being admitted to the facility. While
assessing the client, Nurse Hazel inspects the clients abdomen and notice that it is slightly concave. Additional assessment should
proceed in which order:
a. Palpation, auscultation, and percussion.
b. Percussion, palpation, and auscultation.
c. Palpation, percussion, and auscultation.
d. Auscultation, percussion, and palpation.
46. Nurse Betty is assessing tactile fremitus in a client with pneumonia. For this examination, nurse Betty should use the:
a. Fingertips
b. Finger pads
c. Dorsal surface of the hand
d. Ulnar surface of the hand
47. Which type of evaluation occurs continuously throughout the teaching and learning process?
a. Summative
b. Informative
c. Formative
d. Retrospective
48. A 45-year-old client, has no family history of breast cancer or other risk factors for this disease. Nurse John should instruct her
to have mammogram how often?
a. Twice per year
b. Once per year
c. Every 2 years
d. Once, to establish baseline
49. A male client has the following arterial blood gas values: pH 7.30; Pao2 89 mmHg; Paco2 50 mmHg; and HCO3 26 mEq/L.
Based on these values, Nurse Patricia should expect which condition?
a. Respiratory acidosis
b. Respiratory alkalosis
c. Metabolic acidosis
d. Metabolic alkalosis
50. Nurse Len refers a female client with terminal cancer to a local hospice. What is the goal of this referral?
a. To help the client find appropriate treatment options.
b. To provide support for the client and family in coping with terminal illness.
c. To ensure that the client gets counseling regarding health care costs.
d. To teach the client and family about cancer and its treatment.
51. When caring for a male client with a 3-cm stage I pressure ulcer on the coccyx, which of the following actions can the nurse
institute independently?
a. Massaging the area with an astringent every 2 hours.
b. Applying an antibiotic cream to the area three times per day.
c. Using normal saline solution to clean the ulcer and applying a protective dressing as necessary.
d. Using a povidone-iodine wash on the ulceration three times per day.
52. Nurse Oliver must apply an elastic bandage to a clients ankle and calf. He should apply the bandage beginning at the clients:
a. Knee
b. Ankle
c. Lower thigh
d. Foot
53. A 10 year old child with type 1 diabetes develops diabetic ketoacidosis and receives a continuous insulin infusion. Which
condition represents the greatest risk to this child?
a. Hypernatremia
b. Hypokalemia
c. Hyperphosphatemia
d. Hypercalcemia
54. Nurse Len is administering sublingual nitroglycerin (Nitrostat) to the newly admitted client. Immediately afterward, the client
may experience:
a. Throbbing headache or dizziness
b. Nervousness or paresthesia.
c. Drowsiness or blurred vision.
d. Tinnitus or diplopia.
55. Nurse Michelle hears the alarm sound on the telemetry monitor. The nurse quickly looks at the monitor and notes that a
client is in a ventricular tachycardia. The nurse rushes to the clients room. Upon reaching the clients bedside, the nurse would
take which action first?
a. Prepare for cardioversion
b. Prepare to defibrillate the client
c. Call a code
d. Check the clients level of consciousness
56. Nurse Hazel is preparing to ambulate a female client. The best and the safest position for the nurse in assisting the client is to
stand:
a. On the unaffected side of the client.
b. On the affected side of the client.
c. In front of the client.
d. Behind the client.
57. Nurse Janah is monitoring the ongoing care given to the potential organ donor who has been diagnosed with brain death. The
nurse determines that the standard of care had been maintained if which of the following data is observed?
a. Urine output: 45 ml/hr
b. Capillary refill: 5 seconds
c. Serum pH: 7.32
d. Blood pressure: 90/48 mmHg
58. Nurse Amy has an order to obtain a urinalysis from a male client with an indwelling urinary catheter. The nurse avoids which
of the following, which contaminate the specimen?
a. Wiping the port with an alcohol swab before inserting the syringe.
b. Aspirating a sample from the port on the drainage bag.
c. Clamping the tubing of the drainage bag.
d. Obtaining the specimen from the urinary drainage bag.
59. Nurse Meredith is in the process of giving a client a bed bath. In the middle of the procedure, the unit secretary calls the nurse
on the intercom to tell the nurse that there is an emergency phone call. The appropriate nursing action is to:
a. Immediately walk out of the clients room and answer the phone call.
b. Cover the client, place the call light within reach, and answer the phone call.
c. Finish the bed bath before answering the phone call.
d. Leave the clients door open so the client can be monitored and the nurse can answer the phone call.
60. Nurse Janah is collecting a sputum specimen for culture and sensitivity testing from a client who has a productive cough.
Nurse Janah plans to implement which intervention to obtain the specimen?
a. Ask the client to expectorate a small amount of sputum into the emesis basin.
b. Ask the client to obtain the specimen after breakfast.
c. Use a sterile plastic container for obtaining the specimen.
d. Provide tissues for expectoration and obtaining the specimen.
61. Nurse Ron is observing a male client using a walker. The nurse determines that the client is using the walker correctly if the
client:
a. Puts all the four points of the walker flat on the floor, puts weight on the hand pieces, and then walks into it.
b. Puts weight on the hand pieces, moves the walker forward, and then walks into it.
c. Puts weight on the hand pieces, slides the walker forward, and then walks into it.
d. Walks into the walker, puts weight on the hand pieces, and then puts all four points of the walker flat on the floor.
62. Nurse Amy has documented an entry regarding client care in the clients medical record. When checking the entry, the nurse
realizes that incorrect information was documented. How does the nurse correct this error?
a. Erases the error and writes in the correct information.
b. Uses correction fluid to cover up the incorrect information and writes in the correct information.
c. Draws one line to cross out the incorrect information and then initials the change.
d. Covers up the incorrect information completely using a black pen and writes in the correct information
63. Nurse Ron is assisting with transferring a client from the operating room table to a stretcher. To provide safety to the client,
the nurse should:
a. Moves the client rapidly from the table to the stretcher.
b. Uncovers the client completely before transferring to the stretcher.
c. Secures the client safety belts after transferring to the stretcher.
d. Instructs the client to move self from the table to the stretcher.
64. Nurse Myrna is providing instructions to a nursing assistant assigned to give a bed bath to a client who is on contact
precautions. Nurse Myrna instructs the nursing assistant to use which of the following protective items when giving bed bath?
a. Gown and goggles
b. Gown and gloves
c. Gloves and shoe protectors
d. Gloves and goggles
65. Nurse Oliver is caring for a client with impaired mobility that occurred as a result of a stroke. The client has right sided arm
and leg weakness. The nurse would suggest that the client use which of the following assistive devices that would provide the
best stability for ambulating?
a. Crutches
b. Single straight-legged cane
c. Quad cane
d. Walker
66. A male client with a right pleural effusion noted on a chest X-ray is being prepared for thoracentesis. The client experiences
severe dizziness when sitting upright. To provide a safe environment, the nurse assists the client to which position for the
procedure?
a. Prone with head turned toward the side supported by a pillow.
b. Sims position with the head of the bed flat.
c. Right side-lying with the head of the bed elevated 45 degrees.
d. Left side-lying with the head of the bed elevated 45 degrees.
67. Nurse John develops methods for data gathering. Which of the following criteria of a good instrument refers to the ability of
the instrument to yield the same results upon its repeated administration?
a. Validity
b. Specificity
c. Sensitivity
d. Reliability
68. Harry knows that he has to protect the rights of human research subjects. Which of the following actions of Harry ensures
anonymity?
a. Keep the identities of the subject secret
b. Obtain informed consent
c. Provide equal treatment to all the subjects of the study.
d. Release findings only to the participants of the study
69. Patients refusal to divulge information is a limitation because it is beyond the control of the researcher. What type of
research is appropriate for this study?
a. Descriptive- correlational
b. Experiment
c. Quasi-experiment
d. Historical
70. Nurse Ronald is aware that the best tool for data gathering is?
a. Interview schedule
b. Questionnaire
c. Use of laboratory data
d. Observation
71. Monica is aware that there are times when only manipulation of study variables is possible and the elements of control or
randomization are not attendant. Which type of research is referred to this?
a. Field study
b. Quasi-experiment
c. Solomon-Four group design
d. Posttest only design
72. Cherry notes down ideas that were derived from the description of an investigation written by the person who conducted it.
Which type of reference source refers to this?
a. Footnote
b. Bibliography
c. Primary source
d. Endnotes
73. When Nurse Trish is providing care to his patient, she must remember that her duty is bound not to do doing any action that
will cause the patient harm. This is the meaning of the bioethical principle:
a. Non-maleficence
b. Beneficence
c. Justice
d. Solidarity
74. When a nurse in-charge causes an injury to a female patient and the injury caused becomes the proof of the negligent act, the
presence of the injury is said to exemplify the principle of:
a. Force majeure
b. Respondeat superior
c. Res ipsa loquitur
d. Holdover doctrine
75. Nurse Myrna is aware that the Board of Nursing has quasi-judicial power. An example of this power is:
a. The Board can issue rules and regulations that will govern the practice of nursing
b. The Board can investigate violations of the nursing law and code of ethics
c. The Board can visit a school applying for a permit in collaboration with CHED
d. The Board prepares the board examinations
76. When the license of nurse Krina is revoked, it means that she:
a. Is no longer allowed to practice the profession for the rest of her life
b. Will never have her/his license re-issued since it has been revoked
c. May apply for re-issuance of his/her license based on certain conditions stipulated in RA 9173
d. Will remain unable to practice professional nursing
77. Ronald plans to conduct a research on the use of a new method of pain assessment scale. Which of the following is the second
step in the conceptualizing phase of the research process?
a. Formulating the research hypothesis
b. Review related literature
c. Formulating and delimiting the research problem
d. Design the theoretical and conceptual framework
78. The leader of the study knows that certain patients who are in a specialized research setting tend to respond psychologically
to the conditions of the study. This referred to as:
a. Cause and effect
b. Hawthorne effect
c. Halo effect
d. Horns effect
79. Mary finally decides to use judgment sampling on her research. Which of the following actions of is correct?
a. Plans to include whoever is there during his study.
b. Determines the different nationality of patients frequently admitted and decides to get representations samples from each.
c. Assigns numbers for each of the patients, place these in a fishbowl and draw 10 from it.
d. Decides to get 20 samples from the admitted patients
80. The nursing theorist who developed transcultural nursing theory is:
a. Florence Nightingale
b. Madeleine Leininger
c. Albert Moore
d. Sr. Callista Roy
81. Marion is aware that the sampling method that gives equal chance to all units in the population to get picked is:
a. Random
b. Accidental
c. Quota
d. Judgment
82. John plans to use a Likert Scale to his study to determine the:
a. Degree of agreement and disagreement
b. Compliance to expected standards
c. Level of satisfaction
d. Degree of acceptance
83. Which of the following theorem addresses the four modes of adaptation?
a. Madeleine Leininger
b. Sr. Callista Roy
c. Florence Nightingale
d. Jean Watson
84. Ms. Garcia is responsible to the number of personnel reporting to her. This principle refers to:
a. Span of control
b. Unity of command
c. Downward communication
d. Leader
85. Ensuring that there is an informed consent on the part of the patient before a surgery is done, illustrates the bioethical
principle of:
a. Beneficence
b. Autonomy
c. Veracity
d. Non-maleficence
86. Nurse Reese is teaching a female client with peripheral vascular disease about foot care; Nurse Reese should include which
instruction?
a. Avoid wearing cotton socks.
b. Avoid using a nail clipper to cut toenails.
c. Avoid wearing canvas shoes.
d. Avoid using cornstarch on feet.
87. A client is admitted with multiple pressure ulcers. When developing the clients diet plan, the nurse should include:
a. Fresh orange slices
b. Steamed broccoli
c. Ice cream
d. Ground beef patties
88. The nurse prepares to administer a cleansing enema. What is the most common client position used for this procedure?
a. Lithotomy
b. Supine
c. Prone
d. Sims left lateral
89. Nurse Marian is preparing to administer a blood transfusion. Which action should the nurse take first?
a. Arrange for typing and cross matching of the clients blood.
b. Compare the clients identification wristband with the tag on the unit of blood.
c. Start an I.V. infusion of normal saline solution.
d. Measure the clients vital signs.
90. A 65 years old male client requests his medication at 9 p.m. instead of 10 p.m. so that he can go to sleep earlier. Which type of
nursing intervention is required?
a. Independent
b. Dependent
c. Interdependent
d. Intradependent
91. A female client is to be discharged from an acute care facility after treatment for right leg thrombophlebitis. The Nurse Betty
notes that the clients leg is pain-free, without redness or edema. The nurses actions reflect which step of the nursing process?
a. Assessment
b. Diagnosis
c. Implementation
d. Evaluation
92. Nursing care for a female client includes removing elastic stockings once per day. The Nurse Betty is aware that the rationale
for this intervention?
a. To increase blood flow to the heart
b. To observe the lower extremities
c. To allow the leg muscles to stretch and relax
d. To permit veins in the legs to fill with blood.
93. Which nursing intervention takes highest priority when caring for a newly admitted client whos receiving a blood
transfusion?
a. Instructing the client to report any itching, swelling, or dyspnea.
b. Informing the client that the transfusion usually take 1 to 2 hours.
c. Documenting blood administration in the client care record.
d. Assessing the clients vital signs when the transfusion ends.
94. A male client complains of abdominal discomfort and nausea while receiving tube feedings. Which intervention is most
appropriate for this problem?
a. Give the feedings at room temperature.
b. Decrease the rate of feedings and the concentration of the formula.
c. Place the client in semi-Fowlers position while feeding.
d. Change the feeding container every 12 hours.
95. Nurse Patricia is reconstituting a powdered medication in a vial. After adding the solution to the powder, she nurse should:
a. Do nothing.
b. Invert the vial and let it stand for 3 to 5 minutes.
c. Shake the vial vigorously.
d. Roll the vial gently between the palms.
96. Which intervention should the nurse Trish use when administering oxygen by face mask to a female client?
a. Secure the elastic band tightly around the clients head.
b. Assist the client to the semi-Fowler position if possible.
c. Apply the face mask from the clients chin up over the nose.
d. Loosen the connectors between the oxygen equipment and humidifier.
97. The maximum transfusion time for a unit of packed red blood cells (RBCs) is:
a. 6 hours
b. 4 hours
c. 3 hours
d. 2 hours
98. Nurse Monique is monitoring the effectiveness of a clients drug therapy. When should the nurse Monique obtain a blood
sample to measure the trough drug level?
a. 1 hour before administering the next dose.
b. Immediately before administering the next dose.
c. Immediately after administering the next dose.
d. 30 minutes after administering the next dose.
99. Nurse May is aware that the main advantage of using a floor stock system is:
a. The nurse can implement medication orders quickly.
b. The nurse receives input from the pharmacist.
c. The system minimizes transcription errors.
d. The system reinforces accurate calculations.
100. Nurse Oliver is assessing a clients abdomen. Which finding should the nurse report as abnormal?
a. Dullness over the liver.
b. Bowel sounds occurring every 10 seconds.
c. Shifting dullness over the abdomen.
d. Vascular sounds heard over the renal arteries
Answers & Rationale
Gauge your performance by counter checking your answers to the answers below. Learn more about the question by reading the
rationale. If you have any disputes or questions, please direct them to the comments section.
1. Answer: (D) The actions of a reasonably prudent nurse with similar education and experience.
Rationale: The standard of care is determined by the average degree of skill, care, and diligence by nurses in similar circumstances.
2. Answer: (B) I.M
Rationale: With a platelet count of 22,000/l, the clients tends to bleed easily. Therefore, the nurse should avoid using the I.M. route
because the area is a highly vascular and can bleed readily when penetrated by a needle. The bleeding can be difficult to stop.
3. Answer: (C) Digoxin 0.125 mg P.O. once daily
Rationale: The nurse should always place a zero before a decimal point so that no one misreads the figure, which could result in a
dosage error. The nurse should never insert a zero at the end of a dosage that includes a decimal point because this could be
misread, possibly leading to a tenfold increase in the dosage.
4. Answer: (A) Ineffective peripheral tissue perfusion related to venous congestion.
Rationale: Ineffective peripheral tissue perfusion related to venous congestion takes the highest priority because venous
inflammation and clot formation impede blood flow in a client with deep vein thrombosis.
5. Answer: (B) A 44 year-old myocardial infarction (MI) client who is complaining of nausea.
Rationale: Nausea is a symptom of impending myocardial infarction (MI) and should be assessed immediately so that treatment can
be instituted and further damage to the heart is avoided.
6. Answer: (C) Check circulation every 15-30 minutes.
Rationale: Restraints encircle the limbs, which place the client at risk for circulation being restricted to the distal areas of the
extremities. Checking the clients circulation every 15-30 minutes will allow the nurse to adjust the restraints before injury from
decreased blood flow occurs.
7. Answer: (A) Prevent stress ulcer
Rationale: Curlings ulcer occurs as a generalized stress response in burn patients. This results in a decreased production of mucus
and increased secretion of gastric acid. The best treatment for this prophylactic use of antacids and H2 receptor blockers.
8. Answer: (D) Continue to monitor and record hourly urine output
Rationale: Normal urine output for an adult is approximately 1 ml/minute (60 ml/hour). Therefore, this clients output is normal.
Beyond continued evaluation, no nursing action is warranted.
9. Answer: (A) My ankle looks less swollen now.
Rationale: Ice application decreases pain and swelling. Continued or increased pain, redness, and increased warmth are signs of
inflammation that shouldnt occur after ice application
10. Answer: (C) Hypokalemia
Rationale: A loop diuretic removes water and, along with it, sodium and potassium. This may result in hypokalemia, hypovolemia,
and hyponatremia.
11. Answer:(A) Have condescending trust and confidence in their subordinates
Rationale: Benevolent-authoritative managers pretentiously show their trust and confidence to their followers.
12. Answer: (A) Provides continuous, coordinated and comprehensive nursing services.
Rationale: Functional nursing is focused on tasks and activities and not on the care of the patients.
13. Answer: (B) Standard written order
Rationale: This is a standard written order. Prescribers write a single order for medications given only once. A stat order is written
for medications given immediately for an urgent client problem. A standing order, also known as a protocol, establishes guidelines
for treating a particular disease or set of symptoms in special care areas such as the coronary care unit. Facilities also may institute
medication protocols that specifically designate drugs that a nurse may not give.
14. Answer: (D) Liquid or semi-liquid stools
Rationale: Passage of liquid or semi-liquid stools results from seepage of unformed bowel contents around the impacted stool in the
rectum. Clients with fecal impaction dont pass hard, brown, formed stools because the feces cant move past the impaction. These
clients typically report the urge to defecate (although they cant pass stool) and a decreased appetite.
15. Answer: (C) Pulling the helix up and back
Rationale: To perform an otoscopic examination on an adult, the nurse grasps the helix of the ear and pulls it up and back to
straighten the ear canal. For a child, the nurse grasps the helix and pulls it down to straighten the ear canal. Pulling the lobule in any
direction wouldnt straighten the ear canal for visualization.
16. Answer: (A) Protect the irritated skin from sunlight.
Rationale: Irradiated skin is very sensitive and must be protected with clothing or sunblock. The priority approach is the avoidance of
strong sunlight.
17. Answer: (C) Assist the client in removing dentures and nail polish.
Rationale: Dentures, hairpins, and combs must be removed. Nail polish must be removed so that cyanosis can be easily monitored
by observing the nail beds.
18. Answer: (D) Sudden onset of continuous epigastric and back pain.
Rationale: The autodigestion of tissue by the pancreatic enzymes results in pain from inflammation, edema, and possible
hemorrhage. Continuous, unrelieved epigastric or back pain reflects the inflammatory process in the pancreas.
19. Answer: (B) Provide high-protein, high-carbohydrate diet.
Rationale: A positive nitrogen balance is important for meeting metabolic needs, tissue repair, and resistance to infection. Caloric
goals may be as high as 5000 calories per day.
20. Answer: (A) Blood pressure and pulse rate.
Rationale: The baseline must be established to recognize the signs of an anaphylactic or hemolytic reaction to the transfusion.
21. Answer: (D) Immobilize the leg before moving the client.
Rationale: If the nurse suspects a fracture, splinting the area before moving the client is imperative. The nurse should call for
emergency help if the client is not hospitalized and call for a physician for the hospitalized client.
22. Answer: (B) Admit the client into a private room.
Rationale: The client who has a radiation implant is placed in a private room and has a limited number of visitors. This reduces the
exposure of others to the radiation.
23. Answer: (C) Risk for infection
Rationale: Agranulocytosis is characterized by a reduced number of leukocytes (leucopenia) and neutrophils (neutropenia) in the
blood. The client is at high risk for infection because of the decreased body defenses against microorganisms. Deficient knowledge
related to the nature of the disorder may be appropriate diagnosis but is not the priority.
24. Answer: (B) Place the client on the left side in the Trendelenburg position.
Rationale: Lying on the left side may prevent air from flowing into the pulmonary veins. The Trendelenburg position increases
intrathoracic pressure, which decreases the amount of blood pulled into the vena cava during aspiration.
25. Answer: (A) Autocratic.
Rationale: The autocratic style of leadership is a task-oriented and directive.
26. Answer: (D) 2.5 cc
Rationale: 2.5 cc is to be added, because only a 500 cc bag of solution is being medicated instead of a 1 liter.
27. Answer: (A) 50 cc/ hour
Rationale: A rate of 50 cc/hr. The child is to receive 400 cc over a period of 8 hours = 50 cc/hr.
28. Answer: (B) Assess the client for presence of pain.
Rationale: Assessing the client for pain is a very important measure. Postoperative pain is an indication of complication. The nurse
should also assess the client for pain to provide for the clients comfort.
29. Answer: (A) BP 80/60, Pulse 110 irregular
Rationale: The classic signs of cardiogenic shock are low blood pressure, rapid and weak irregular pulse, cold, clammy skin,
decreased urinary output, and cerebral hypoxia.
30. Answer: (A) Take the proper equipment, place the client in a comfortable position, and record the appropriate information in
the clients chart.
Rationale: It is a general or comprehensive statement about the correct procedure, and it includes the basic ideas which are found in
the other options
31. Answer: (B) Evaluation
Rationale: Evaluation includes observing the person, asking questions, and comparing the patients behavioral responses with the
expected outcomes.
32. Answer: (C) History of present illness
Rationale: The history of present illness is the single most important factor in assisting the health professional in arriving at a
diagnosis or determining the persons needs.
33. Answer: (A) Trochanter roll extending from the crest of the ileum to the mid-thigh.
Rationale: A trochanter roll, properly placed, provides resistance to the external rotation of the hip.
34. Answer: (C) Stage III
Rationale: Clinically, a deep crater or without undermining of adjacent tissue is noted.
35. Answer: (A) Second intention healing
Rationale: When wounds dehisce, they will allowed to heal by secondary intention
36. Answer: (D) Tachycardia
Rationale: With an extracellular fluid or plasma volume deficit, compensatory mechanisms stimulate the heart, causing an increase
in heart rate.
37. Answer: (A) 0.75
Rationale: To determine the number of milliliters the client should receive, the nurse uses the fraction method in the following
equation.
75 mg/X ml = 100 mg/1 ml
To solve for X, cross-multiply:
75 mg x 1 ml = X ml x 100 mg
75 = 100X
75/100 = X
0.75 ml (or ml) = X
38. Answer: (D) Its a measure of effect, not a standard measure of weight or quantity.
Rationale: An insulin unit is a measure of effect, not a standard measure of weight or quantity. Different drugs measured in units
may have no relationship to one another in quality or quantity.
39. Answer: (B) 38.9 C
Rationale: To convert Fahrenheit degrees to Centigrade, use this formula
C = (F 32) 1.8
C = (102 32) 1.8
C = 70 1.8
C = 38.9
40. Answer: (C) Failing eyesight, especially close vision.
Rationale: Failing eyesight, especially close vision, is one of the first signs of aging in middle life (ages 46 to 64). More frequent aches
and pains begin in the early late years (ages 65 to 79). Increase in loss of muscle tone occurs in later years (age 80 and older).
41. Answer: (A) Checking and taping all connections
Rationale: Air leaks commonly occur if the system isnt secure. Checking all connections and taping them will prevent air leaks. The
chest drainage system is kept lower to promote drainage not to prevent leaks.
42. Answer: (A) Check the clients identification band.
Rationale: Checking the clients identification band is the safest way to verify a clients identity because the band is assigned on
admission and isnt be removed at any time. (If it is removed, it must be replaced). Asking the clients name or having the client
repeated his name would be appropriate only for a client whos alert, oriented, and able to understand what is being said, but isnt
the safe standard of practice. Names on bed arent always reliable
43. Answer: (B) 32 drops/minute
Rationale: Giving 1,000 ml over 8 hours is the same as giving 125 ml over 1 hour (60 minutes). Find the number of milliliters per
minute as follows:
125/60 minutes = X/1 minute
60X = 125 = 2.1 ml/minute
To find the number of drops per minute:
2.1 ml/X gtt = 1 ml/ 15 gtt
X = 32 gtt/minute, or 32 drops/minute
44. Answer: (A) Clamp the catheter
Rationale: If a central venous catheter becomes disconnected, the nurse should immediately apply a catheter clamp, if available. If a
clamp isnt available, the nurse can place a sterile syringe or catheter plug in the catheter hub. After cleaning the hub with alcohol or
povidone-iodine solution, the nurse must replace the I.V. extension and restart the infusion
45. Answer: (D) Auscultation, percussion, and palpation.
Rationale: The correct order of assessment for examining the abdomen is inspection, auscultation, percussion, and palpation. The
reason for this approach is that the less intrusive techniques should be performed before the more intrusive techniques. Percussion
and palpation can alter natural findings during auscultation.
46. Answer: (D) Ulnar surface of the hand
Rationale: The nurse uses the ulnar surface, or ball, of the hand to asses tactile fremitus, thrills, and vocal vibrations through the
chest wall. The fingertips and finger pads best distinguish texture and shape. The dorsal surface best feels warmth.
47. Answer: (C) Formative
Rationale: Formative (or concurrent) evaluation occurs continuously throughout the teaching and learning process. One benefit is
that the nurse can adjust teaching strategies as necessary to enhance learning. Summative, or retrospective, evaluation occurs at the
conclusion of the teaching and learning session. Informative is not a type of evaluation.
48. Answer: (B) Once per year
Rationale: Yearly mammograms should begin at age 40 and continue for as long as the woman is in good health. If health risks, such
as family history, genetic tendency, or past breast cancer, exist, more frequent examinations may be necessary.
49. Answer: (A) Respiratory acidosis
Rationale: The client has a below-normal (acidic) blood pH value and an above-normal partial pressure of arterial carbon dioxide
(Paco2) value, indicating respiratory acidosis. In respiratory alkalosis, the pH value is above normal and in the Paco2 value is below
normal. In metabolic acidosis, the pH and bicarbonate (Hco3) values are below normal. In metabolic alkalosis, the pH and Hco3
values are above normal.
50. Answer: (B) To provide support for the client and family in coping with terminal illness.
Rationale: Hospices provide supportive care for terminally ill clients and their families. Hospice care doesnt focus on counseling
regarding health care costs. Most client referred to hospices have been treated for their disease without success and will receive
only palliative care in the hospice.
51. Answer: (C) Using normal saline solution to clean the ulcer and applying a protective dressing as necessary.
Rationale: Washing the area with normal saline solution and applying a protective dressing are within the nurses realm of
interventions and will protect the area. Using a povidone-iodine wash and an antibiotic cream require a physicians order. Massaging
with an astringent can further damage the skin.
52. Answer: (D) Foot
Rationale: An elastic bandage should be applied form the distal area to the proximal area. This method promotes venous return. In
this case, the nurse should begin applying the bandage at the clients foot. Beginning at the ankle, lower thigh, or knee does not
promote venous return.
53. Answer: (B) Hypokalemia
Rationale: Insulin administration causes glucose and potassium to move into the cells, causing hypokalemia.
54. Answer: (A) Throbbing headache or dizziness
Rationale: Headache and dizziness often occur when nitroglycerin is taken at the beginning of therapy. However, the client usually
develops tolerance
55. Answer: (D) Check the clients level of consciousness
Rationale: Determining unresponsiveness is the first step assessment action to take. When a client is in ventricular tachycardia,
there is a significant decrease in cardiac output. However, checking the unresponsiveness ensures whether the client is affected by
the decreased cardiac output.
56. Answer: (B) On the affected side of the client.
Rationale: When walking with clients, the nurse should stand on the affected side and grasp the security belt in the mid spine area of
the small of the back. The nurse should position the free hand at the shoulder area so that the client can be pulled toward the nurse
in the event that there is a forward fall. The client is instructed to look up and outward rather than at his or her feet.
57. Answer: (A) Urine output: 45 ml/hr
Rationale: Adequate perfusion must be maintained to all vital organs in order for the client to remain visible as an organ donor. A
urine output of 45 ml per hour indicates adequate renal perfusion. Low blood pressure and delayed capillary refill time are
circulatory system indicators of inadequate perfusion. A serum pH of 7.32 is acidotic, which adversely affects all body tissues.
58. Answer: (D ) Obtaining the specimen from the urinary drainage bag.
Rationale: A urine specimen is not taken from the urinary drainage bag. Urine undergoes chemical changes while sitting in the bag
and does not necessarily reflect the current client status. In addition, it may become contaminated with bacteria from opening the
system.
59. Answer: (B) Cover the client, place the call light within reach, and answer the phone call.
Rationale: Because telephone call is an emergency, the nurse may need to answer it. The other appropriate action is to ask another
nurse to accept the call. However, is not one of the options. To maintain privacy and safety, the nurse covers the client and places
the call light within the clients reach. Additionally, the clients door should be closed or the room curtains pulled around the bathing
area.
60. Answer: (C) Use a sterile plastic container for obtaining the specimen.
Rationale: Sputum specimens for culture and sensitivity testing need to be obtained using sterile techniques because the test is done
to determine the presence of organisms. If the procedure for obtaining the specimen is not sterile, then the specimen is not sterile,
then the specimen would be contaminated and the results of the test would be invalid.
61. Answer: (A) Puts all the four points of the walker flat on the floor, puts weight on the hand pieces, and then walks into it.
Rationale: When the client uses a walker, the nurse stands adjacent to the affected side. The client is instructed to put all four points
of the walker 2 feet forward flat on the floor before putting weight on hand pieces. This will ensure client safety and prevent stress
cracks in the walker. The client is then instructed to move the walker forward and walk into it.
62. Answer: (C) Draws one line to cross out the incorrect information and then initials the change.
Rationale: To correct an error documented in a medical record, the nurse draws one line through the incorrect information and then
initials the error. An error is never erased and correction fluid is never used in the medical record.
63. Answer: (C) Secures the client safety belts after transferring to the stretcher.
Rationale: During the transfer of the client after the surgical procedure is complete, the nurse should avoid exposure of the client
because of the risk for potential heat loss. Hurried movements and rapid changes in the position should be avoided because these
predispose the client to hypotension. At the time of the transfer from the surgery table to the stretcher, the client is still affected by
the effects of the anesthesia; therefore, the client should not move self. Safety belts can prevent the client from falling off the
stretcher.
64. Answer: (B) Gown and gloves
Rationale: Contact precautions require the use of gloves and a gown if direct client contact is anticipated. Goggles are not necessary
unless the nurse anticipates the splashes of blood, body fluids, secretions, or excretions may occur. Shoe protectors are not
necessary.
65. Answer: (C) Quad cane
Rationale: Crutches and a walker can be difficult to maneuver for a client with weakness on one side. A cane is better suited for
client with weakness of the arm and leg on one side. However, the quad cane would provide the most stability because of the
structure of the cane and because a quad cane has four legs.
66. Answer: (D) Left side-lying with the head of the bed elevated 45 degrees.
Rationale: To facilitate removal of fluid from the chest wall, the client is positioned sitting at the edge of the bed leaning over the
bedside table with the feet supported on a stool. If the client is unable to sit up, the client is positioned lying in bed on the
unaffected side with the head of the bed elevated 30 to 45 degrees.
67. Answer: (D) Reliability
Rationale: Reliability is consistency of the research instrument. It refers to the repeatability of the instrument in extracting the same
responses upon its repeated administration.
68. Answer: (A) Keep the identities of the subject secret
Rationale: Keeping the identities of the research subject secret will ensure anonymity because this will hinder providing link between
the information given to whoever is its source.
69. Answer: (A) Descriptive- correlational
Rationale: Descriptive- correlational study is the most appropriate for this study because it studies the variables that could be the
antecedents of the increased incidence of nosocomial infection.
70. Answer: (C) Use of laboratory data
Rationale: Incidence of nosocomial infection is best collected through the use of biophysiologic measures, particularly in vitro
measurements, hence laboratory data is essential.
71. Answer: (B) Quasi-experiment
Rationale: Quasi-experiment is done when randomization and control of the variables are not possible.
72. Answer: (C) Primary source
Rationale: This refers to a primary source which is a direct account of the investigation done by the investigator. In contrast to this is
a secondary source, which is written by someone other than the original researcher.
73. Answer: (A) Non-maleficence
Rationale: Non-maleficence means do not cause harm or do any action that will cause any harm to the patient/client. To do good is
referred as beneficence.
74. Answer: (C) Res ipsa loquitur
Rationale: Res ipsa loquitur literally means the thing speaks for itself. This means in operational terms that the injury caused is the
proof that there was a negligent act.
75. Answer: (B) The Board can investigate violations of the nursing law and code of ethics
Rationale: Quasi-judicial power means that the Board of Nursing has the authority to investigate violations of the nursing law and
can issue summons, subpoena or subpoena duces tecum as needed.
76. Answer: (C) May apply for re-issuance of his/her license based on certain conditions stipulated in RA 9173
Rationale: RA 9173 sec. 24 states that for equity and justice, a revoked license maybe re-issued provided that the following
conditions are met: a) the cause for revocation of license has already been corrected or removed; and, b) at least four years has
elapsed since the license has been revoked.
77. Answer: (B) Review related literature
Rationale: After formulating and delimiting the research problem, the researcher conducts a review of related literature to
determine the extent of what has been done on the study by previous researchers.
78. Answer: (B) Hawthorne effect
Rationale: Hawthorne effect is based on the study of Elton Mayo and company about the effect of an intervention done to improve
the working conditions of the workers on their productivity. It resulted to an increased productivity but not due to the intervention
but due to the psychological effects of being observed. They performed differently because they were under observation.
79. Answer: (B) Determines the different nationality of patients frequently admitted and decides to get representations samples
from each.
Rationale: Judgment sampling involves including samples according to the knowledge of the investigator about the participants in
the study.
80. Answer: (B) Madeleine Leininger
Rationale: Madeleine Leininger developed the theory on transcultural theory based on her observations on the behavior of selected
people within a culture.
81. Answer: (A) Random
Rationale: Random sampling gives equal chance for all the elements in the population to be picked as part of the sample.
82. Answer: (A) Degree of agreement and disagreement
Rationale: Likert scale is a 5-point summated scale used to determine the degree of agreement or disagreement of the respondents
to a statement in a study
83. Answer: (B) Sr. Callista Roy
Rationale: Sr. Callista Roy developed the Adaptation Model which involves the physiologic mode, self-concept mode, role function
mode and dependence mode.
84. Answer: (A) Span of control
Rationale: Span of control refers to the number of workers who report directly to a manager.
85. Answer: (B) Autonomy
Rationale: Informed consent means that the patient fully understands about the surgery, including the risks involved and the
alternative solutions. In giving consent it is done with full knowledge and is given freely. The action of allowing the patient to decide
whether a surgery is to be done or not exemplifies the bioethical principle of autonomy.
86. Answer: (C) Avoid wearing canvas shoes.
Rationale: The client should be instructed to avoid wearing canvas shoes. Canvas shoes cause the feet to perspire, which may, in
turn, cause skin irritation and breakdown. Both cotton and cornstarch absorb perspiration. The client should be instructed to cut
toenails straight across with nail clippers.
87. Answer: (D) Ground beef patties
Rationale: Meat is an excellent source of complete protein, which this client needs to repair the tissue breakdown caused by
pressure ulcers. Oranges and broccoli supply vitamin C but not protein. Ice cream supplies only some incomplete protein, making it
less helpful in tissue repair.
88. Answer: (D) Sims left lateral
Rationale: The Sims left lateral position is the most common position used to administer a cleansing enema because it allows gravity
to aid the flow of fluid along the curve of the sigmoid colon. If the client cant assume this position nor has poor sphincter control,
the dorsal recumbent or right lateral position may be used. The supine and prone positions are inappropriate and uncomfortable for
the client.
89. Answer: (A) Arrange for typing and cross matching of the clients blood.
Rationale: The nurse first arranges for typing and cross matching of the clients blood to ensure compatibility with donor blood. The
other options, although appropriate when preparing to administer a blood transfusion, come later.
90. Answer: (A) Independent
Rationale: Nursing interventions are classified as independent, interdependent, or dependent. Altering the drug schedule to coincide
with the clients daily routine represents an independent intervention, whereas consulting with the physician and pharmacist to
change a clients medication because of adverse reactions represents an interdependent intervention. Administering an already-
prescribed drug on time is a dependent intervention. An intradependent nursing intervention doesnt exist.
91. Answer: (D) Evaluation
Rationale: The nursing actions described constitute evaluation of the expected outcomes. The findings show that the expected
outcomes have been achieved. Assessment consists of the clients history, physical examination, and laboratory studies. Analysis
consists of considering assessment information to derive the appropriate nursing diagnosis. Implementation is the phase of the
nursing process where the nurse puts the plan of care into action.
92. Answer: (B) To observe the lower extremities
Rationale: Elastic stockings are used to promote venous return. The nurse needs to remove them once per day to observe the
condition of the skin underneath the stockings. Applying the stockings increases blood flow to the heart. When the stockings are in
place, the leg muscles can still stretch and relax, and the veins can fill with blood.
93. Answer:(A) Instructing the client to report any itching, swelling, or dyspnea.
Rationale: Because administration of blood or blood products may cause serious adverse effects such as allergic reactions, the nurse
must monitor the client for these effects. Signs and symptoms of life-threatening allergic reactions include itching, swelling, and
dyspnea. Although the nurse should inform the client of the duration of the transfusion and should document its administration,
these actions are less critical to the clients immediate health. The nurse should assess vital signs at least hourly during the
transfusion.
94. Answer: (B) Decrease the rate of feedings and the concentration of the formula.
Rationale: Complaints of abdominal discomfort and nausea are common in clients receiving tube feedings. Decreasing the rate of the
feeding and the concentration of the formula should decrease the clients discomfort. Feedings are normally given at room
temperature to minimize abdominal cramping. To prevent aspiration during feeding, the head of the clients bed should be elevated
at least 30 degrees. Also, to prevent bacterial growth, feeding containers should be routinely changed every 8 to 12 hours.
95. Answer: (D) Roll the vial gently between the palms.
Rationale: Rolling the vial gently between the palms produces heat, which helps dissolve the medication. Doing nothing or inverting
the vial wouldnt help dissolve the medication. Shaking the vial vigorously could cause the medication to break down, altering its
action.
96. Answer: (B) Assist the client to the semi-Fowler position if possible.
Rationale: By assisting the client to the semi-Fowler position, the nurse promotes easier chest expansion, breathing, and oxygen
intake. The nurse should secure the elastic band so that the face mask fits comfortably and snugly rather than tightly, which could
lead to irritation. The nurse should apply the face mask from the clients nose down to the chin not vice versa. The nurse should
check the connectors between the oxygen equipment and humidifier to ensure that theyre airtight; loosened connectors can cause
loss of oxygen.
97. Answer: (B) 4 hours
Rationale: A unit of packed RBCs may be given over a period of between 1 and 4 hours. It shouldnt infuse for longer than 4 hours
because the risk of contamination and sepsis increases after that time. Discard or return to the blood bank any blood not given
within this time, according to facility policy.
98. Answer: (B) Immediately before administering the next dose.
Rationale: Measuring the blood drug concentration helps determine whether the dosing has achieved the therapeutic goal. For
measurement of the trough, or lowest, blood level of a drug, the nurse draws a blood sample immediately before administering the
next dose. Depending on the drugs duration of action and half-life, peak blood drug levels typically are drawn after administering
the next dose.
99. Answer: (A) The nurse can implement medication orders quickly.
Rationale: A floor stock system enables the nurse to implement medication orders quickly. It doesnt allow for pharmacist input, nor
does it minimize transcription errors or reinforce accurate calculations.
100. Answer: (C) Shifting dullness over the abdomen.
Rationale: Shifting dullness over the abdomen indicates ascites, an abnormal finding. The other options are normal abdominal
findings.
Fundamentals of Nursing Exam 15: NLE Pre-Boards (100 Items)

Introduction
This a 100-item examination covering the concepts of Fundamentals of Nursing and some questions about the laws, organizations
and processes of nursing in the Philippines. If you are taking the Philippine Nurse Licensure Examination (PNLE) then this exam is a
great review.
Topics
Topics or concepts included in this exam are:
Guidelines
Questions
InText Mode: All questions and answers are given for reading and answering at your own pace. You can also copy this exam and
make a print out.
SITUATION: A Nurse utilizes the nursing process in managing patient care. Knowledge of this process is essential to deliver high
quality care and to focus on the clients response to their illness.
1. During the planning phase of the nursing process, which of the following is the product developed?
A. Nursing care plan
B. Nursing diagnosis
C. Nursing history
D. Nursing notes
2. Objective data are also known as?
A. Covert data
B. Inferences
C. Overt data
D. Symptoms
3. Data or information obtained from the assessment of a patient is primarily used by nurse to:
A. Ascertain the patients response to health problems
B. Assist in constructing the taxonomy of nursing intervention
C. Determine the effectiveness of the doctors order
D. Identify the patients disease process
4. What is an example of a subjective data?
A. Color of wound drainage
B. Odor of breath
C. Respiration of 14 breaths/minute
D. The patients statement of I feel sick to my stomach
SITUATION : Correct application of the Nursing Process is vital in providing quality care. The nurse must use her skills and knowledge
in proper assessment, planning and evaluating to meet the patients need and address the priority response of the client to his or
her illness.
5. Which statement is a difference between comprehensive and focused assessment?
A. Comprehensive assessments cant include any focus assessments
B. Focused assessments are more important than comprehensive assessments
C. Focused assessments are usually ongoing and concerning specific problems
D. Objective data are included only in comprehensive assessments
6. Two-year-old Bens mother states Ben vomited 8 ounces of his formula this morning. This statement is an example of:
A. Objective data from a primary source
B. Objective data from a secondary source
C. Subjective data from a primary source
D. Subjective data from a secondary source
7. Which expected outcome is correctly written?
A. The patient will be less edematous in 24 hours
B. The patient will drink an adequate amount of fluid daily
C. The patient will identify 5 high-salt foods from prepared list by discharge
D. The patient will soon sleep well through the night
8. An expected outcome on a patients care plan reads: Patient will state seven warning signs of cancer by discharge. When the
nurse evaluates the patient progress, the patient is able to state that a change in wart or mole, a sore that doesnt heal and a
change in bowel or bladder habits are warning signals of cancer. Which of the following would be an appropriate evaluative
statement for the nurse to place on the patients nursing care plan?
A. Patient understands the warning signals of cancer
B. Goal met; Patient cited a change in wart of mole, sore that doesnt heal and a change in bowel or bladder habits as warning
signals of cancer.
C. Goal not met
D. Goal partially met
9. A quality assurance nurse sends questionnaire to patients after discharge to determine their level of satisfaction with the nurse
care they received in the facility. What type of nursing audit is this?
A. Concurrent
B. Outcome
C. Terminal
D. Retrospective
10. The nurse makes the following entry in the patients record: Goal not met; patient refuses to attend smoking cessation
classes. Because this goal hasnt been met, the nurse should:
A. Develop a completely new nursing care plan
B. Assign the patient to a more experienced nurse
C. Critique the steps involved in the development of the goal
D. Transfer the patient to another facility
SITUATION : Health Care Delivery System affects the health status of every Filipino. As a nurse knowledgeable of this system, she is
expected to ensure quality of life.
11. When should rehabilitation commence?
A. The day before discharge
B. When the patient desires
C. Upon admission
D. 24 hours after discharge
12. What exemplified the preventive and promotive programs in the hospital?
A. Hospital as a center to prevent and control infection
B. Program for smokers
C. Program for alcoholics and drug addicts
D. Hospital Wellness Center
13. Which makes nursing dynamic?
A. Every patient is a unique physical, emotional, social and spiritual being
B. The patient participate in the overall nursing care plan
C. Nursing practice is expanding in the light of modern developments that takes place
D. The health status of the patient is constantly changing and the nurse must be cognizant and responsive to these changes
14. Prevention is an important responsibility of the nurse in:
A. Hospitals
B. Community
C. Workplace
D. All of the above
15. This form of Health Insurance provides comprehensive prepaid health services to enrollees for a fixed periodic payment.
A. Health Maintenance Organization
B. Medicare
C. Philippine Health Insurance Act
D. Hospital Maintenance Organization
SITUATION : Maslows hierarchy of needs is an integral component of prioritization. As a nurse, you must know the clients needs
that request for an immediate action.
16. The client with history of pulmonary emboli is scheduled for insertion of an inferior vena cava filter. The nurse checks on the
client 1 hour after the physician has explained the procedure and obtained consent from the client. The client is lying in bed,
wringing the hands, and says to the nurse, Im not sure about this. What if it doesnt work, and Im just as bad off as before?
The nurse addresses which of the following primary concerns of the client?
A. Fear related to the potential risk and outcome of surgery
B. Anxiety related to the fear of death
C. Ineffective individual coping related to the therapeutic regimen
D. Knowledge deficit related to the surgical procedure
17. A nurse is formulating a plan of care for a client receiving enteral feedings. The nurse identifies which nursing diagnosis as the
highest priority for this client?
A. Altered nutrition, less than body requirements
B. High risk for aspiration
C. High risk for fluid volume deficit
D. Diarrhea
18. A home care nurse finds a client in the bedroom, unconscious, with pill bottle in hand. The pill bottle contained the SSRI
Sertraline (Zoloft). The nurse immediately assesses the clients:
A. Blood pressure
B. Respirations
C. Pulse
D. Urine Output
19. A nurse is caring for a client admitted to the hospital for subclavian line placement. Which psychosocial area of assessment
should the nurse address with the client?
A. Strict restrictions of neck mobility
B. Loss of ability to ambulate as tolerated
C. Possible body image disturbance
D. Continuous pain related to ongoing placement of the subclavian line
20. A hospitalized client who has a living will is being fed through a nasogastric tube. During a bolus feeding, the client vomits and
begins choking. Which of the following actions is most appropriate for the nurse to take?
A. Clear the clients airway
B. Make the client comfortable
C. Start CPR
D. Stop feeding and remove the NG tube
SITUATION: Health Promotion is the priority nursing responsibility. The nurse should be able to promote the clients well being and
identify ways on improving the clients quality of life.
21. The nurse is caring for a 16 year old female client who isnt sexually active. The client asks if she needs a Papanicolaou test.
The nurse should reply:
A. Yes, she should have a Pap test after the onset of menstruation
B. No, because she isnt sexually active
C. Yes, because shes 16 years old
D. No, because she is not yet 21 years old
22. The nurse is assessing a client who complains of abdominal pain, nausea and diarrhea. When examining the clients abdomen,
which sequence should the nurse use?
A. Inspection, palpation, percussion, auscultation
B. Inspection, auscultation, percussion, palpation
C. Auscultation, inspection, percussion, palpation
D. Palpation, auscultation, percussion, inspection
23. The nurse is speaking to a group of women about early detection of breast cancer. The average age of the women in the group
is 45. Following the Philippine Cancer Society guidelines, the nurse should recommend that the women:
A. Perform breast self examination annually
B. Have a mammogram annually
C. Have a hormonal receptor assay annually
D. Have a physician conduct a clinical examination every 2 years
24. The school nurse is planning a program for a group of teenagers on skin cancer prevention. Which of the following
instruction should the nurse emphasize in her talk?
A. Stay out of the sun between 1 p.m and 3 p.m
B. Tanning booth are a safe alternative for those who wish to tan
C. Sun exposure is safe, provided the client wears protective clothing
D. Examine skin once per month, looking for suspicious lesions or changes in moles
25. An employer establishes a physical exercise area in the workplace and encourages all employees to use it. This is an example
of which level of prevention?
A. Primary prevention
B. Secondary prevention
C. Tertiary prevention
D. Passive prevention
SITUATION : Basic nursing skills are essential for they are vital in many nursing procedures. Such skills are needed in order to
promote health, prevent illness, cure a disease and rehabilitate infirmities.
26. A nurse has just been told by the physician that an order has been written to administer an iron injection to a client. The nurse
plans to give the medication in the:
A. Gluteal muscle using Z-Track technique
B. Deltoid muscle using an air lock
C. Subcutaneous tissue of the abdomen
D. Anterolateral thigh using 5/8-inch needle
27. A client has just been told by the physician that a cerebral angiogram will be obtained. The nurse then collects data from the
client about which of the following pieces of information?
A. Claustrophobia
B. Excessive weight
C. Allergy to eggs
D. Allergy to iodine or shellfish
28. A client has an order for a wound culture to be performed with the next wound irrigation and dressing change. The nurse
would plan to use which of the following solutions for irrigation before this particular procedure?
A. Povidone-iodine (Betadine)
B. One-half-strength hydrogen peroxide
C. Normal saline
D. Acetic acid
29. Which of the following is the best indicator in determining that the clients airway needs suctioning?
A. Oxygen saturation measurement
B. Respiratory rate
C. Breath sounds
D. Arterial blood gas results
30. A nurse is planning to assist a diabetic client to perform self-monitoring of blood glucose levels. The nurse should incorporate
which of the following strategies to best help the client obtain an adequate capillary sample?
A. Cleanse the hands beforehand using cool water
B. Let the arm hang dependently and milk the digit
C. Puncture the center of the finger pad
D. Puncture the finger as deeply as possible
SITUATION : Mastery of Intravenous therapy and all aspects that address the response of the client to complication related to it will
help the new nurse in providing quality care.
31. One hour after the IV was inserted, Nurse Net found out that the 1 liter of D5NSS was empty. Patient was in severe
respiratory distress with pinkish frothy sputum. The most probable complication is:
A. Speed shock
B. Renal failure
C. Congestive heart failure
D. Pulmonary edema
32. When an IV of D5W is not infusing well on patient, it is best to first:
A. Pinch the rubber part to flush out clogged blood
B. Coil tube and squeeze hard
C. Lower IV to check for return flow
D. Restart the IV
33. The doctor ordered venoclysis of dextrose 5% in water one liter KVO for 24 hours as a vehicle for IV medications. How many
drops per minute should the fluid be regulated?
A. 20 drops per minute
B. 5 drops per minute
C. 10 drops per minute
D. 15 drops per minute
34. The nurse is administering I.V fluids to an infant. Infants receiving I.V Therapy are particularly vulnerable to:
A. Hypotension
B. Fluid overload
C. Cardiac arrhythmias
D. Pulmonary emboli
35. A client with severe inflammatory bowel disease is receiving TPN. When administering TPN, the nurse must take care to
maintain the prescribed flow rate because giving TPN too rapidly may cause:
A. Hyperglycemia
B. Air embolism
C. Constipation
D. Dumping syndrome
SITUATION : Teaching and learning is a nursing responsibility. The ability of a nurse to educate and change the clients behavior
through health teaching is one important goal of nursing.
36. A clients hypertension has been recently diagnosed. The nurse would plan to do which of the following as the first step in
teaching the client about the disorder?
A. Gather all available resource material
B. Plan for the evaluation of the session
C. Assess the clients knowledge and needs
D. Decide on the teaching approach
37. A diabetic client who is performing self-monitoring of blood glucose at home asks a clinic nurse why a glycosylated
hemoglobin level needs to be measured. The nurse should plan to incorporate which of the following into a response?
A. This laboratory test is done yearly to predict likelihood of long term complication
B. This laboratory test gives an indication of glycemic control over the last 3 months
C. It is done as a method of verifying the accuracy of the meter used at Home
D. It is done to predict risk of hypoglycemia with the current diet and medication regimen
38. In teaching the sister of a diabetic client about the proper use of a glucometer in determining the blood sugar level of the
client, The nurse is focusing in which domain of learning according to bloom?
A. Cognitive
B. Affective
C. Psychomotor
D. Affiliative
39. Which of the following is the most important condition for diabetic client to learn how to control their diet?
A. Use of pamphlets and effective teaching devices during health instruction
B. Motivation to be symptom free
C. Ability of the client to understand teaching instruction
D. Language and appropriateness of the instruction
40. When you prepare your teaching plan for a group of hypertensive clients, you first formulate your learning objectives. Which
of the following steps in the nursing process corresponds to the writing of learning objectives?
A. Planning
B. Implementing
C. Evaluating
D. Assessing
SITUATION : Nursing is a science. It involves a wide spectrum of theoretical foundation applied in different health care situation. The
nurse must use these theories in order to deliver the most needed quality care.
41. The theorist who believes that adaptation and manipulation of stressors are needed to foster change is:
A. Betty Neuman
B. Dorothea Orem
C. Martha Rogers
D. Sister Callista Roy
42. The theorist whose theory can be defined as the development of a science of humankind, incorporating the concepts of
energy fields, openness, pattern and organization is:
A. Dorothy Johnson
B. Hildegard Peplau
C. Martha Rogers
D. Myra Levine
43. A theorist whose major theme is the idea of transcultural nursing and caring nursing is:
A. Dorothea Orem
B. Madeleine Leininger
C. Sister Callista Roy
D. Virginia Henderson
44. Florence Nightingale was born in
A. Italy, May 12, 1840
B. Italy, May 12, 1820
C. England, May 12, 1840
D. England, May 12, 1820
45. Smith conceptualizes this health model as a condition of actualization or realization of persons potential. Avers that the
highest aspiration of people is fulfillment and complete developmental actualization.
A. Clinical Model
B. Role performance Model
C. Adaptive Model
D. Eudaemonistic Model
SITUATION : Oxygen is the most important physiologic requirement of the body. Absence of this vital element for over 6 minutes
leads to irreversible brain damage. Measures that promotes oxygenation is integral in successfully managing clients response to
illnesses.
46. The primary effect of oxygen therapy is to:
A. Increase oxygen in the tissues and cells
B. Increase oxygen carrying capacity of the blood
C. Increase respiratory rate
D. Increase oxygen pressure in the alveolar sac
47. A nurse suctions a client who has an endotracheal tube in place. Following the procedure, which of the following would
indicate to the nurse that the client is experiencing an adverse effect of this procedure?
A. Hypertension
B. Cardiac Irregularities
C. A reddish coloration in the clients face
D. Oxygen saturation level of 95%
48. The GAUGE size in ET tubes determines:
A. The external circumference of the tube
B. The internal diameter of the tube
C. The length of the tube
D. The tubes volumetric capacity
49. The nurse is correct in performing suctioning when she applies the suction intermittently during:
A. Insertion of the suction catheter
B. Withdrawing of the suction catheter
C. both insertion and withdrawing of the suction catheter
D. When the suction catheter tip reaches the bifurcation of the trachea
50. The purpose of the cuff in Tracheostomy tube is to:
A. Separate the upper and lower airway
B. Separate trachea from the esophagus
C. Separate the larynx from the nasopharynx
D. Secure the placement of the tube
SITUATION : To deliver a safe and quality care, Knowledge about wound care is necessary. The nurse will have to deal with different
types of wound during practice. It is of outmost important to apply this knowledge to ensure optimum wound healing.
51. Based on the nurses knowledge of surgical wounds, simple surgical incisions heal by:
A. Primary intention
B. Secondary intention
C. Tertiary intention
D. Quarternary intention
52. The nurse documents that the wound edges are approximated. When the edges of an incision are said to be approximated,
this means edges are:
A. Brought together by sutures, tapes or staples
B. Eythematous and swollen
C. Gaping and draining
D. Necrotic and draining
53. Which vitamin is most essential for collagen synthesis?
A. Vitamin A
B. Vitamin B
C. Vitamin C
D. Vitamin D
54. When assessing the clients wound for sign of infection, the nurse should look for the presence of which of the following?
A. Granulation tissue
B. Pink tissue
C. Purulent drainage
D. Well approximated edges
55. The nurse is changing dressing and providing wound care. Which activity should she perform first?
A. Assess the drainage in the dressing
B. Slowly remove the soiled dressing
C. Wash hands thoroughly
D. Put on latex gloves
SITUATION : Physical examination and health assessment are important in rendering care. The nurse must use assessment
knowledge in order to determine and prioritize clients response to his or her illness.
56. The component that should receive the highest priority before physical examination is the:
A. Psychological preparation of the client
B. Physical Preparation of the client
C. Preparation of the Environment
D. Preparation of the Equipments
57. When inspecting a clients skin, the nurse finds a vesicle on the clients arm. Which description applies to a vesicle?
A. A flat, nonpalpable, and colored
B. Solid, elevated, and circumscribed
C. Circumscribed, elevated, and filled with serous fluid
D. Elevated, pus-filled, and circumscribed
58. When assessing a client with abdominal pain, the nurse should assess:
A. Any quadrant first
B. The symptomatic quadrant first
C. The symptomatic quadrant last
D. The sympomatic quadrant either second or third
59. To assess the clients dorsalis pedis pulse, the nurse should palpate the:
A. Medial surface of the ankle
B. Lateral surface of the ankle
C. Ventral aspect of the top of the foot
D. Medial aspect of the dorsum of the foot
60. Which of the following assessment would be a priority for a 2 year old after bronchoscopy?
A. Cardiac rate
B. Respiratory quality
C. Sputum color
D. Pulse pressure changes
61. The nurse checks the clients gag reflex. The recommended technique for testing the gag reflex is to:
A. touch the back of the clients throat with a tongue depressor
B. observe the client for evidence of spontaneous swallowing when the neck is stroked
C. place a few milliliters of water on the clients tongue and note whether or not he swallows
D. observe the clients response to the introduction of a catheter for endotracheal suctioning
62. The nurse is evaluating a clients lung sounds. Which of the following breath sounds indicate adequate ventilation when
auscultated over the lung fields?
A. Vesicular
B. Bronchial
C. Bronchovesicular
D. Adventitious
63. The night nurse informs the primary nurse that a client receiving intermittent gavage feedings is not tolerating them. The
primary nurse should first:
A. Change the feeding schedule to omit nights
B. Request that the type of solution be changed
C. Observe the night nurse administering a feeding
D. Suggest that the prescribed antiemetic be given first
64. A client has a chest tubes attached to a pleural drainage system. When caring for this client, the nurse should:
A. Palpate the surrounding are for crepitus
B. Clamp the chest tubes when suctioning the client
C. Change the dressing daily using aseptic technique
D. Empty drainage chamber at the end of the shift
65. The nurse, aware of a clients 25 year history of excessive alcohol use, would expect the physical assessment to reveal a:
A. Liver infection
B. Low blood ammonia
C. Small liver with a rough surface
D. High fever with a generalizedrash
SITUATION : Nursing is a profession. Miss Linda is a newly appointed nurse in a hospital in Manila. Born May 1985, Her knowledge of
nursings professional development is required in order to project the profession in a way that it lifts the standards of nursing.
66. Mrs. Linda defined nursing as one of the following except:
A. Assisting individual, family and community in attaining health
B. Assisting basic health needs
C. Establishing nursing diagnosis and implementing nursing care
D. Diagnosing, treating, prescribing medication and doing minor surgery
67. Philippine Nurses Association (PNA) was established in:
1. 1922
2. 1926
3. With Mrs. Rosario Delgado as first president
4. With Mrs. Anastacia Tupas as first president
A. 1, 3
B. 1, 4
C. 2, 3
D. 2, 4
68. As a national nurses association, it is characterized as follows except:
A. Both a professional body and a labor union
B. Affiliated with the International Council Of Nurses
C. Advocating for improved work and life condition of Nurses
D. Accrediting body for continuing education program
69. CPE Units per year is needed for license renewal. The number of CPE units per year should be:
A. 20 units
B. 30 units
C. 60 units
D. 100 units
70. As a nurse, R.A 1080 exempts her from:
A. Paying her professional tax
B. Examination for civil service eligibility
C. Qualifying for the CGFNS
D. Paying business permit
71. In resigning for her job as a staff nurse, she must give advance notice of:
A. 15 days
B. 30 days
C. 45 days
D. 60 days
72. Why is there an ethical dilemma?
A. Because the law do not clearly state what is right from what is wrong
B. Because morality is subjective and it differs from each individual
C. Because the patients right coincide with the nurses responsibility
D. Because the nurse lacks ethical knowledge to determine what action is correct and what action is unethical
73. Who among the following can work as a practicing nurse in the Philippines without taking the Licensure examination?
A. Internationally well known experts which services are for a fee
B. Those that are hired by local hospitals in the country
C. Expert nurse clinicians hired by prestigious hospitals
D. Those involved in medical mission whos services are for free
74. The nurse is correct in determining that a License is:
A. A personal property
B. Can be revoked by the Board of Nursing
C. A Right
D. Can be revoked by the PNA
75. A License is renewed every:
A. 1 year
B. 2 years
C. 3 years
D. 4 years
76. Which of the following persons cannot have free access to a patients record?
A. The patient
B. The physical therapist
C. The pharmacist
D. The lawyer
77. Ms. Lindas license will expire in 2007, She must renew her license when?
A. January 2007
B. December 2007
C. May 2007
D. May 2008
78. The practice of Nursing in the Philippines is regulated by:
A. RA 9163
B. RA 9173
C. RA 7164
D. RA 7146
79. This quality is being demonstrated by a Nurse who raise the side rails of a confuse and disoriented patient?
A. Autonomy
B. Responsibility
C. Prudence
D. Resourcefulness
80. Nurse Joel and Ana is helping a 16 year old Nursing Student in a case filed against the student. The case was frustrated
homicide. Nurse Joel and Ana are aware of the different circumstances of crimes. They are correct in identifying which of the
following Circumstances that will be best applied in this case?
A. Justifying
B. Aggravating
C. Mitigating
D. Exempting
SITUATION : This is the first day of Mark, R.N. to report as a staff nurse in a tertiary hospital. As a morning duty nurse, she is about to
chart her nursing care.
81. Which of the following is not an accepted medical abbreviation?
A. NPO
B. PRN
C. OD
D. NON
82. Communication is best undertaken if barriers are first removed. Considering this statement, which of the following is
considered as deterrent factor in communication?
A. Not universally accepted abbreviation
B. Wrong Grammar
C. Poor Penmanship
D. Old age of the client
83. Which of the following chart entries are not acceptable?
A. Patient complained of chest pain
B. Patient ambulated to B/R
C. Vital signs 130/70; 84; 20
D. Pain relieved by Nitrogylcerine gr 1/150 sublingually
84. Which of the following indicates narrative charting?
A. Written descriptive nurses notes
B. Date recorded on nurse activity sheet
C. Use of checklist
D. Use of flowsheet
85. Being a new staff nurse, Mark remembers that information needed for daily nursing care of clients are readily available in
which of the following?
A. Kardex
B. Order sheet
C. Admission notes
D. Nurses notes
SITUATION : Mr. Orlando is assigned to Mang Carlos, A 60 years old, newly diagnosed diabetes patient. He is beginning to write
objectives for his teaching plan.
86. Which of the following is written in behavioral term?
A. Mang Carlos will know about diabetes related foot care and the techniques and equipment necessary to carry it out
B. Mang Carlos sister will be able to determine his insulin requirement based on blood glucose levels obtained from glucometer in
two days
C. Mang Carlos daugter should learn about diabetes milletus within the week
D. Mang Carlos wife needs to understand the side effects of insulin
87. Which of the following is the best rationale for written objectives?
A. Document the quality of care
B. Facilitate evaluation of the performance of the nurse
C. Ensure learning on the part of the nurse
D. Ensure communication among staff members
88. Mang Carlos has been terminally ill for 5 years. He asked his wife to decide for him when he is no longer capable to do so. As a
Nurse, You know that this is called:
A. Last will and testament
B. DNR
C. Living will
D. Durable Power of Attorney
89. Which of the following behavior of Mang Carlos least indicates readiness to learn?
A. Talking with the nurse in charge and doctor
B. Reading brochures and pamphlets about diabetes
C. Inquiring about date of discharge
D. Asking question about diabetes milletus
90. Which of the following behaviors best contribute to the learning of Mang Carlos regarding his disease condition?
A. Drawing him into discussion about diabetes
B. Frequent use of technical terms
C. Loosely structured teaching session
D. Detailed lengthy explanation
SITUATION : Mrs. Purificacion is now the Chief nurse of a hospital in Manila. She is carefully reviewing different management styles
and theories that will best help her in running the nursing services in the hospital.
91. Which leadership style best empower the staff towards excellence?
A. Autocratic
B. Situational
C. Democratic
D. Laissez Faire
92. As a Nurse Manager, DSJ enjoys his staff of talented and self motivated individuals. He knew that the leadership style to suit
the needs of this kind of people is called:
A. Autocratic
B. Participative
C. Democratic
D. Laissez Faire
93. A fire has broken in the unit of DSJ R.N. The best leadership style suited in cases of emergencies like this is:
A. Autocratic
B. Participative
C. Democratic
D. Laissez Faire
94. Mrs. Purificacion is thinking of introducing the Primary Nursing Model Approach. You understand that this nursing model is:
A. The nurse manager assigns tasks to the staff members
B. Critical paths are used in providng nursing care
C. A single registered nurse is responsible for planning and providing individualized nursing care
D. Nursing staff are led by an RN leader in providing care to a group of clients
95. Structure, Process and Outcome are components of which step of the management process?
A. Planning
B. Organizing
C. Directing
D. Controlling
SITUATION : Nursing research is the term used to describe the evidence used to support nursing practice. Nursing, as an evidence
based area of practice, has been developing since the time of Florence Nightingale to the present day, where many nurses now work
as researchers based in universities as well as in the health care setting.
96. Mr. DSJ Plans to undertake a research of Community 1 and 2 on how they manage their health using Primary health care after
an organization and training seminars. This type of research is:
A. Experimental
B. Historical
C. Descriptive
D. Basic
97. The independent variable is:
A. Primary Health Care
B. Community 1 and 2
C. Organization and training seminars
D. Management of their health
98. In this design, the variable that is being manipulated is
1. Independent
2. Organization and training seminars
3. Dependent
4. Management of Primary Health Care
A. 1,2
B. 1,4
C. 2,3
D. 3,4
99. In general, the research process follows the ff. ordered sequence:
1. Determination of design
2. Statement of the problem
3. Definition of variables
4. Collection and analysis of data
5. Review of related literature
A. 2,5,3,1,4
B. 3,5,4,1,2
C. 2,5,3,4,1
D. 2,5,1,3,4
100. Studies done in natural setting such as this one, posses difficulty of controlling which variable?
A. Independent
B. Dependent
C. Extraneous
D. Organismic
Answers
Here are the answers for the exam. Unfortunately, rationales are not given. If you need clarifications or disputes, please direct them
to the comments section and well be glad to give you an explanation.
1. A. Nursing care plan
2. C. Overt data
3. A. Ascertain the patients response to health problems
4. D. The patients statement of I feel sick to my stomach
5. C. Focused assessments are usually ongoing and concerning specific problems
6. B. Objective data from a secondary source
7. C. The patient will identify 5 high-salt foods from prepared list by discharge
8. D. Goal partially met
9. C. Terminal
10. C. Critique the steps involved in the development of the goal
11. C. Upon admission
12. D. Hospital Wellness Center
13. D. The health status of the patient is constantly changing and the nurse must be cognizant and responsive to these changes
14. D. All of the above
15. A. Health Maintenance Organization
16. A. Fear related to the potential risk and outcome of surgery
17. B. High risk for aspiration
18. B. Respirations
19. C. Possible body image disturbance
20. A. Clear the clients airway
21. B. No, because she isnt sexually active
22. B. Inspection, auscultation, percussion, palpation
23. B. Have a mammogram annually
24. D. Examine skin once per month, looking for suspicious lesions or changes in moles
25. A. Primary prevention
26. A. Gluteal muscle using Z-Track technique
27. D. Allergy to iodine or shellfish
28. C. Normal saline
29. C. Breath sounds
30. D. Puncture the finger as deeply as possible
31. D. Pulmonary edema
32. C. Lower IV to check for return flow
33. C. 10 drops per minute
34. B. Fluid overload
35. A. Hyperglycemia
36. C. Assess the clients knowledge and needs
37. B. This laboratory test gives an indication of glycemic control over the last 3 months
38. C. Psychomotor
39. B. Motivation to be symptom free
40. A. Planning
41. D. Sister Callista Roy
42. C. Martha Rogers
43. B. Madeleine Leininger
44. B. Italy, May 12, 1820
45. D. Eudaemonistic Model
46. A. Increase oxygen in the tissues and cells
47. B. Cardiac Irregularities
48. B. The internal diameter of the tube
49. B. Withdrawing of the suction catheter
50. A. Separate the upper and lower airway
51. A. Primary intention
52. A. Brought together by sutures, tapes or staples
53. C. Vitamin C
54. C. Purulent drainage
55. C. Wash hands thoroughly
56. A. Psychological preparation of the client
57. C. Circumscribed, elevated, and filled with serous fluid
58. C. The symptomatic quadrant last
59. D. Medial aspect of the dorsum of the foot
60. B. Respiratory quality
61. A. touch the back of the clients throat with a tongue depressor
62. A. Vesicular
63. C. Observe the night nurse administering a feeding
64. A. Palpate the surrounding are for crepitus
65. C. Small liver with a rough surface
66. D. Diagnosing, treating, prescribing medication and doing minor surgery
67. A. 1,3
68. A. Both a professional body and a labor union
69. A. 20 units
70. B. Examination for civil service eligibility
71. B. 30 days
72. C. Because the patients right coincide with the nurses responsibility
73. D. Those involved in medical mission whos services are for free
74. B. Can be revoked by the Board of Nursing
75. C. 3 years
76. D. The lawyer
77. C. May 2007
78. B. RA 9173
79. C. Prudence
80. C. Mitigating
81. D. NON
82. A. Not universally accepted abbreviation
83. C. Vital signs 130/70; 84; 20
84. A. Written descriptive nurses notes
85. A. Kardex
86. B. Mang Carlos sister will be able to determine his insulin requirement based on blood glucose levels obtained from
glucometer in two days
87. A. Document the quality of care
88. D. Durable Power of Attorney
89. C. Inquiring about date of discharge
90. C. Loosely structured teaching session
91. C. Democratic
92. D. Laissez Faire
93. A. Autocratic
94. C. A single registered nurse is responsible for planning and providing individualized nursing care
95. D. Controlling
96. A. Experimental
97. C. Organization and training seminars
98. A. 1,2
99. A. 2,5,3,1,4
100. C. Extraneous







Fundamentals of Nursing Exam 16 (85 Items)
InText Mode: All questions and answers are given for reading and answering at your own pace. You can also copy this exam and
make a print out.
Situation 1: Nursing is a profession. The nurse should have a background on the theories and foundation of nursing as it influenced
what is nursing today.
1. Nursing is the protection, promotion and optimization of health and abilities, prevention of illness and injury, alleviation of
suffering through the diagnosis and treatment of human response and advocacy in the care of the individuals, families,
communities and population. This is the most accepted definition of nursing as defined by the:
A. PNA
B. ANA
C. Nightingale
D. Henderson
2. Advancement in Nursing leads to the development of the Expanded Career Roles. Which of the following is NOT expanded
career role for nurse?
A. Nurse practitioner
B. Clinical Nurse Specialist
C. Nurse Researcher
D. Nurse anaesthesiologist
3. The Board of Nursing regulated the Nursing profession in the Philippines and is responsible for the maintenance of the quality
of nursing in the country. Powers and duties of the board of nursing are the following EXCEPT:
A. Issue, suspend revoke certificates of registration
B. Issue subpoena duces tecum, ad testificandum
C. Open and close colleges of nursing
D. Supervise and regulate the practice
4. A nursing student or a beginning staff nurse who has not yet experienced enough in a situation to make judgments about them
is in what stage of Nursing Expertise?
A. Novice
B. Newbie
C. Advanced Beginner
D. Competent
5. Benners Proficient nurse level is different from the other levels in nursing expertise in the context of having:
A. The ability to organize and plan activities
B. Having attained an advanced level of education
C. A holistic understanding and perception of the client
D. Intuitive and analytic ability in new situations
Situation 2: The nurse has been asked to administer an injection via Z TRACK technique. Questions 6 to 10 refer this.
6. The nurse prepares an IM injection for an adult client using the Z track techniques, 4 ml of medication is to be administered to
the client. Which of the following site will you choose?
A. Deltoid
B. Rectus Femoris
C. Ventrogluteal
D. Vastus lateralis
7. In infants 1 year old and below, which of the following is the site of choice in intramuscular injection?
A. Deltoid
B. Rectus Femoris
C. Ventrogluteal
D. Vastus lateralis
8. In order to decrease discomfort in Z track administration, which of the following should be done?
A. Pierce the skin quickly and smoothly at 90 degree angle
B. Inject the medication at around 10 minutes per millilitre
C. Pull back the plunger and aspirate for 1 minute t make sure that the needle did not hit a blood vessel
D. Pierce the skin slowly and carefully at a 90 degree angle
9. After injection using the Z track technique, the nurse should know that she needs to wait for few second before withdrawing
the needle and this is to allow the medication to disperse into the muscle tissue thus decreasing the clients discomfort. How
many seconds should the nurse wait before withdrawing the needle?
A. 2 second
B. 5 seconds
C. 10 seconds
D. 15 seconds
10. The rationale in using the Z track technique in an intramuscular injection is:
A. It decreases the leakage of discolouring and irritating medication into the subcutaneous tissue.
B. It will allow a faster absorption of the medication
C. The Z track technique prevent irritation of the muscle
D. It is much more convenient for the nurse
Situation 3: A client was rushed to the emergency room and you are his attending nurse. You are performing a vital sign
assessment:
11. All of the following are correct methods in assessment of the blood pressure EXCEPT:
A. Take the blood pressure reading on both arms for comparison
B. Listen to and identify the phases of Korotkoff sound
C. Pump the cuff to around 50mmHg above the point where the pulse is obliterated
D. Observe procedures for infection control
12. You attached a pulse oximeter to the client. You know that the purpose id to:
A. Determine if the clients hemoglobin level is low and if he needs blood transfusion
B. Check level of clients tissue perfusion
C. Measure the efficacy of the clients anti-hypertension medications
D. Detect oxygen saturation of arterial blood before symptoms of hypoxemia develops
13. After a few hours in the Emergency Room, the client is admitted to the ward with an order of hourly monitoring of blood
pressure. The nurse finds that the cuff is too narrow and this will cause the blood pressure reading to be:
A. Inconsistent
B. Low systolic and high diastolic
C. Higher than what the reading should be
D. Lower than what the reading should be
14. Through the clients health history, you gather that the patient smokes and drinks coffee. When taking the blood pressure of a
client who recently smoked or drank coffee, how long should the nurse wait before taking the clients blood pressure for accurate
reading?
A. 15 minutes
B. 30 minutes
C. 1 hour
D. 5 minutes
15. While the client has pulse oximeter on his fingertip, you notice that the sunlight is shining on the area where the oximeter is.
Your action will be to:
A. Set and turn on the alarm of the oximeter
B. Do nothing since there is no identified problem
C. Cover the fingertip sensor with a towel or bedsheet
D. Change the location of the sensor every four hours
16. The nurse finds it necessary to recheck the blood pressure reading. In case of such reassessment, the nurse should wait for a
period of:
A. 15 seconds
B. 1 to 2 minutes
C. 30 minutes
D. 15 minutes
17. If the arm is said to be elevated when taking the blood pressure. It will create a:
A. False high reading
B. False low reading
C. True False reading
D. Indeterminate
18. You are to assessed the temperature of the client the next morning and found out that he ate ice cream. How many minutes
should you wait before assessing the clients oral temperature?
A. 10 minutes
B. 20 minutes
C. 30 minutes
D. 15 minutes
19. When auscultating the clients blood pressure the nurse hears the following: From 150 mmHg to 130 mmHg: Silence, Then: a
thumping sound continuing down to 100 mmHg: muffled sound continuing down to 80 mmHg and then silence. What is the
clients pressure?
A. 130/80
B. 150/100
C. 100/80
D. 150/100
20. In a client with a previous blood pressure of 130/80 4 hours ago, how long will it take to release the blood pressure cuff to
obtain an accurate reading?
A. 10 20 seconds
B. 30 45 seconds
C. 1 1.5 minutes
D. 3 3.5 minutes
Situation 4 Oral care is an important part of hygienic practices and promoting client comfort.
21. An elderly client, 84 years old, is unconscious. Assessment of the mouth reveals excessive dryness and presence of sores.
Which of the following is BEST to use for oral care?
A. lemon glycerine
B. hydrogen peroxide
C. Mineral oil
D. Normal saline solution
22. When performing oral care to an unconscious client, which of the following is a special consideration to prevent aspiration of
fluids into the lungs?
A. Put the client on a side lying position with head of bed lowered
B. Keep the client dry by placing towel under the chin
C. Wash hands and observe appropriate infection control
D. Clean mouth with oral swabs in a careful and an orderly progression
23. The advantages of oral care for a client include all of the following, EXCEPT:
A. decreases bacteria in the mouth and teeth
B. reduces need to use commercial mouthwash which irritate the buccal mucosa
C. improves clients appearance and self-confidence
D. improves appetite and taste of food
24. A possible problem while providing oral care to unconscious clients is the risk of fluid aspiration to lungs. This can be avoided
by:
A. Cleaning teeth and mouth with cotton swabs soaked with mouthwash to avoid rinsing the buccal cavity
B. swabbing the inside of the cheeks and lips, tongue and gums with dry cotton swabs
C. use fingers wrapped with wet cotton washcloth to rub inside the cheeks, tongue, lips and ums
D. suctioning as needed while cleaning the buccal cavity
25. Your client has difficulty of breathing and is mouth breathing most of the time. This causes dryness of the mouth with
unpleasant odor. Oral hygiene is recommended for the client and in addition, you will keep the mouth moistened by using:
A. salt solution
B. water
C. petroleum jelly
D. mentholated ointment
Situation 5: Ensuring safety before, during and after a diagnostic procedure is an important responsibility of the nurse.
26. To help Fernan better tolerate the bronchoscopy, you should instruct him to practice which of the following prior to the
procedure:
A. Clenching his fist every 2 minutes
B. Breathing in and out through the nose with his mouth open
C. Tensing the shoulder muscles while lying on his back
D. Holding his breath periodically for 30 seconds
27. Following a bronchoscopy, which of the following complains to Fernan should be noted as a possible complication:
A. Nausea and vomiting
B. Shortness of breath and laryngeal stridor
C. Blood tinged sputum and coughing
D. Sore throat and hoarseness
28. Immediately after bronchoscopy, you instructed Fernan to:
A. Exercise the neck muscles
B. Breathe deeply
C. Refrain from coughing and talking
D. Clear his throat
29. Thoracentesis may be performed for cytologic study of pleural fluid. As a nurse your most important function during the
procedure is to:
A. Keep the sterile equipment from contamination
B. Assist the physician
C. Open and close the three-way stopcock
D. Observe the patients vital signs
30. Right after thoracentesis, which of the following is most appropriate intervention?
A. Instruct the patient not to cough or deep breathe for two hours
B. Observe for symptoms of tightness of chest or bleeding
C. Place an ice pack to the puncture site
D. Remove the dressing to check for bleeding
Situation 6: Knowledge of the acid base disturbance and the functions of the electrolytes is necessary to determine appropriate
intervention and nursing actions.
31. A client with diabetes mellitus has glucose level of 644 mg/dL. The nurse interprets that this client is at most risk for the
involvement at which type of acid base imbalance?
A. Respiratory acidosis
B. Respiratory alkalosis
C. Metabolic acidosis
D. Metabolic alkalosis
32. In a client in the health care clinic, arterial blood gas analysis gives the following results: pH 7.48, PCO2 32mmHg, PO2 94
mmHg, HCO3 24 mEq/L. The nurse interprets that the client has which acid base disturbance?
A. Respiratory acidosis
B. Respiratory alkalosis
C. Metabolic acidosis
D. Metabolic alkalosis
33. A client has an order for ABG analysis on radial artery specimens. The nurse ensures that which of the following has been
performed or tested before the ABG specimen are drawn?
A. Guthing test
B. Allens test
C. Rombergs test
D. Webers test
34. A nurse is reviewing the arterial blood gas values of a client and notes that the pH is 7.31, Pco2 is 500 mmHg, and the
bicarbonate is 27 mEq/L. The nurse concludes that which acid base disturbance is present in this client?
A. Respiratory acidosis
B. Respiratory alkalosis
C. Metabolic acidosis
D. Metabolic alkalosis
35. Allens test checks the patency of the:
A. Ulnar artery
B. Radial artery
C. Carotid artery
D. Brachial artery
36. (skip) (Pretend Im not here.)
37. After IVP a renal stone was confirmed, a left nephrectomy was done. Her post operative order includes daily urine specimen
to be sent to the laboratory. Eileen has a foley catheter attached to a urinary drainage system. How will you collect the urine
specimen?
A. remove urine from drainage tube with sterile needle and syringe and empty urine from the syringe into the specimen container
B. empty a sample urine from the collecting bag into the specimen container
C. disconnect the drainage tube from the indwelling catheter and allow urine to flow from catheter into the specimen container.
D. disconnect the drainage the from the collecting bag and allow the urine to flow from the catheter into the specimen container.
38. Where would the nurse tape Eileens indwelling catheter in order to reduce urethral irritation?
A. to the patients inner thigh
B. to the patients lower thigh
C. to the patients buttocks
D. to the patient lower abdomen
39. Which of the following menu is appropriate for one with low sodium diet?
A. instant noodles, fresh fruits and ice tea
B. ham and cheese sandwich, fresh fruits and vegetables
C. white chicken sandwich, vegetable salad and tea
D. canned soup, potato salad, and diet soda
40. How will you prevent ascending infection to Eileen who has an indwelling catheter?
A. see to it that the drainage tubing touches the level of the urine
B. change he catheter every eight hours
C. see to it that the drainage tubing does not touch the level of the urine
D. clean catheter may be used since urethral meatus is not a sterile area
Situation 7: Hormones are secreted by the various glands in the body. Basic knowledge of the endocrine system is necessary.
41. Somatotropin or the Growth Hormone releasing hormone is secreted by the anterior pituitary gland:
A. Hypothalamus
B. Anterior pituitary gland
C. Posterior pituitary gland
D. Thyroid gland
42. All of the following are secreted by the anterior pituitary gland except:
A. Somatotropin/Growth hormone
B. Follicle stimulating hormone
C. Thyroid stimulating hormone
D. Gonadotropin hormone releasing hormone
43. All of the following hormones are hormones secreted by the Posterior pituitary gland except:
A. Vasopressin
B. Oxytocin
C. Anti-diuretic hormone
D. Growth hormone
44. Calcitonin, a hormone necessary for calcium regulation is secreted in the:
A. Thyroid gland
B. Hypothalamus
C. Parathyroid gland
D. Anterior pituitary gland
45. While Parathormone, a hormone that regulates the effect of calcitonin is secreted by the:
A. Thyroid gland
B. Hypothalamus
C. Parathyroid gland
D. Anterior pituitary gland
Situation 8 The staff nurse supervisor requests all the staff nurses to brainstorm and learn ways to instruct diabetic clients on
self-administration of insulin. She wants to ensure that there are nurses available daily to do health education classes.
46. The plan of the nurse supervisor is an example of
A. in service education process
B. efficient management of human resources
C. increasing human resources
D. primary prevention
47. When Mrs. Guevarra, a nurse, delegates aspects of the clients care to the nurse-aide who is an unlicensed staff, Mrs. Guevarra
A. makes the assignment to teach the staff member
B. is assigning the responsibility to the aide but not the accountability for those tasks
C. does not have to supervise or evaluate the aide
D. most know how to perform task delegated
48. Connie, the new nurse, appears tired and sluggish and lacks the enthusiasm she had six weeks ago when she started the job.
The nurse supervisor should
A. empathize with the nurse and listen to her
B. tell her to take the day off
C. discuss how she is adjusting to her new job
D. ask about her family life
49. Process of formal negotiations of working conditions between a group of registered nurses and employer is
A. grievance
B. arbitration
C. collective bargaining
D. strike
50. You are attending a certification on cardiopulmonary resuscitation (CPR) offered and required by the hospital employing you.
This is
A. professional course towards credits
B. inservice education
C. advance training
D. continuing education
Situation 9: As a nurse, you are aware that proper documentation in the patient chart is your responsibility.
51. Which of the following is NOT a legally binding document but nonetheless very important in the care of all patients in any
setting?
A. Bill of rights as provided in the Philippine Constitution
B. Scope of nursing practice as defined in R.A. 9173
C. Board of Nursing resolution adopting the Code of Ethics
D. Patients Bill of Rights
52. A nurse gives a wrong medication to the client. Another nurse employed by the same hospital as a risk manager will expect to
receive which of the following communication?
A. Incident Report
B. Oral report
C. Nursing kardex
D. Complain report
53. Performing a procedure on a client in the absence of an informed consent can lead to which of the following charges?
A. Fraud
B. Assault and Battery
C. Harassment
D. Breach of confidentiality
54. Which of the following is the essence of informed consent?
A. It should have a durable power of attorney
B. It should have coverage from an insurance company
C. It should respect the clients freedom from coercion
D. It should discloses previous diagnosis, prognosis and alternative treatments available for the client.
55. Delegation is the process of assigning tasks that can be performed by a subordinate. The RN should always be accountable
and should not lose his accountability. Which of the following is a role included in delegation?
A. The RN must supervise all delegated tasks
B. After a task has been delegated. It is no longer a responsibility of the RN.
C. The RN is responsible and accountable for the delegated task in a adjunct with the delegate.
D. Follow up with a delegated task necessary only if the assistive personnel is not trustworthy.
Situation 10 When creating your lesson plan for cerebrovascular disease or STROKE. It is important to include the risk factors of
stroke.
56. The most important risk factor is:
A. Cigarette smoking
B. Hypertension
C. binge drinking
D. heredity
57. Part of your lesson plan is to talk about etiology or cause of stroke. The types of stroke based on cause are the following
EXCEPT:
A. Embolic stroke
B. Hemorrhagic stroke
C. diabetic stroke
D. thrombotic stroke
58. Hemmorhagic stroke occurs suddenly usually when the person is active. All are causes of hemorrhage, EXCEPT:
A. phlebitis
B. trauma
C. damage to blood vessel
D. aneurysm
59. The nurse emphasizes that intravenous drug abuse carries a high risk of stroke. Which drug is closely linked to this?
A. Amphetamines
B. Cocaine
C. shabu
D. Demerol
60. A participant in the STROKE class asks what is a risk factor of stroke. Your best response is:
A. More red blood cells thicken blood and make clots more possible.
B. Increased RBC count is linked to high cholesterol.
C. More red blood cell increases hemoglobin content.
D. High RBC count increases blood pressure.
Situation 11: Recognition of normal values is vital in assessment of clients with various disorders.
61. A nurse is reviewing the laboratory test results for a client with a diagnosis of severe dehydration. The nurse would expect the
hematocrit level for this client to be which of the following?
A. 60%
B. 47%
C. 45%
D. 32%
62. A nurse is reviewing the electrolyte results of an assigned client and notes that the potassium level is 5.6 mEq/L. Which of the
following would the nurse expect to note on the ECG as a result of this laboratory value?
A. ST depression
B. Inverted t wave
C. Prominent U wave
D. Tall peaked T waves
63. A nurse is reviewing the electrolyte results of an assigned client and notes that the potassium level is 3.2 mEq/L. Which of the
following would the nurse expect to note on the ECG as a result of this laboratory value?
A. U waves
B. Absent P waves
C. Elevated T waves
D. Elevated ST segment
64. Dorothy underwent diagnostic test and the result of the blood examination are back. On reviewing the result the nurse
notices which of the following as abnormal finding?
A. Neutrophils 60%
B. White blood cells (WBC) 9000/mm
C. Erythrocyte sedimentation rate (ESR) is 39 mm/hr
D. Iron 75 mg/100 ml
65. Which of the following laboratory test result indicate presence of an infectious process?
A. Erythrocyte sedimentation rate (ESR) 12 mm/hr
B. White blood cells (WBC) 18,000/mm3
C. Iron 90 g/100ml
D. Neutrophils 67%
Situation 12: Pleural effusion is the accumulation of fluid in the pleural space.
66. Which of the following is a finding that the nurse will be able to assess in a client with pleural effusion?
A. Reduced or absent breath sound at the base of the lungs, dyspnea, tachypnea and shortness of breath.
B. Hypoxemia
C. Noisy respiration, crackles, stridor and wheezing
D. Tracheal deviation towards the affected side, increased fremitus and loud breath sounds
67. Thoracentesis is performed to the client with effusion. The nurse knows that he removal of fluid should be slow. Rapid
removal of fluid in thoracentesis might cause:
A. Pneumothorax
B. Pleurisy or Pleuritis
C. Cardiovascular collapse
D. Hypertension
68. Three days after thoracentesis, the client again exhibited respiratory distress. The nurse will know that the pleural effusion
has reoccurred when she noticed a sharp stabbing pain during inspiration. The physician ordered a closed tube thoracotomy for
the client. The nurse knows that the primary function of the chest tube is to:
A. Restore positive intrathoracic pressure
B. Restore negative intrathoracic pressure
C. To visualize the intrathoracic content
D. As a method of air administration
69. The chest tube is functioning properly if:
A. There is an oscillation
B. There is no bubbling in the drainage bottle
C. There is a continuous bubbling in the water seal.
D. The suction control bottle has a continuous bubbling
70. In a client with pleural effusion, the nurse is instructing a appropriate breathing technique. Which of the following is included
in the teaching?
A. Breath normally
B. Hold the breath after each inspiration for 1 full minute
C. Practice abdominal breathing
D. Inhale slowly and hold the breath for 3-5 seconds after each inhalation.
Situation 13: Health care delivery system affects the health status of every Filipino. As a Nurse, Knowledge of this system is expected
to ensure quality of life.
71. When should rehabilitation commence?
A. The day before discharge
B. When the patient desires
C. Upon admission
D. 24hours after discharge
72. What exemplified the preventive and promotive programs in the hospital?
A. Hospitals as a center to prevent and control infection
B. Program for smokers
C. Program for alcoholics and drug addicts
D. Wellness Center
73. Which makes nursing dynamic?
A. Every patient is a unique physical, emotional, social and spiritual being
B. The patient participate in the over all nursing care plan
C. Nursing practice is expanding in the light of modern development that takes place
D. The health status of the patient is constantly changing and the nurse must be cognizant and responsive to these changes.
74. Prevention is an important responsibility of the nurse in:
A. Hospitals
B. Community
C. Workplace
D. All of the above
75. This form of Health Insurance provides comprehensive prepaid health services to enrollees for a periodic payment.
A. Health Maintenance Organization
B. Medicare
C. Philippine Health Insurance Act
D. Hospital Maintenance Organization
76. Health care reports have different purposes. The availability of patients record to all health ream members demonstrates
which of the following purposes:
A. Legal documentation
B. Education
C. Research
D. Vehicle for communication
77. When a nurse commits medication error she should accurately document clients response and her corresponding action. This
is very important for which of the following purposes:
A. Research
B. Nursing Audit
C. Legal documentation
D. Vehicle for communication
78. POMR has been widely used in many teaching hospitals. One of its unique features is SOAPIE charting. The P in SOAPIE
charting should include:
A. Prescription of the doctor to the patients illness
B. Plan of care for patient
C. Patients perception of ones illness
D. Nursing Problem and Nursing Diagnosis
79. The medical records that are organized into separate section from doctors or nurses has more disadvantages than advantages.
This is classified as what type of recording?
A. POMR
B. SOAPIE
C. Modified POMR
D. SOMR
80. Which of the following is the advantage of SOMR or Traditional Recording?
A. Increase efficiency of Data gathering
B. Reinforces the use of the nursing process
C. The caregiver can easily locate proper section for making charting entries
D. Enhances effective communication among health care team members
Situation 17: June is 24 year old client with symptoms of dyspnea, absent breath sounds on the right lung and chest X-ray revealed
pleural effusion. The physician will perform thoracentesis
81. Thoracentesis is useful in treating which of the following pulmonary disorders except:
A. Hemothorax
B. Tuberculosis
C. Hydrothorax
D. Empyema
82. Which of the following psychological preparation is not relevant for him?
A. Telling him that the gauge of the needle and anesthesia to be used
B. Telling him to keep still during the procedure to facilitate the insertion of the needle in the correct place.
C. Allow June to express his feeling and concerns
D. Physicians explanation on the purpose of the procedure and how it will be done.
83. Before thoracentesis, the legal consideration you must check is:
A. Consent is signed by the client
B. Medicine preparation is correct
C. Position of the client is correct
D. Consent is signed by relative and physician
84. As a nurse, you know that the position for June before thoracentesis is:
A. Orthopneic
B. Knee-chest
C. Low fowlers
D. Sidelying position on the affected side
85. Which of the following anesthetic drug is used for thoracentesis?
A. Procaine 2 %
B. Valium 250 mg
C. Demerol 75 mg
D. Phenobarbital
Answers
Here are the answers for the exam. Unfortunately, rationales are not given. If you need clarifications or disputes, please direct them
to the comments section and well be glad to give you an explanation.
1. B. ANA
2. D. Nurse anaesthesiologist
3. C. Open and close colleges of nursing
4. A. Novice
5. C. A holistic understanding and perception of the client
6. C. Ventrogluteal
7. D. Vastus lateralis
8. A. Pierce the skin quickly and smoothly at 90 degree angle
9. C. 10 seconds
10. A. It decreases the leakage of discolouring and irritating medication into the subcutaneous tissue.
11. C. Pump the cuff to around 50 mmHg above the point where the pulse is obliterated
12. D. Detect oxygen saturation of arterial blood before symptoms of hypoxemia develops
13. C. Higher than what the reading should be
14. B. 30 minutes
15. C. Cover the fingertip sensor with a towel or bed sheet
16. B. 1 to 2 minutes
17. B. False low reading
18. C. 30 minutes
19. A. 130/80
20. C. 1 1.5 minutes
21. D. Normal saline solution
22. A. Put the client on a side lying position with head of bed lowered
23. B. reduces need to use commercial mouthwash which irritate the buccal mucosa
24. D. suctioning as needed while cleaning the buccal cavity
25. C. petroleum jelly
26. B. Breathing in and out through the nose with his mouth open
27. B. Shortness of breath and laryngeal stridor
28. B. Breathe deeply
29. D. Observe the patients vital signs
30. B. Observe for symptoms of tightness of chest or bleeding
31. A. Respiratory acidosis
32. B. Respiratory alkalosis
33. B. Allens test
34. A. Respiratory acidosis
35. B. Radial artery
36. (skip)
37. A. remove urine from drainage tube with sterile needle and syringe and empty urine from the syringe into the specimen
container
38. A. to the patients inner thigh
39. C. white chicken sandwich, vegetable salad and tea
40. C. see to it that the drainage tubing does not touch the level of the urine
41. A. Hypothalamus
42. D. Gonadotropin hormone releasing hormone
43. D. Growth hormone
44. A. Thyroid gland
45. C. Parathyroid gland
46. B. efficient management of human resources
47. B. is assigning the responsibility to the aide but not the accountability for those tasks
48. C. discuss how she is adjusting to her new job
49. C. collective bargaining
50. B. in-service education
51. D. Patients Bill of Rights
52. A. Incident Report
53. B. Assault and Battery
54. D. It should discloses previous diagnosis, prognosis and alternative treatments available for the client.
55. C. The RN is responsible and accountable for the delegated task in a adjunct with the delegate.
56. B. Hypertension
57. C. diabetic stroke
58. A. phlebitis
59. B. Cocaine
60. A. More red blood cells thicken blood and make clots more possible.
61. A. 60%
62. D. Tall peaked T waves
63. A. U waves
64. C. Erythrocyte sedimentation rate (ESR) is 39 mm/hr
65. B. White blood cells (WBC) 18,000/mm3
66. A. Reduced or absent breath sound at the base of the lungs, dyspnea, tachypnea and shortness of breath.
67. C. Cardiovascular collapse
68. B. Restore negative intrathoracic pressure
69. A. There is an oscillation
70. D. Inhale slowly and hold the breath for 3-5 seconds after each inhalation.
71. C. Upon admission
72. D. Wellness Center
73. D. The health status of the patient is constantly changing and the nurse must be cognizant and responsive to these changes.
74. D. All of the above
75. A. Health Maintenance Organization
76. D. Vehicle for communication
77. C. Legal documentation
78. B. Plan of care for patient
79. D. SOMR
80. C. The caregiver can easily locate proper section for making charting entries
81. B. Tuberculosis
82. A. Telling him that the gauge of the needle and anesthesia to be used
83. A. Consent is signed by the client
84. A. Orthopneic
85. A. Procaine 2 %

Anda mungkin juga menyukai